Anda di halaman 1dari 102

Problemas Resolvidos

de Eletromagnetismo
Prof. Luiz Antonio Righi
SUM

ARIO
1. Sistemas de coordenadas, carga e corrente eletrica . . . . . . . . . . . . . . . . . . . . . . . . 3
2. Lei de Coulomb . . . . . . . . . . . . . . . . . . . . . . . . . . . . . . . . . . . . . . . . . . . . 9
3. Campo e potencial eletrico . . . . . . . . . . . . . . . . . . . . . . . . . . . . . . . . . . . . . 16
4. Lei de Gauss e Capacitancia . . . . . . . . . . . . . . . . . . . . . . . . . . . . . . . . . . . . . 23
5. Campo magnetico . . . . . . . . . . . . . . . . . . . . . . . . . . . . . . . . . . . . . . . . . . 32
6. Fluxo, inducao e forca magnetica . . . . . . . . . . . . . . . . . . . . . . . . . . . . . . . . . . 38
7. Indutancia e circuitos magneticos . . . . . . . . . . . . . . . . . . . . . . . . . . . . . . . . . . 47
8. Fasores . . . . . . . . . . . . . . . . . . . . . . . . . . . . . . . . . . . . . . . . . . . . . . . . 58
9. Campos Dinamicos - Lei de Faraday . . . . . . . . . . . . . . . . . . . . . . . . . . . . . . . . 62
10. Corrente de Deslocamento . . . . . . . . . . . . . . . . . . . . . . . . . . . . . . . . . . . . . . 71
11. Ondas eletromagneticas . . . . . . . . . . . . . . . . . . . . . . . . . . . . . . . . . . . . . . . 79
12. Linhas de Transmissao . . . . . . . . . . . . . . . . . . . . . . . . . . . . . . . . . . . . . . . . 90
13. Carta de Smith . . . . . . . . . . . . . . . . . . . . . . . . . . . . . . . . . . . . . . . . . . . . 95
14. Equacoes de Maxwell . . . . . . . . . . . . . . . . . . . . . . . . . . . . . . . . . . . . . . . . . 100
1. SISTEMAS DE COORDENADAS, CARGA E CORRENTE EL

ETRICA
PR 1.1: Explicar a diferenca entre submeter um BOM condutor e um BOM isolante (mau condutor)
num campo eletrico.
Resposta: O bom condutor tem grande n umero de eletrons livres de conducao, e eles se deslocam
com determinada velocidade escalar de deriva, enquanto que o bom isolante tem baixssimo n umero de
eletrons livres, e quando submetido ao campo eletrico acontece a polarizacao das moleculas. Os atomos
mais positivos (Kations) deslocam-se no sentido do campo, e os mais negativos (

Anios), vice-versa. A
forca eletrica faz a molecula girar, segundo o sentido do campo externo.
PR 1.2: Fazer a analogia entre circuitos eletricos e hidraulicos, citando as grandezas fundamentais de
potencial e uxo.
PR 1.3: Um motor de corrente contnua de 10 CV solicita uma corrente de 40 A quando operado `a
plena carga ligado a uma rede de 220 V em corrente contnua (CC). Determinar: (a) o rendimento deste
motor; (b) qual a potencia perdida.
Solucao:
(a) Com o motor operando a plena carga (potencia nominal), a potencia na sada (no eixo do motor)
e P
S
= 10 736 = 7360W. A potencia na entrada (fornecida pela rede) e P
E
= 220 40 = 8800W. O
rendimento e = 83, 64%.
(b) As perdas no motor sao: P
PERDA
= P
E
P
S
= 1140W.
PR 1.4: Durante a operacao de descarga de uma bateria, a corrente eletrica foi de 50 A e a tensao 11,5
V. Sabendo que a taxa de transferencia de calor e de 30 W, qual e a taxa de diminuicao da energia
interna da bateria?
Solucao: Como as variacoes de energia potencial e cinetica nao sao signicativas, a equacao do
equilbrio energetico da bateria e:

Q =
dU
dt
+

W
onde

W = 50 11, 5 = 575W. Portanto
dU
dt
= 30 575 = 605 W.
PR 1.5: O que e Efeito Joule? Qual a equacao para a potencia e a energia de um resistor?
1. Sistemas de coordenadas, carga e corrente eletrica 4
PR 1.6: Quais sao as unidades de resistencia, diferenca de potencial e intensidade de corrente?
PR 1.7: Utilizando os dados disponveis em tabelas, calcule a resistencia de 1 m de um o de ferro
envolto com alumnio, se o diametro do n ucleo de ferro e 0.25 pol e o diametro externo e 0.50 pol. Se
o condutor transporta uma corrente contnua de 50 A, determine a potencia dissipada por polegada
quadrada de superfcie do condutor externo.
PR 1.8: O elemento de aquecimento de uma certa torradeira eletrica consiste de uma tira de certa
qualidade de Nicromo, cujo comprimento e 1.5 m e a secao reta mede 0.05 m por 0.8 mm, com uma
resistividade de 1.1E-4.cm. Encontre a corrente que circula no elemento quando ligamos entre os seus
terminais, uma fonte de 120 Vcc. Determine tambem a potencia dissipada.
PR 1.9: Em uma casa, abastecida com tensao de 110 V, seus moradores utilizam um chuveiro com duas
temperaturas: inverno e verao. Quando a chave esta acionada, o chuveiro trabalha com 5600 W. No
verao, opera com 3000 W. Qual e a diferenca de resistencia entre as duas faixas de temperatura? (R:
1,873 )
PR 1.10: Um chuveiro eletrico possui tres opcoes de conguracao: quente, morno e desligado. Na opcao
A, o aquecimento dagua se da por meio de uma resistencia de secao 1 mm
2
e comprimento de 2 m. Na
opcao B utiliza-se a mesma secao, porem com 1 m de comprimento. Considerando a resistividade de 1
mm
2
/m, pergunta-se: a) Qual a resistencia eletrica do chuveiro nas tres conguracoes? (R: 1 e 2
e innito (circuito aberto). b) Qual a potencia de cada opcao, sabendo que o chuveiro esta ligado em
110 V? (R:12100 W e 6050 W).
PR 1.11: Qual e a resistencia de uma lampada em cujo bulbo se le 60 W e 110 V?
PR 1.12: Por que as linhas de transmissao de energia a longas distancias operam sob altas tensoes?
PR 1.13: Um chuveiro eletrico submetido `a tensao constante, pode ser regulado para fornecer agua a
maior ou menor temperatura (inverno e verao respectivamente). A resistencia eletrica do chuveiro e
maior quando se deseja agua mais aquecida (inverno)? Por que?
PR 1.14: Um chuveiro eletrico foi construdo para operar sob a tensao de 110 V. Para opera-lo a uma
tensao de 220 V , sem modicar a potencia de aquecimento, de quanto deve-se alterar a sua resistencia?
PR 1.15: Suponhamos que se necessita construir uma resistencia eletrica de 500 ohm com um condutor
de comprimento 100 m. Qual o valor da queda de tensao em cada espira, sabendo-se que a corrente
total e 2 A e que cada espira possui 1 cm de diametro?
1. Sistemas de coordenadas, carga e corrente eletrica 5
PR 1.16: Ao realizar um experimento em laboratorio, um estudante submeteu um resistor a diversas
diferencas de potencial V , e para cada caso mediu a corrente eletrica i. Com esses dados tracou um
graco de V em funcao de i, onde os pontos lidos foram: Qual a resistencia eletrica desse resistor?
V(Volt) i(Ampere)
5 0,1
10 0,2
20 0,4
30 0,6
PR 1.17: Considerando um resistor de o com resistividade = 1/, comprimento , secao A, e re-
sistencia R, demonstrar a lei de Ohm sob a forma local, quando ele e percorrido pela corrente I, e possui
uma tensao ou d.d.p. V nos seus terminais.
Solucao: Partindo da lei de Ohm
R =
V
I
=

A
e, considerando que V = E e I = J A, encontra-se
E
J A
=

A
que simplicando os termos e A resulta:
=
E
J
ou

J =

E
C

PR 1.18: Demonstrar e explicar a equacao da refracao da corrente eletrica.
R: Vamos supor que a corrente passa de um meio 1 para um meio 2, com condutividades
1
e
2
,
respectivamente, onde e o angulo de

E ou

J com a normal nos meios 1 ou 2. No contorno de dois
materiais com condutividades diferentes
1
e
2
, o princpio da continuidade da corrente (a integral da
densidade de corrente numa superfcie fechada e igual a zero):
_

J d

S = 0
garante a continuidade da componente normal da densidade de corrente
J
n1
= J
n2
(1.1)
e o princpio da circulacao do campo eletrico nao conservativo (a integral de linha fechada e igual a
zero), pois a diferenca de potencial na fronteira e igual para os dois meios:
_

E d

= 0
garante a continuidade da componente tangencial do campo
E
t1
= E
t2
(1.2)
1. Sistemas de coordenadas, carga e corrente eletrica 6
ou
J
t1

1
=
J
t2

2
(1.3)
Dividindo (1.3) por (1.1) temos
J
t1

1
J
n1
=
J
t2

2
J
n2
ou
tan
1
tan
2
=

1

2
(1.4)

PR 1.19: Energia de uma bateria. Considere uma fonte de tensao contnua V , como uma bateria,
por exemplo. Certamente, Existe energia potencial eletrica armazenada? Calcular o seu valor, em
Joules, caso seja possvel.
Solucao: Com certeza, existe energia potencial, mas nao temos dados para calcular sua quantidade.
Vamos ver que a energia potencial e W = 0, 5QV J, ou seja, precisamos da quantidade de carga, ou da
corrente eletrica e do tempo.
PR 1.20: Explicar o que e, e estimar a velocidade escalar de deriva dos eletrons livres de conducao de
um o de cobre com A = 2, 5 mm
2
de secao, quando passa uma corrente de 1,3 A.
Resposta: Consideremos, como uma analogia, um grande n umero de pessoas correndo em direcoes
aleatorias, e empurrando-se constantemente umas contra as outras. Elas seriam os eletrons livres ou
eletrons de conducao. Se esta multidao estiver sobre uma superfcie inclinada (campo ou diferenca de
potencial eletrico), prosseguira lentamente no seu caminho segundo a direcao e o sentido da inclinacao.
Uma pequena, mas direcionada velocidade de deriva (corrente eletrica) sera superposta ao movimento
aleatorio dos empurroes (agitacao termica). No cobre, existe em media um eletron livre de conducao
por atomo. O n umero n de eletrons e, portanto igual ao n umero de atomos por unidade de volume.
n =
N
A

M
= 8, 47 10
28
eletrons/m
3
onde:
N
A
- n umero de avogadro (6, 02 10
23
eletrons/m
3
);
- densidade do cobre (9, 0 10
3
kg/m
3
); e,
M - massa molar do cobre (64 10
3
kg/mol).
A carga livre total num trecho de o de comprimento L e:
q = (n A L) e
e a velocidade de deriva
v
d
=
L
t
=
L
q/i
=
i
A
L
L n e
1. Sistemas de coordenadas, carga e corrente eletrica 7
v
d
=
J
ne
que para o nosso exemplo ca em torno de 14 cm/hora.
Obs.: cuidado para nao confundir velocidade escalar de deriva com velocidade de propagacao do
campo eletromagnetico, que e da ordem da velocidade da luz (c = 3 10
8
m/s. .
PR 1.21: Uma tira de silcio (semicondutor tipo n, dopado com uma impureza controlada de fosforo),
tem largura w = 3, 2 mm; espessura t = 250m; e, n = 1, 5 10
23
m
3
. Determinar: a) a densidade de
corrente na tira; e, b) a velocidade escalar de deriva.
Resposta:
J =
i
wt
= 6500 A/m
2
ou 0, 0065 A/mm
2
v
d
=
J
ne
= 0, 27 m/s
Este valor e muito menor do que no cobre, porque . . .
PR 1.22: Vetor deslocamento innitesimal de uma carga puntiforme Sejam duas cargas pun-
tiformes Q
1
= 5C e Q
2
= 3C, localizadas em P
1
= (1, 1, 1)m e P
2
= (2, 3, 4)m, respectivamente.
Qual e a energia necessaria para deslocar a carga Q
2
para o ponto P
3
, localizado no meio do segmento
de reta P
1
P
2
?
Solucao: Calculou-se a energia potencial em P
1
P
2
e P
1
P
3
U
12
= 0, 02647 J
U
13
= 0, 05295 J
logo, a energia de um agente externo, necessaria para deslocar Q
2
de P
2
para P
3
e 0, 02648 joules.
Pode-se calcular esta energia de uma maneira mais geral, calculando o trabalho ao longo do trecho
P
2
P
3
. Sabemos que o trabalho e o produto escalar do vetor forca pelo vetor deslocamento. E, quando
a forca nao e constante, precisamos usar a integral de linha, com o vetor deslocamento innitesimal d

.
O vetor deslocamento innitesimal, em coordenadas cartesianas, e sempre dado por:
d

= dx a
x
+dy a
y
+dz a
z
onde precisamos deixa-lo em termos de apenas uma variavel, que denominaremos t.
Fazendo t = 0 no ponto inicial, e t = 1 no ponto nal, podemos escrever a equacao da reta como
x = a
1
t +b
1
y = a
2
t +b
2
z = a
3
t +b
3
1. Sistemas de coordenadas, carga e corrente eletrica 8
PR 1.23: Tres cargas puntuais de 4C, localizam-se nos vertices de um triangulo eq uilatero de 0, 5mm,
situado no vacuo. Que trabalho deve ser realizado para deslocar uma das cargas ate o ponto medio do
segmento determinado pelas outras duas cargas? Sugestao: calcular a diferenca de potencial entre o
ponto nal e inicial (R: 575 J)
2. LEI DE COULOMB
PR 2.1: Lei da atracao das massas e das cargas. Sabendo que a distancia media entre o proton e
o eletron no atomo de hidrogenio e de 5, 3 10
11
m, compare as forcas eletrica e gravitacional.
Solucao: Considerando-se duas massas m
1
e m
2
, afastadas de uma distancia d, o modulo da forca
gravitacional vale
F
g
= 6, 67 10
11
m
1
m
2
d
2
Newton
F
g
= 6, 67 10
11
(9, 11 10
31
) (1, 67 10
27
)
(5, 3 10
11
)
2
F
g
= 3, 6 10
47
N.
O modulo da forca eletrica vale
F
e
= 9, 0 10
9
(1, 61 10
19
)
2
(5, 3 10
11
)
2
= 8, 2 10
8
N
A forca gravitacional e muitas vezes mais fraca do que a forca eletrostatica. Entretanto, a forca grav-
itacional e sempre atrativa, e pode agregar grandes massas, enquanto a eletrostatica pode ser repulsiva.

PR 2.2: Campo eletrico. Considere uma carga puntiforme Q


1
C, situada no ponto P
1
(x
1
, y
1
, z
1
) m e
outra carga puntiforme Q
2
C, situada no ponto P
2
(x
2
, y
2
, z
2
) m. Escrever a equacao do vetor campo
eletrico produzido por esta carga Q em qualquer ponto P(x, y, z) do espaco.
Solucao: Vamos supor que no ponto P tenhamos uma carga de prova q. Usando a Lei de Coulomb,
vamos escrever a equacao da forca entre as duas cargas Q
1
e q; e depois Q
2
e q. Para isto, podemos
seguir a seq uencia abaixo:
- Vetor deslocamento de P
1
ate P, que denominaremos vetor r
1
r
1
= (x x
1
) a
x
+ (y y
1
) a
y
+ (z z
1
) a
z
- Modulo ou intensidade do vetor deslocamento |r
1
| ou simplesmente r
1
|r
1
| = r
1
=
_
(x x
1
)
2
+ (y y
1
)
2
+ (z z
1
)
2
- Vetor unitario u
r1
u
r1
=
r
1
|r
1
|
2. Lei de Coulomb 10
- Vetor campo eletrico

E
1
, que e a forca dividida pela carga de prova q.

E
1
= k
Q
1
r
2
1
u
r1

E
1
= k Q
1
r
1
r
3
1
Da mesma forma obtemos

E
2
, e o campo resultante

E =

E
1
+

E
2

PR 2.3: Potencial eletrico de um dipolo. Considere um dipolo, formado por duas cargas pun-
tiformes +Q = 10C e Q = 10C, localizadas em +P = (1, 0, 0)m e P = (1, 0, 0)m, respectiva-
mente. Explicar o que e potencial eletrostatico, e escrever sua equacao no plano z = 1 metros, referente
ao dipolo?
Solucao: O potencial de 1 Volt e a energia potencial de 1 Joule para deslocar uma carga de prova
unitaria q = 1 Coulomb desde o innito ate o ponto de raio R. O potencial num ponto distante R de
uma carga puntiforme de fonte Q e:
W =
_
Fdr =
_
R

K
Q q
r
2
dr
V = K Q

1
r

V = K Q
1
R
O potencial resultante e a soma dos potenciais das cargas positiva e negativa.
V = V
+
+V


PR 2.4: Energia armazenada em cargas puntiformes. Demonstrar a equacao da energia ar-
mazenada em cargas puntiformes.
**Sistema com duas cargas puntiformes
Consideremos um sistema com duas cargas puntiformes Q
1
e Q
2
, situadas a uma distancia innita,
tal que nao exista forca em nenhuma carga, e elas estao em equilbrio. De acordo com a denicao de
potencial: uma intensidade de trabalho igual a Q
2
vezes o potencial de Q
1
deve ser aplicado sobre a
carga Q
2
e para aproxima-la de Q
1
. Entao a energia potencial do sistema e aumentada da quantia
W
2
= Q
2
V
1
2
onde V
1
2
e o potencial de Q
1
no local que o agente externo xou Q
2
.
**Sistema com tres cargas puntiformes
Consideremos, agora, um sistema com tres cargas puntiformes Q
1
, Q
2
e Q
3
, situadas a uma distancia
innita, tal que nao exista forca em nenhuma carga, e elas estejam em equilbrio. De acordo com a
2. Lei de Coulomb 11
denicao de potencial: uma intensidade de trabalho igual a Q
3
vezes o potencial de Q
1
, mais Q
3
vezes
o potencial de Q
2
deve ser aplicado sobre a carga Q
3
e para aproxima-la de Q
1
e Q
2
. Entao a energia
potencial do sistema e aumentada da quantia
W
3
= Q
3
V
1
3
+Q
3
V
2
3
Entao o trabalho total necessario para aproximar Q
2
e Q
3
de Q
1
e
W
e
= W
2
+W
3
= Q
2
V
1
2
+ (Q
3
V
1
3
+Q
3
V
2
3
)
**Sistema com n cargas puntiformes
Assim, o trabalho para aproximar n 1 cargas da carga Q
1
W
e
= W
2
+W
3
+. . . +W
n
W
e
= Q
2
V
1
2
+ (Q
3
V
1
3
+Q
3
V
2
3
) + (Q
4
V
1
4
+Q
4
V
2
4
+Q
4
V
3
4
+) +. . .
ou
W
e
=
n

i=2
i1

j=1
Q
i
V
j
i
(2.1)
onde
Q
i
V
j
i
= Q
i
Q
j
4
0
R
ji
= Q
j
Q
i
4
0
R
ij
= Q
j
V
i
j
Tambem pode-se escrever a energia como:
W
e
= W
1
+W
2
+. . . +W
n1
W
e
= Q
1
V
2
1
+ (Q
1
V
3
1
+Q
2
V
3
2
) + (Q
1
V
4
1
+Q
2
V
4
2
+Q
3
V
4
3
+) +. . .
ou
W
e
=
n

i=2
i1

j=1
Q
j
V
i
j
(2.2)
Somando-se 2.1 e 2.2, tem-se
2W
e
= Q
1
(V
2
1
+V
3
1
+V
4
1
+. . .) +Q
2
(V
1
2
+V
3
2
+V
4
2
+. . .) +. . .
Que resulta na energia potencial armazenada num sistema de n cargas puntiformes:
W
e
=
1
2
n

i=1
Q
i
V
i
onde V
i
e o potencial criado por todas as outras cargas j = i.
Observamos que quando se transfere uma carga positiva de um ponto de menor potencial para outro
ponto do espaco com maior potencial, um agente externo precisa realizar um trabalho.
2. Lei de Coulomb 12
PR 2.5: Energia de duas cargas puntiformes. Sejam duas cargas puntiformes Q
1
= 5C e Q
2
=
3C, localizadas em P
1
= (1, 1, 1)m e P
2
= (2, 3, 4)m, respectivamente. Qual e a energia potencial
armazenada no campo eletrostatico?
Solucao: Vamos calcular a energia potencial pelo trabalho realizado por um agente externo para
deslocar uma das cargas (vamos escolher Q
2
) desde o innito ate o ponto nal (P
2
).
W =
_
Fdr =
_
d
12

K
Q
1
Q
2
r
2
dr
W = K Q
1
Q
2

1
r

d
12

W = 0, 02647 J
Tambem podemos calcular a energia potencial usando o potencial eletrico, como feito na rotina EP2Q.for,
descrita abaixo.
c Duas cargas puntiformes
Q1=5.0E-6
Q2=3.0E-6
x1=1.
y1=1.
z1=1.
x2=2.
y2=-3.
z2=4.
c Distancia entre 1 e 2
d12=sqrt((x2-x1)**2+(y2-y1)**2+(z2-z1)**2)
c Potencial da carga 1 em 2
V12= 9.0E9*Q1/d12
c Potencial da carga 2 em 1
V21= 9.0E9*Q2/d12
c Energia potencial de duas cargas
EP2q=0.5*V12*Q2+0.5*V21*Q1
write(*,*) EP2q
stop
end
PR 2.6: Carga total de um o. Um o retilneo, com 3 m de comprimento, esta situado sobre a reta
x = 2 e y = 3, desde z
1
= 0 ate z
2
= 3m. A densidade de carga linear
z
= 4zC/m. Qual a carga do
o?
2. Lei de Coulomb 13
Solucao: Q =
_
z
2
z
1

z
dz
Q =
_
3
0
4, 0E 6 z dz = 18 C.
PR 2.7: Carga de uma gura bidimensional. Calcular a carga compreendida na superfcie delimi-
tada pelas curvas y = x/2 e y =

x, desde x = 2 a x = 4, quando a carga supercial


s
= xy C/m
2
.
Solucao: Vamos encontrar inicialmente a densidade de carga linear para cada valor de x, que de-
nominaremos q(x). Assim:
q(x) =
_

x
x/2
xy dy =
x
2
2

x
3
8
C/m
Agora, podemos calcular a carga total, fazendo a integral em x.
Q =
_
x
2
x
1
q(x) dx =
_
4
2
(
x
2
2

x
3
8
) dx =
11
6
C.
PR 2.8: Carga total de um disco. Um disco de raio R, centrado na origem, esta situado sobre no
plano x y, e possui densidade de carga supercial
s
= r
2
C/m
2
. Qual a carga total do disco?
Solucao: Q =
_ _

s
ds
Q = 4
_
y
2
=0
y
1
=R
_
x
2
=

R
2
y
2
x
1
=0
(x
2
+y
2
) dx dy
Q = 4
_
y
2
=0
y
1
=R
_
(R
2
y
2
)
3/2
3
+y
2
(R
2
y
2
)
1/2
_
dy
Q =
R
4
2
C.
Lembrete: Sempre que voce encontrar uma expressao envolvendo x
2
+ y
2
no integrando, precisa
considerar a possibilidade de converter para coordenadas polares. Vejamos como caria a solucao deste
exemplo:
d
S
= r d dr
Q =
_
r
2
=R
r
1
=0
_

2
=2

1
=0
r
2
r d dr =
R
4
2
C.
PR 2.9: Densidade de carga volumetrica. Demonstrar a equacao do volume de uma esfera de raio
R, e a seguir, fazer o calculo da carga total, quando a densidade volumetrica e = 3r cos C/m
3
.
Solucao: Em coordenadas esfericas, o volume innitesimal e
dv = dr r d r sen d
e a integral em dv vale
V =
_

2
=

1
=0
_

2
=2

1
=0
_
r
2
=R
r
1
=0
r
2
sen d dr d
2. Lei de Coulomb 14
V =
4
3
R
3
Observacao: se tivermos d uvidas sobre os limites de integracao, podemos calcular o volume, a area ou
o comprimento da gura cuja resposta ja seja conhecida. Agora que acertamos os limites de integracao,
podemos fazer o calculo da carga com seguranca.
PR 2.10: Produtos escalar e vetorial. Estas duas operacoes com vetores sao muito usadas no
eletromagnetismo, pois estao presentes em todas as equacoes de Maxwell. Sejam dois vetores

A =
A
1
a
x
+A
2
a
y
+A
3
a
z
e

B = B
1
a
x
+B
2
a
y
+B
3
a
z
, defasados de um angulo , explicar os produtos escalar
e vetorial.
Produto escalar - esta associado ao movimento de translacao, isto e, quanto um vetor contribui com
o outro para modicar o seu modulo. O produto escalar e utilizado para calcular o uxo de um vetor,
ou o trabalho realizado por uma forca ao longo de um percurso. O resultado e um escalar, que vale zero
quando os vetores sao ortogonais.

A

B = A
1
B
1
+A
2
B
2
+A
3
B
3
= |

A| |

B| cos (2.3)
Produto vetorial - esta associado ao movimento de rotacao, isto e, quanto um vetor contribui com
o outro para modicar o seu angulo. O produto vetorial e usado para calcular um momento angular.
O resultado e um vetor ortogonal ao plano formado pelos dois vetores que estao sendo multiplicados.
Quando os dois vetores sao paralelos, o resultado e o vetor nulo.

A

B =

a
x
a
y
a
z
A
1
A
2
A
3
B
1
B
2
B
3

= |

A| |

B|sen n (2.4)
onde n e um vetor ortogonal ao plano formado por

A e

B, e sentido dado pela regra da mao direita (ou
do parafuso), de

A e

B.
PR 2.11: Equacao do trabalho realizado por uma forca. Qual e o trabalho realizado por uma
forca do agente externo de 20 Newton na direcao 45
o
Nordeste, que movimenta um corpo por 0,3 metros
na direcao Oeste para Leste?
Solucao: O trabalho W e denido como uma forca

F agindo atraves de um deslocamento innitesi-
mal d

, onde a forca e aplicada na direcao do deslocamento. Assim, temos a integral do produto escalar
entre os dois vetores:
W =
_
2
1

F d

(2.5)
Para o nosso exemplo
W = 20 . 0, 3 . cos 45
o
= 4, 24 J
2. Lei de Coulomb 15
W =

F

= 20(cos 45 a
x
+ sen 45 a
y
)

(0, 3 a
x
)
W = 20 0, 3(cos 45 a
x
a
x
+ sen 45 a
y
a
x
= 4, 24) J.
Embora a forma vetorial nos pareca mais difcil, ela simplica muito o estudo, a representacao e o
calculo de campos eletromagneticos.
3. CAMPO E POTENCIAL EL

ETRICO
PR 3.1: Dipolo eletrico - Considere duas cargas puntiforme +Q, situada em (+d/2, 0, 0), e Q, situada
em (d/2, 0, 0). Obter a equacao do campo eletrico num ponto P(x
p
, y
p
, 0).
Solucao: O campo resultante das duas cargas e

E =

E
+
+

E

E =
Q
4
0
r
2
+
u
r
+

Q
4
0
r
2

u
r

Denominando u
r
o vetor unitario radial, onde u
r
u
r
+
= cos
+
e u
r
u
r

= cos

, inserindo o angulo
formado entre r com a linha do dipolo, e desenvolvendo a componente radial tem-se a componente
radial do campo tem-se
E
r
=
Q
4
0
r
2
+
cos
+

Q
4
0
r
2

cos

E
r

=
Q
4
0
r
3
2 d cos
O produto Q d e conhecido como momento de dipolo p. Duas cargas de mesma intensidade e opostas
mantidas a certa distancia d uma da outra, possuem momento de dipolo eletrico (nao e momento
mecanico) p, que e um vetor de modulo igual ao produto da intensidade das cargas pela distancia entre
elas.
p = Q

d
onde

d aponta da carga negativa para a positiva. As componentes do campo eletrico em um ponto
generico P situado a uma distancia r do centro do dipolo sao dados por:
E
N
=
2p cos
4
0
r
3
E
T
=
psen
4
0
r
3
E assim, o modulo do campo eletrico do dipolo vale

E =
p
4
0
r
3
(2 cos u
r
+ 2sen u

)
Observa-se que o campo e inversamente proporcional ao cubo da distancia. Se tivessemos 3 cargas,
o campo seria inversamente proporcional a r
4
, e assim por diante.
Fazendo r = x, temos o campo na direcao do eixo
E =
2p
4
0
x
3
(3.1)
3. Campo e potencial eletrico 17
PR 3.2: Tres cargas puntuais iguais a 20pC localizam-se, no vacuo, sobre o eixo x em x = 1, x = 0
e x = 1. (a) Determine a forca resultante que age sobre uma carga de 1C situada em P(1, 10, 2). (b)
Substitua as tres cargas por uma unica carga igual a 60pC localizada na origem, e determine a forca na
carga de 1C. (c) Por que as respostas dos itens (a) e (b) sao semelhantes?
PR 3.3: Fio innito carregado - Imaginemos um o innito carregado com uma carga denida por sua
densidade linear de carga q
L
, conforme a Fig. 3.1.
q
L
R
d

E
r
d

E
Q
Q
Q
Q
Q
Q
Q
Q
Q
Q
Q
Q
r
O
P
L
dL
dQ
Q
Q
Q
Q
Qs
-
Fig. 3.1: Fio innito carregado.
O o e constitudo de uma innidade de cargas puntuais dQ. Cada carga dQ produzira um vetor
campo eletrico d

E no ponto P, que tem modulo


dE =
1
4
dQ
r
2
Tendo em vista que a distribuicao de cargas e uniforme, escreve-se
dQ = q
L
dL
Por outro lado, observa-se que a componente vertical de d

E sera anulada pela componente correspon-


dente quando considerarmos um outro dQ situado simetricamente em relacao ao ponto O. A componente
horizontal, que nos interessa, e
dE
r
=
1
4
q
L
dL
r
2
cos
As variaveis L, r e sao interdependentes; e pode-se reescrever dE
r
como:
dE
r
=
1
4
q
L
dL
r
2
cos
Agora, pode-se fazer o somatorio dos campos dE
r
causados pelas cargas dQ, usando os angulos +/2 e
/2 como limites de integracao. Observa-se que +/2 e /2 sao os angulos que denem a dimensao
innita do condutor. Caso o o fosse nito, seriam outros os angulos limites que determinariam o incio
e o m do condutor, bem como a posicao do ponto P em relacao ao segmento de condutor em questao.
E
r
=
_
+/2
/2
1
4
q
L
dL
r
2
cos =
q
L
2R
V/m
3. Campo e potencial eletrico 18
B
A
P
R
Na continuidade do curso, calcularemos este campo de um modo bem mais simples, usando a lei de
Gauss.
PR 3.4: Deduzir a equacao do campo eletrico produzido por um segmento de o retilneio, uniforme-
mente carregado com carga total Q, de comprimento A+B, em um ponto P, situado longitudinalmente
a uma distancia R do o.
PR 3.5: Campo de um cabo coaxial. Considere um cabo coaxial, onde o condutor interno, de raio a,
esta carregado com densidade de carga +
s
, o condutor externo, de raio b, esta carregado com densidade
de carga linear
L
coulomb/metro. Determinar o campo no isolante situado entre os dois condutores,
a uma distancia r do centro (a < r < b).
Resposta: Este problema classico, muito importante, tem solucao
E = E
r
=

L
2
0

r
r
A constante
r
e a constante dieletrica do isolante, e e apenas um fator que altera a intensidade do
campo, como veremos mais adiante. Observamos tambem que o campo e maximo quando o raio for
mnimo, isto e r = a, e vice-versa quando r = b.
PR 3.6: Campo de um anel carregado de raio R. Determinar o campo num ponto situado no eixo do
anel a uma distancia z do seu centro.
Resposta: A componente radial vale
E =
Q z
4
0
(z
2
+R
2
)
3/2
Exerccio: dividir o anel em N cargas puntiformes, e fazer uma rotina para calcular as componentes e
a resultante do campo num ponto qualquer do espaco.
3. Campo e potencial eletrico 19
PR 3.7: Disco carregado. Determinar o campo eletrico num ponto situado ao longo do eixo de um disco
de raio R carregado uniformemente com carga
s
.
Resposta: Situando o disco no plano xy, com centro coincidente com a origem do sistema de
coordenadas, substituindo os elementos innitesimais e integrando, obtemos
E = E
z
=

s
2
0
(1
z

z
2
+R
2
)

PR 3.8: Chapa plana innita. Determinar o campo num ponto situado a uma distancia d de uma
chapa innita, carregada uniformemente com densidade de carga supercial
s
.
Resposta: Este e um problema muito importante, devido sua grande aplicac ao.

E importante obter
a resposta
E = E
n
=

s
2
0
onde observamos que a intensidade e direcao do campo e independente da distancia d do ponto ate a
chapa. Inicialmente, isto poderia parecer impossvel, mas e bom destacar que estamos tratando de uma
chapa plana innita.
PR 3.9: Meia-casca esferica. Determinar o campo num ponto situado no centro de uma meia-casca
esferica de raio R, carregada uniformemente com densidade de carga supercial
s
.
Resposta: Este e outro problema classico de Eletromagnetismo, que tem como solucao
E = E
r
=

s
4
0
Sugestao: utilizar coordenadas esfericas, usando dS = R
2
sendd.
PR 3.10: Dado o campo eletrico

E = z a
x
3y
2
a
y
+x a
z
V/m, calcular o trabalho realizado por um agente
externo, para mover uma carga de 7C ao longo de um caminho incremental de 1mm de comprimento,
na direcao do vetor 2 a
x
6 a
y
3 a
z
m, localizado em: a) P
A
(1, 2, 3) (R: 511 nJ) e em b) P
B
(2, 0, 4)
(R:-98 nJ)
PR 3.11: Para o campo eletrico

E = z a
x
3y
2
a
y
+ x a
z
V/m, determinar o trabalho realizado por um
agente externo, para deslocar uma carga de 7C ao longo de um caminho retilneo desde A(1, 0, 2) ate
B(3, 2, 2), fazendo o calculo analtico e numerico, dividindo o trecho de reta em 05(cinco) segmentos.
(R: 28 J)
3. Campo e potencial eletrico 20
PR 3.12: A Fig. abaixo ilustra o movimento de uma carga q
0
, na presenca do campo eletrostatico
produzido por outra carga q. O trabalho realizado sobre a carga q
0
, num deslocamento innitesimal d

e
dW =

F d

= q
0

E d


O sinal e negativo quando a forca for interna ou de reacao ao agente externo.
PR 3.13: Consideremos inicialmente o trecho 1 2. A variacao da energia cinetica da carga q
0
neste
trecho e
T
2
T
1
=
_
2
1
q
0

E d

= k q
0
q
_
2
1
r d

r
2
T
2
T
1
= k q
0
q
_
2
1
dr
r
2
= k q
0
q
_
1
r
2

1
r
1
_
Suponhamos agora que a carga q
0
percorra todo o trajeto mostrado na gura, retornando ao ponto 1
de partida. Caso sua energia cinetica fosse, por exemplo, maior que a inicial, teramos uma forma de
produzir energia do nada! Sabemos que isto nao e possvel, pois nao existe um moto perpetuo. Portanto,
devemos ser capazes de demonstrar que o trabalho realizado ao longo de qualquer trajetoria
fechada e nulo. Caso uma determinada trajetoria resultasse em um trabalho negativo (diminuindo a
energia cinetica da carga q
0
), poderamos inverter o sentido da trajetoria obtendo assim um ganho de
energia cinetica. Vamos primeiro mostrar que o trabalho e de fato nulo para a trajetoria simples vista
na gura. Note que, nos trechos 2 3, 4 5, 6 7 e 8 1, a carga q
0
desloca-se perpendicularmente
`a direcao do campo radial

E. Portanto, o trabalho e nulo nestes trechos (dW =

E d

= 0). Nos trechos


onde o trabalho nao e nulo temos
W
12
= k q
0
q
_
2
1
dr
r
2
= k q
0
q
_
1
r
2

1
r
1
_
W
34
= k q
0
q
_
4
3
dr
r
2
= k q
0
q
_
1
r
4

1
r
3
_
W
56
= k q
0
q
_
6
5
dr
r
2
= k q
0
q
_
1
r
6

1
r
5
_
W
78
= k q
0
q
_
8
7
dr
r
2
= k q
0
q
_
1
r
8

1
r
7
_
O trabalho total e a soma dos trabalhos em cada trecho;
W = k q
0
q
_
1
r
2

1
r
1
+
1
r
4

1
r
3
+
1
r
6

1
r
5
+
1
r
8

1
r
7
_
notando que r
2
= r
3
, r
4
= r
5
, r
6
= r
7
e r
1
= r
8
, conclumos facilmente que W = 0.
A curva utilizada no exemplo anterior pode parecer muito especial. Deixamos como trabalho, veri-
car o que acontece em uma situacao mais geral. Se o trabalho total para deslocar uma carga ao longo
de uma superfcie fechada for nulo, chamamos este campo de campo conservativo.
Conclumos este item dizendo que o trabalho realizado por uma forca conservativa so depende da
posicao dos pontos inicial e nal.
3. Campo e potencial eletrico 21
PR 3.14: Tres cargas puntuais Q
A
= 5nC, Q
B
= 2nC e Q
C
= 1nC se localizam no vacuo, nos pontos
P
A
(1, 2, 2), P
B
(2, 4, 1) e P
C
(3, 0, 2), respectivamente. Determine o potencial em P(3, 2, 4) se: (a)
V = 0 no innito (R: 4,15 V); (b) V = 5 V no innito (R: 9,15 V); e, (c) V = 0 em P(0, 5, 2) (R:-6,52
V).
PR 3.15: Uma distribuicao linear e uniforme de carga, de 0, 6 nC/m, esta situada ao longo do eixo z
no vacuo. Determine o potencial em P(3, 4, 2) se: (a) V = 0 em A(2, 9, 3) (R: 6,6 V); (b) V = 24 V
em B(10, 24, 1) (R:41,8 V).
PR 3.16: Um longo o cilndrico, reto, de diametro muito pequeno, e colocado no espaco livre, e possui
uma carga Q por unidade de comprimento. Usando a lei do inverso do quadrado das distancias, entre
um ponto distante x do centro do o, que e muito maior que o diametro do o, denir qual e a forma
das superfcies equipotenciais, e qual e a diferenca de potencial entre dois pontos distantes x
1
e x
2
do
o condutor? Se Q = 0.10E9 coulomb por metro, x
1
= 0.40 cm e x
2
= 15.70 cm, qual e a diferenca
de potencial entre os dois pontos?
PR 3.17: Uma distribuicao supercial plana e uniforme de carga
S
= 40
0
C/m
2
, localiza-se no vacuo,
no plano x = 0. Qual e o potencial em B(7, 3, 1) se V = 0 volt em A(2, 4, 6). (R: -100 V)
PR 3.18: Fazer uma tabela resumo com o campo eletrico e o potencial para as principais distribuicoes
de carga, estudadas ate o momento, e compara-las.
PR 3.19: Dado o campo eletrico

E =
10x a
x
x
2
+y
2
+
10y a
y
x
2
+y
2
2 a
z
V/m
e sabendo-se que o potencial no ponto (3, 4, 5) metros vale 10 volt, determine o potencial no ponto
(6, 8, 7). (R:7,07 V)
PR 3.20: Qual e o trabalho necessario para movimentar uma carga q = 100C ao longo da circunferencia
de raio r = 0, 1m centrada na origem, no campo eletrico

E = (40/r)u

V/mm? Existe diferenca de


potencial entre o incio e o m do percurso? (R: 25,13 mJ)
PR 3.21: Um campo escalar e representado por T = 2xy 5z. (a) Determine o campo vetorial

S = (T/x) a
x
+ (T/y) a
y
+ (T/z) a
z
para o ponto P(1, 2, 3), e determine: (b) T; (c)

S; e (d) u
S
.
PR 3.22: Admitindo que o campo eletrico seja uniforme na direcao ox, calcular o campo eletrico

E
entre duas placas metalicas planas, espacadas de uma distancia l.
3. Campo e potencial eletrico 22
-x
6
y
V =1000V V =600V
-
-
l=0,2 mm
Solucao: O modulo do campo eletrico e 400V/0, 0002m = 2 10
6
V/m. Como o campo eletrico
somente possui a componente horizontal, resulta

E = E
x
a
x
= 2 10
6
a
x
V/m
Conclumos que, estabelecendo V em todo o domnio, estamos estabelecendo tambem o campo
eletrico

E.
O componente de

E em qualquer direcao e o negativo da taxa de variacao do potencial eletrico com
a distancia naquela direcao. Se tomarmos o sistema de coordenadas cartesianas, o campo eletrico em
qualquer ponto e
E
x
=
V
x
E
y
=
V
y
E
z
=
V
z

PR 3.23: Existe relacao entre gradiente e integral de linha? Gradiente do potencial = campo e
_
campo
= potencial ?
PR 3.24: Qual e o campo eletrico no interior de um condutor ideal? Por que?
PR 3.25: Considerando que o potencial eletrico de um ponto qualquer seja dado pela equacao V (x, y) =
3xy
2
, obter a equacao para o campo eletrico.
PR 3.26: Suponha que o potencial eletrico aumente 100 kV/m na direcao Norte-Sul e diminua 50 kV/m
na direcao Leste-Oeste. Qual e o modulo e a direcao do campo eletrico?
PR 3.27: Se o potencial eletrico e dado por V (r, , z) = 24rsen ( + /9) V, qual e o valor do modulo
e a direcao do campo eletrico em x = 2, y = 1 e z = 0?
PR 3.28: Sabendo-se que nos pontos P
1
(2, 1), P
2
(7, 4) e P
3
(3, 8) os potenciais sao respectivamente V
1
=
5V, V
2
= 10V e V
3
= 19V, obter os coecientes a, b e c para o potencial V = a + bx + cy. Qual e o
campo eletrico correspondente a este potencial?
PR 3.29: Demonstrar que o potencial num ponto P devido `a duas cargas Q e +Q e a soma dos
potenciais devidos `as duas cargas, isoladamente, e
V =
Q
4
p r
r
2
e que o campo eletrico tambem pode ser obtido pelo gradiente de V :

E =

V
4. LEI DE GAUSS E CAPACIT

ANCIA
PR 4.1: Um capacitor de placas planas paralelas tem uma area de 1.51 m
2
distantes de 1.0 mm, com
um isolante de constante 3,2. Este capacitor esta ligado a uma d.d.p. de 13800 Volts. Determinar os
valores de:
Intensidade do campo eletrico;
Intensidade do vetor inducao eletrica;
Intensidade do vetor polarizacao eletrica.
Densidade de carga;
Carga livre acumulada; e,
Capacitancia.
PR 4.2: Duas placas planas paralelas tem uma area de 1,51 m
2
e separacao de 10 mm (com ar
r
= 1),
e estao submetidas a uma d.d.p. de 50 V. Desprezando o efeito de borda, calcular: (a) O campo
eletrostatico E; (b) A inducao eletrostatica D; (c) A polarizacao P; e, (d) A carga livre Q e polarizada
q. A seguir, inseriu-se um dieletrico com constante dieletrica
r
= 2, 5. Quais serao os novos valores de
E, D e P?
Solucao: O campo eletrostatico vale
E = 50/10/0, 001 = 5000 V/m
A inducao eletrostatica D e
D =
0

r
E = 8, 85 10
12
1 5000 = 44, 25 nC/m
2
A polarizacao P vale
P = D
0
E = 0
A carga livre nas placas e
Q = D S = 44, 25 1, 51 = 66, 81 nC
e nao existe carga polarizada. Este resultado era esperado, pois o dieletrico inicial e o ar.
4. Lei de Gauss e Capacitancia 24
PR 4.3: Para o exemplo anterior, com um dieletrico de constante dieletrica
r
= 2, 5, temos ainda
E = 5000 V/m, pois a tensao aplicada se manteve constante. A nova inducao eletrostatica D sera
D =
0

r
E = 8, 85 10
12
2, 5 5000 = 110, 625 nC/m
2
e a polarizacao P
P = D
0
E = 110, 625 44, 25 = 63, 375 nC/m
2
A carga livre nas placas e
Q = D S = 110, 625 1, 51 = 167, 04 nC
e a carga polarizada.
q = P S = 63, 375 1, 51 = 95, 69 nC
Observamos que houve um aumento da carga livre nas placas, mas a diferenca Qq permaneceu con-
stante. O estudo dos dieletricos adquire grande relevancia na construcao de dispositivos armazenadores
de energia eletrica, tambem conhecidos como condensadores ou capacitores, os quais constam basica-
mente de duas placas condutoras com potencial eletrico distinto, entre as quais se intercala a substancia
dieletrica. A capacidade de armazenamento de um condensador se avalia mediante um coeciente -
conhecido como capacitancia - que depende de suas caractersticas fsicas e geometricas. Essa grandeza
tem dimensoes de carga por potencial eletrico e se mede comumente em farads (coulombs por volts).
PR 4.4: A constante dieletrica da Ebonite e 2.8, e sua rigidez dieletrica vale 18106 V/mm. Qual a
menor area que podem ter as placas de um capacitor plano de 7.0E-2 microfarad, usando a ebonite
como dieletrico, para que este suporte uma diferenca de potencial de 4000 V.
PR 4.5: O campo eletrico, num ponto qualquer do espaco, e

E = 2 a
x
+ 3 a
y
5 a
z
V/m. Sabendo que
a permissividade relativa do meio e 4, 5, qual e o vetor e o modulo da densidade de uxo eletrico, ou
intensidade do vetor deslocamento eletrico, ou inducao eletrica

D, neste ponto?
PR 4.6: Uma esfera metalica tem 0,5 m de raio com 20 C distribudo em sua superfcie. Qual a
densidade de uxo eletrico na superfcie externa da esfera? Qual e a densidade de uxo a 0,5 m distante
da esfera? (R: 6,36 C/m
2
e 1,59 C/m
2
)
PR 4.7: Uma esfera condutora metalica de raio R = 0, 5 m foi carregada com uma carga de 1 C.
Como se distribuem as cargas nesta esfera? Qual a densidade supercial de carga nessa esfera? (R:
1/C/m
2
)
PR 4.8: Em um ponto P(2, 5, 4) em uma superfcie condutora esferica, a densidade supercial de
carga e 75 nC/mm
2
. Se o condutor esta isolado no vacuo, encontre

E fora e dentro do condutor nas
vizinhancas do ponto P.
4. Lei de Gauss e Capacitancia 25
PR 4.9: A densidade de carga eletrica de uma casca de raio interno 10 cm e raio externo 11 cm e dada
por (r, , ) = 2 10
10
/r
2
C/m
3
. Pergunta-se: a) Qual a carga total da casca carregada? b) Qual a
intensidade, direcao e sentido do campo eletrico na superfcie interna; e, c) na superfcie externa?
PR 4.10: Fio innito carregado - Este problema pode ser resolvido de duas maneiras distintas: imagi-
nando uma superfcie simetrica ou fazendo a integral dos uxos.
Se um condutor longo, reto e cilndrico tem uma carga eletrica uniforme ao longo de seu comprimento
e esta isolado de outras cargas de modo que sua carga esteja uniformemente distribuda em sua periferia,
o uxo sera radial. Por simetria, todos os pontos eq uidistantes desse condutor tem a mesma densidade
de uxo eletrico. Assim, a densidade de uxo eletrico a r metros do condutor pode ser calculada
imaginando uma superfcie gaussiana cilndrica concentrica ao condutor, de raio r. A area da superfcie
lateral do cilindro com 1m de altura e 2r m
2
. Considerando a carga q
L
Coulombs por metro de
condutor, o modulo da densidade de uxo eletrico e
D =
q
L
2r
coulombs/metro quadrado
Finalmente, dividindo D pela permissividade do meio , encontramos o campo eletrico no ponto P.
E =
q
L
2
0
r
V/m
que e igual ao valor obtido pela lei de Coulomb.
PR 4.11: Cabo coaxial - Demonstre a equacao do campo eletrico de um cabo coaxial.
E =
q
L
2
0

r
r
O resultado e igual ao campo de um condutor retilneo, obtido pela lei de Coulomb.
PR 4.12: Superfcie plana innita. Se cortarmos um cilindro (superfcie gaussiana) exatamente no seu
centro, com o plano das cargas, o uxo total do cilindro e igual a duas vezes o uxo de cada base.
2 D S =
S
S
isolando D e dividindo pela permissividade
E =

S
2
0

r
O resultado indica que o campo nao varia com a distancia. Esta equacao pode ser usada para calcular
o campo de duas placas paralelas.
4. Lei de Gauss e Capacitancia 26
PR 4.13: Calcular para o ponto P(0.1, 4.3, 0.6), a divergencia de cada um dos seguintes campos:

F
1
= xze
2y
(z a
x
+xz a
y
+x a
z
)

F
2
= (x a
x
+y a
y
+z a
z
)/
_
x
2
+y
2
+z
2

F
3
= 0.2 a
x
0.6 a
y
+ 0.35 a
z

F
4
= xy
2
z
3
( a
x
+ 2.0 a
y
+ 3.0 a
z
)
PR 4.14: Considere-se que um cilindro longo e oco esteja cheio de ar sob pressao; e a tampa de uma
extremidade seja retirada rapidamente. A velocidade v do ar tem divergente; pois se colocarmos um
pequeno volume num ponto, vemos que a quantidade de ar que entra nao e igual `a que sai pelo lado
oposto. Esta e a forma de ver se um campo tem divergente: colocar um pequeno volume, e ver se o
balanco lquido e diferente de zero.
PR 4.15: Comprovar o teorema da divergencia para

D = 2xy a
x
+x
2
a
y
e o paraleleppedo formado pelos
planos x = 0, x = 1, y = 0, y = 2, z = 0 e z = 3.
Com integral de superfcie:
_

D d

S =
_
3
0
_
2
0
(2y a
x
+ a
y
) (dydz a
x
)
+
_
3
0
_
1
0
(x
2
a
y
) (dxdz a
y
) +
_
3
0
_
1
0
(4x a
x
+x
2
a
y
) (dxdz a
y
)
=
_
3
0
_
2
0
2y dydz = 12
Com integral de volume:


D =

x
(2xy) +

y
(x
2
) = 2y
_


Ddv =
_
3
0
_
2
0
_
1
0
2y dx dy dz = 12.
PR 4.16: V = 1000r
c
2
V em coordenadas cilndricas.
1. Se a regiao 0.1 < r
c
< 0.3 m e vacuo e as superfcies 0.1 e 0.3 sao condutoras, especique a
densidade supercial de carga de cada condutor.
2. Qual e a carga ao longo de 1 m de comprimento da regiao onde ha vacuo?
3. Qual e a carga total ao longo de 1 m de comprimento, incluindo ambas as cargas superciais?
PR 4.17: Um campo potencial eletrico e dado por V = x
4
+y
4
1 V.
1. Esboce as superfcies equipotenciais V = 0 e V = 100 V. Estas sao duas superfcies condutoras.
4. Lei de Gauss e Capacitancia 27
2. Determine a densidade volumetrica de carga no ponto P(2, 1, 3), situado no vacuo, entre as su-
perfcies.
3. Encontre a densidade supercial de carga no ponto C(1,0,0) em um dos condutores.
PR 4.18: A superfcie x + 2y
2
+ 4z
3
= 100 e o contorno de um objeto condutor situado no vacuo. A
origem esta situada no interior do condutor e o ponto A(18, 5, 2) em sua superfcie. Se |

E| = 50 V/m
e o campo esta orientado para fora do condutor, determine

E,

D, e
S
neste ponto.
PR 4.19: Um campo potencial e dado por
V = 100 ln
(x + 1)
2
+y
2
(x 1)
2
+y
2
V.
Sabendo que o ponto P(2, 1, 1) esta na superfcie do condutor e que ele esta situado no vacuo, determinar
o vetor unitario normal `a superfcie bem como a densidade supercial de carga no condutor.
PR 4.20: Uma carga puntual Q localiza-se a uma distancia h de um plano condutor. Determinar o
lugar geometrico dos pontos do condutor para os quais a densidade supercial de carga e 0.1Q/h
2
.
PR 4.21: Duas cargas puntuais de 100C estao localizadas em (2, 1, 0) e (2, 1, 0). A superfcie
x = 0 e um plano condutor.
1. Determine a densidade supercial de carga na origem.
2. Determine
S
no ponto P(0, h, 0).
PR 4.22: Seja um sistema de coordenadas esfericas e uma densidade volumetrica de carga variando
linearmente com o raio,
V
=
0
r/a (
0
e a constantes). Determine a carga contida: (a) na esfera
r a; (b) no cone r a, 0 0, 1; (c) na regiao r a, 0 ; 0 0, 2.
PR 4.23: Seja = (10
6
/r) C/m3 em coordenadas esfericas. Determine: (a) a densidade volumetrica
de carga na origem; (b) a carga contida em uma esfera de 1mm de raio centrada na origem; (c) a carga
contida na regiao 10 r 20mm; 28
o
31
o
; 0, 9 0, 96.
PR 4.24: Calcular a funcao para a densidade volumetrica de cargas quando o potencial eletrico seja
V = 8(x
2
+y
2
). (R: = 32
0

r
)
PR 4.25: Um capacitor, formado por duas placas paralelas, distantes 0,1mm, e cujas areas sao 200
cm
2
, esta ligado a uma fonte de tensao contnua de 12 V. Admitindo que o isolante entre as placas
seja plastico, com permissividade relativa igual a 3,0, calcular a carga acumulada em cada placa do
capacitor? (R: 63,7 nC)
4. Lei de Gauss e Capacitancia 28
PR 4.26: Um capacitor tem capacitancia de 0,1F, e esta ligado a uma rede eletrica de 220 V. Qual a
carga contida neste capacitor? O que signica este valor ... e a carga total das duas placas? (R: 22C)
PR 4.27: Um capacitor a ar de placas planas paralelas (considere campo uniforme entre as placas), tem
uma area de 1,51 m
2
e separacao entre as placas de 10 mm, e ligado a uma fonte de tensao contnua de
240 Volts. A seguir todo o espaco livre entre as placas e preenchido com oleo (
r
= 2, 5). Determinar o
que acontece com: a) a carga do capacitor; b) o campo eletrico entre as placas; c) a carga de polarizacao;
d) a inducao ou a densidade de uxo eletrico entre as placas.
PR 4.28: Considerando um capacitor de placas paralelas contendo um dieletrico de permissividade ,
de espessura d, area de placas A, e capacitancia C, demonstrar que

D =

E e a forma local da equacao


Q = CV .
Solucao: Partindo da equacao da capacitancia
C =
Q
V
=
A
d
e, considerando que V = E d e Q = D A, encontra-se
D A
E d
=
A
d
que simplicando os termos d e A resulta:
=
D
E
ou

D =

E
PR 4.29: Um capacitor de placas paralelas com area de 0.30 m
2
e separacao de 5.5 mm, contem tres
dieletricos com interfaces normais

D e

E, como segue:
r1
= 3.0 , d
1
= 1.0 mm;
r2
= 4.0 , d
2
= 2.0
mm;
r3
= 6.0 , d
3
= 2.5 mm. Encontre a sua capacitancia.
PR 4.30: Um capacitor a ar possui capacitancia de 10 pF. Quando ele e submerso em oleo seu valor de
capacitancia passa a 18 pF. Qual e a permissividade do oleo? Por que a capacitancia aumentou? (R:
15, 93 x10
12
F/m)
PR 4.31: Um capacitor a ar (tipo variavel) e carregado e depois desligado da fonte. A seguir ele e
imerso em um recipiente com oleo. A energia armazenada no capacitor aumenta ou diminui? Por que?
PR 4.32: Um capacitor a ar com capacitancia 0.005 microfarad e conectado a uma linha de corrente
contnua de 500 volts, desconectado, e entao imerso num oleo com constante dieletrica 2.5. Calcular a
energia armazenada no capacitor antes e depois da imersao no oleo.
4. Lei de Gauss e Capacitancia 29
PR 4.33: Submetendo um capacitor de 6 micro Farad a uma tensao de 50 V e uma freq uencia de 1000
Hz, tem-se a perda de 0,5 W no mesmo. Qual e o seu fator de dissipacao tan?
Solucao:
tan =
P
C
V
2
C
=
0, 5
50
2
21000 6E 6
= 0, 0053
= 0, 303
o

PR 4.34: Um capacitor de 70 F num desbrilador esta com carga de 5.000 V, e a energia armazenada
no capacitor e
U = 1/2CV
2
= (1/2)(70 10
6
)(5.000)
2
= 875J.
Aproximadamente 200J dessa energia sao enviados atraves da vtima durante um pulso de cerca de 2,0
ms. A potencia do pulso e
P =
W
t
=
200
2 10
3
= 100kW,
muito maior que a potencia da propria bateria.
PR 4.35: Tres cargas puntiformes de valor 1, 2 e 3 C estao situadas nos nos de um triangulo eq uilatero
com 1 m de lado. Calcular o trabalho realizado para deslocar estas cargas para um triangulo eq uilatero
com 1/2 m de lado.
Solucao: A energia potencial armazenada no sistema de tres cargas no triangulo eq uilatero maior e
W
e
=
1
2
3

i=1
Q
i
V
i
=
1
2
[
5 + 8 + 9
4
0
] =
11
4
0
J
A energia potencial armazenada no triangulo menor e o dobro deste valor, pois todos potenciais sao
a metade. O aumento da energia potencial e igual a 11/(4
0
)J.
Obviamente, este aumento de energia deve ser suprido por um agente externo.
PR 4.36: Uma isolacao e composta de tres camadas, dispostas como um sanduche, com: 2 mm de oleo
(
r
= 2, 2, V
ruptura
= 80 kV/cm); 1,5 mm de papel (
r
= 6, 0, V
ruptura
= 200 kV/cm); e, 2 mm de oleo
novamente. Qual a tensao de isolacao desta conguracao? O que acontece com a tensao admissvel se
variarmos a espessura relativa entre os dois isolantes?
PR 4.37: Qual e a equacao da capacitancia de um cabo coaxial, de raio interno a e externo b? (R:
C = 2L/ ln(b/a))
PR 4.38: Qual e a capacitancia entre duas esferas concentricas, com raio interno a e raio externo b?
(R: C = 4[ab/(b a)])
4. Lei de Gauss e Capacitancia 30
PR 4.39: Considere dois os innitos paralelos, com raio r
0
e separados por uma distancia d entre seus
centros, e carregados uniformemente com densidade +
L
e
L
. Determinar: a. A equacao do campo
eletrico; b. A equacao do potencial eletrico; c. A capacitancia unitaria (F/m) entre os dois condutores;
PR 4.40: Considere um o innito com raio r
0
, paralelo e separado por uma distancia d/2 entre seu
centro e o solo (V = 0 como uma superfcie innita), carregados uniformemente com densidade +
L
.
Determinar a capacitancia unitaria entre o o e o solo.
PR 4.41: Dado que

E
1
= 2 a
x
3 a
y
+ 5 a
z
V/m, chegando no plano 4x 3y + z = 2, para a interface
dieletrica sem cargas livres, encontre

D
2
, sabendo que
r1
= 3 e
r2
= 1.
PR 4.42: Demonstre a equacao da capacitancia de um capacitor constitudo por dois cilindros concentricos
de raios a e b (a < b) e de altura h. O dieletrico entre os dois cilindros possui constante dieletrica

r
= 3, 0.
PR 4.43: O isolamento de um dispositivo eletromagnetico, como e o caso de um transformador, se con-
stitui numa das principais, senao a maior, preocupacao para o projetista. Em princpio o transformador
opera em tensoes elevadas, pois e esta a caracterstica que o torna util nos sistemas eletricos. A operacao
em tensoes sempre maiores tem feito do isolamento o principal topico de pesquisas e de desenvolvimento
de novos materiais isolantes. O isolamento de um transformador compreende:
1. Isolamento entre bobinas de uma mesma fase;
2. Isolamento das cabeceiras das bobinas;
3. Isolamento entre a bobina interna e o n ucleo;
4. Isolamento entre a bobina externa e a caixa (tanque);
5. Isolamento entre fases;
6. Isolamento entre espiras entre camadas e entre discos;
7. Isolamento dos terminais e buchas passantes; e,
8. Isolamento das ligacoes, comutadores e paineis de ligacao.
Considere o problema do isolamento entre as bobinas de AT e BT com um cilindro isolante dividindo
o canal de dispersao de uxo eletrico em dois canais de oleo e uma camada de papelao, formando um
sanduche. Sendo
oleo
e
p
as constantes dieletricas, respectivamente, do oleo e do papelao (cilindro) e
V
p
a tensao de prova (ensaio), pode-se calcular os gradientes de potencial E
oleo
e E
p
no oleo e papelao,
4. Lei de Gauss e Capacitancia 31
admitindo-se que o campo, entre AT e BT seja uniforme. Fazendo a associacao de capacitores em serie,
pode-se demonstrar que:
E
oleo
=
V
p

oleo
_
a
1

oleo
+
a
2
p
+
a
3

oleo
_
E
p
=
V
p

p
_
a
1

oleo
+
a
2
p
+
a
3

oleo
_
Estas expressoes determinam os gradientes de potencial no oleo e no papelao. Estes gradientes nao
podem ser superiores aos gradientes de ruptura do oleo e/ou do papelao nas condicoes de sobretensoes de
ensaio, e conseq uentemente de operacao. Utiliza-se um coeciente de seguranca que depende do projeto.
Tambem observa-se pelas expressoes que o gradiente sera maior no material com menor constante
dieletrica.
PR 4.44: Considerando o problema do isolamento entre as bobinas de AT e BT de um transformador,
calcular a tensao de prova suportada para duas camadas de oleo com 2cm de espessura, e uma camada
de papelao com 1,5cm. Sabemos que as permissividades relativa do oleo e do papelao valem 2,2 e 6,0
respectivamente; e a rigidez dieletrica do oleo e 80 kV/cm e do papelao 200 kV/cm.
Solucao: Se aplicarmos uma tensao de prova de 100 kV, teremos os campos eletricos
E
oleo
=
100 000
2, 2
_
0,2
2,2
+
0,15
6
+
0,2
2,2
_
E
oleo
= 219, 78 kV/cm
e, de forma semelhante
E
papel
= 80, 586 kV/cm
Observa-se que, nesta conguracao, o oleo romperia primeiro. Assim, fazemos uma regra de tres para
calcular a tensao de prova, que sera 36,4 kV.
5. CAMPO MAGN

ETICO
PR 5.1: Campo magnetico devido a um condutor longo retilneo. Determine o campo magnetico

H
num ponto P distante R metros de um condutor innitamente longo, percorrido por uma corrente de
I amperes. A seguir, calcule o campo a uma distancia de 10 cm do condutor quando ele for percorrido
por uma corrente de 0,1A.
Solucao: Usando a lei de Biot-Savart, a contribuicao

H para o campo no ponto P, devido ao


elemento de corrente i e:

H =
1
4r
3
i

r
A componente

H tera sempre a mesma direcao e sentido, dados pela regra da mao direita. Considerando
o angulo , a componente de

H torna-se
H =
1
4r
3
I r sen (90 )
Lembrando que arco = angulo x raio, podemos escrever a relacao entre e :
r = cos
ou seja
= r

cos
= R

cos
2

Substituindo em H:
H =
I cos
4R
Agora, integrando esta expressao desde = /2 ate = /2 obtem-se o campo total H.
H =
_
/2
/2
I
4R
cos d
que resulta
H =
I
2R
amperes/metro (5.1)
Entao, se R = 0, 1m e I = 0, 1A, a intensidade do campo e H = 0, 159A/m.
PR 5.2: Dois os retilneos paralelos estao afastados de d = 40 cm, e sao percorridos por correntes
I
1
= 100A e I
2
= 60A, em sentidos opostos. Encontrar a distancia x de um ponto P ao primeiro
condutor, onde o campo magnetico total seja nulo.
5. Campo magnetico 33
6
I
1
?
I
2
r
P
Solucao: Aplicando a eq. (5.1) para os dois condutores, e igualando a zero
I
1
2x
=
I
2
2(x d)
100
2x
=
60
2(x 0, 4)
que resulta x = 1, 0 metro do primeiro condutor.
PR 5.3: Uma espira circular, de raio R, e percorrida pela corrente I. Obter a equacao do campo
magnetico no centro da mesma.
Solucao: O produto vetorial do numerador da lei de Biot-Savart e
d

r = R d R = R
2
d
e
H =
_
I
R
2
d
4 R
3
=
I
4R
_
2
0
d =
I
2R
que e o campo no centro da espira circular.
PR 5.4: Campo magnetico de uma espira circular - Neste exemplo, calcularemos o valor do campo
magnetico em um ponto generico P, situado no eixo de uma espira circular percorrida por uma corrente
constante I, conforme esquema da Fig.
Como d

e r, da lei de Biot-Savart, sao perpendiculares entre si, o modulo do produto vetorial e


simplesmente d r. Temos entao o modulo de d

H:
dH =
I d
4 r
2
Na integracao ao longo da espira, cada valor do componente radial dH
R
e anulado pelo seu oposto de
180
0
. Portanto, esses componentes nao entram no calculo de H para um ponto situado no eixo da
espira. Mas o componente axial
dH
A
= dH cos = dH
R
r
=
R I d
4 r
3
5. Campo magnetico 34
Integrando d = Rd:
H =
_
dH
A
=
R I
4 r
3
_
d
=
R I
4 r
3
2R =
R
2
I
2 r
3
Mas como
r =
_
R
2
+x
2
temos, portanto
H = I
R
2
2 (R
2
+x
2
)
3/2
(5.2)
Este resultado e fundamental para calcular o momento de dipolo magnetico.
PR 5.5: As bobinas de Helmotz sao duas bobinas circulares coaxiais, onde seus raios R sao iguais
`a distancia d entre elas, isto e R = d. Elas sao muito conhecidas pelo fato de que o campo magnetico
uniforme ao longo do seu eixo. Fazer um graco com a amplitude do campo ao longo do eixo das
bobinas.
Solucao: Situando o eixo x, tal que x = 0 no ponto equidistante das duas bobinas, o modulo do
campo magnetico dado por (5.2) torna-se
H = NI
R
2
2 (R
2
+ (x d/2)
2
)
3/2
+NI
R
2
2 (R
2
+ (x +d/2)
2
)
3/2
como d = R escreve-se
H = NI
R
2
2 (R
2
+ (x R/2)
2
)
3/2
+NI
R
2
2 (R
2
+ (x +R/2)
2
)
3/2
que resulta
H =
NI
2R
_
_
_
1 +
(x R/2)
2
R
2
_
1,5
_
1 +
(x +R/2)
2
R
2
_
1,5
_
_
(5.3)
e, para x = 0 o campo vale
H
0
= 0, 7155
NI
R
A eq. (5.3) pode ser expandida em serie, para x,
H(x) = H
0
(1 +c
4
x
4
+c
6
x
6
+...)
e para valores de x proximos de zero, a derivada dH/dx

= 0, e H

= H
0
, comprovando que a intensidade
de

H permanece constante para um grande intervalo.
5. Campo magnetico 35
PR 5.6: Campo de um solenoide nito. Vericar a validade da equacao do campo magnetico
H =
NI

4R
2
+L
2
=
NI

D
2
+L
2
onde R e o raio do solenoide e L e o seu comprimento.
Resposta: Quando L 0, H NI / 2R; e, quando L , H NI / L, que demonstra a
validade desta equacao.
PR 5.7: Calcule a intensidade do campo magnetico no centro de uma espira quadrada com 32 cm de
lado, quando e percorrida por uma corrente de 10 A. (Resposta: 59 A/m)
PR 5.8: Um solenoide com 64 cm de comprimento e 2,54 cm de diametro, tem uma distribuicao uniforme
de 600 espiras de condutor de cobre isolado. Calcular a intensidade do campo magnetico no centro do
solenoide, quando e percorrido por uma corrente de 2 A. (Resposta: 1970 A/m)
PR 5.9: Desenhar diagramas mostrando o campo magnetico ao redor de dois condutores retilneos
paralelos percorridos por corrente: (a) no mesmo sentido; e, (b) em sentidos opostos. Se os dois
condutores estiverem afastados de 7,62 cm e tiverem uma corrente de 1000 A no mesmo sentido, qual
a forca por metro de comprimento? (Resposta: 2,6246 N/m, de atracao.)
PR 5.10: Montou-se duas bobinas com 20 cm de diametro e 100 espiras de um condutor muito no,
e dispostas coaxialmente a uma distancia de 10 cm uma da outra. Ligou-se uma fonte de corrente de
1 A, que passa nas duas bobinas em serie, de forma que os campos das duas bobinas sejam aditivos.
Plotar curvas mostrando a intensidade do campo magnetico ao longo do eixo das bobinas. (Demonstrar
qualquer formula utilizada).
PR 5.11: Tem-se dois os innitos separados por uma distancia A, nos quais passam as correntes I e
2I. Calcular o campo devido aos dois os H, no ponto P, situado a uma distancia x do o 1. (R:
H =
I
2
A+x
x(Ax)
A/m)
PR 5.12: Uma barra condutora condutora innita, de secao retangular, com expessura e e largura 2L,
possui uma corrente I uindo transversalmente.
a) Calcule o modulo da densidade de corrente J (A/m
2
) na barra; (R: J =
I
2 e L
A/m
2
)
b) Considere que o afastamento de um ponto ate a barra x << L, e calcule o campo magnetico H
criado pela barra no ponto P, cuja projecao sobre a mesma se situa na sua linha mediana.
PR 5.13: Tendo 3 os innitos percorridos por I, formando um triangulo equilatero com afastamento l
entre os centros, calcular o campo total no baricentro do triangulo, devido aos tres os (R: H
t
= 0)
PR 5.14: Calcule o campo magnetico num ponto P, situado no mesmo plano de uma espira circular de
raio R, que e percorrido corrente I, a uma distancia x do centro da mesma. (R: H =
IR
2
2

(R
2
+x
2
)
3
A/m)
5. Campo magnetico 36
PR 5.15: Calcule o campo magnetico em um ponto P, que se situa no encontro das diagonais de uma
espira retangular de largura 2b e altura 2a, criado por uma corrente I. (R: H =

a
2
+b
2
I
ab
A/m)
PR 5.16: Calcular a intensidade do campo magnetico no centro do sistema de coordenadas, se uma
bobina retangular com uma unica espira esta colocada no plano xy, passando 47.8 ampere nos lados
colocados em x = 17.3cm, x = 2.8cm, y = 12.5cm, e y = 3.2cm.
PR 5.17: Helmholtz, ligou duas bobinas circulares com mesmo raio, em serie, dispostas com os seus
eixos em comum, e observou que a intensidade do campo magnetico no eixo, e na metade entre as duas
bobinas possui gradiente nulo num trecho ao longo do eixo. Considerando N = 167 espiras, i = 13.4A,
calcular o campo quando o raio a = 0.6m. Observacoes: a. As duas bobinas estao associadas para
superposicao do campo. b. A distancia entre as duas bobinas e igual ao raio.
PR 5.18: Campo magnetico de um solenoide. Forma-se um campo magnetico ao redor de uma bobina de
o de cobre, chamada solenoide, cujo comprimento e muito maior do que o seu raio, e consideraremos
o solenoide innito. Usando argumentos de simetria e facil mostrar que sao nulos os campos entre os
os e na parte externa do solenoide. No interior do solenoide o campo tem o sentido indicado pela regra
da mao direita.
Vamos usar a lei de Amp`ere para calcular o modulo de H no interior do solenoide. Vamos escolher
um retangulo como uma linha fechada amperiana, que e cortado pela linha que une os condutores de
um lado do solenoide. A corrente que atravessa o retangulo a amperiana selecionada) e igual `a corrente
i, multiplicada pelo n umero de espiras que atravessa a amperiana. Como o solenoide tem um n umero
innito de espiras (na pratica, um n umero muito grande de espiras), a corrente que entra na lei de
Amp`ere e calculada em termos da densidade de espiras n espiras por metro de solenoide. Supondo que
temos n espiras por metro, num determinado comprimento L, a corrente que atravessa a amperiana
sera n L i. Assim,
_
L
Hd = n L i
A integral fechada pode ser desdobrada, tomando H = 0 na regiao externa ao solenoide, de modo que
H = n i

E claro que para um solenoide nito, o valor do campo e menor. Mas, este valor e uma referencia, ou
um valor limite que nunca sera ultrapassado!
PR 5.19: Campo de um toroide. No interior do toroide, aplicando a lei de ampere, e integrando na
linha amperiana circular de raio r, e isolando H, temos:
H =
1
2 r
N I (5.4)
onde N e o n umero de espiras do toroide.
5. Campo magnetico 37
PR 5.20: Campo magnetico dentro de um o. Consideremos o o condutor como um cilindro innito,
de raio R, transportando uma corrente I
0
, com densidade uniforme.
Solucao: Vamos escolher uma linha amperiana circular, com raio r R. Como a corrente tem
densidade uniforme, temos a corrente I envolvida pela linha amperiana
I = I
0
r
2
R
2
Este resultado sera usado para calcular a indutancia interna de um o.
PR 5.21: Comparar, citando vantagens e desvantagens, de usar ou nao usar; ferramentas matemati-
cas mais avancadas (gradiente, divergente, rotacional, integral de linha, . . . ) para os problemas de
engenharia.
PR 5.22: Calcular a intensidade do vetor campo magnetico a 0,5 m do centro de um condutor de cobre,
com 25 mm
2
, imerso em oleo, que e percorrido por uma corrente de 100 A. A condutividade do cobre e
58 mm
2
/m/. (R: 31,8 A/m)
PR 5.23: Considerando uma corrente i uindo na direcao positiva ao longo do eixo z de a +,
calcular o campo magnetico num ponto P afastado de uma distancia r do condutor, usando: (a) a lei
de Biot-Savart; e, (b) a lei circuital de Ampere. (R:

H =
i
2 r
u

)
PR 5.24: Uma superfcie innita esta situada no plano z = 0, e e percorrida por uma corrente supercial
uniforme

J = k a
y
A/m. Escolher um percurso fechado e calcular o campo magnetico a uma distancia
z da superfcie. (R: H
x
= k/2. O campo para uma superfcie de corrente nao varia com a distancia.)
PR 5.25: Uma densidade de corrente

J = J
z
a
z
, centrada no eixo z, passa atraves de um condutor de
raio R. Calcular o campo magnetico: (a) no interior do condutor; e, (b) externo ao condutor. (R:
H

= J
z
r/2 e H

= J
z
R
2
/(2r))
PR 5.26: Calcule o campo, dentro, no meio e fora de um cabo cilndrico oco de raio interno a e raio
externo b, percorrido por uma corrente I, de maneira uniforme. (R: a) dentro H = 0, b) no meio
H =
I
2r
r
2
a
2
b
2
a
2
e c) fora H = I/(2r))
6. FLUXO, INDUC

AO E FORC A MAGN

ETICA
PR 6.1: A magnetizacao de saturacao do ferro e 1, 710
6
A/m, e sua densidade e 7970 kg/m
3
. Sabendo
que o n umero de Avogadro vale 6, 025 10
26
kg-atomo, e a massa atomica relativa do ferro e 56,
calcular o momento magnetico de cada atomo de ferro, em Am
2
. Solucao: Um metro c ubico tem massa
de 7970kg, e contem o seguinte n umero de atomos:
N =
7970 6, 025 10
26
56
= 8, 58 10
28
atomos
E o modulo do momento magnetico m por atomo vale
m =
1, 7 10
6
8, 58 10
28
= 1, 98 10
23
Am
2

PR 6.2: Considere que o campo magnetico passa do ar para um meio contendo


r
= 850, incidindo a
45
o
com a normal. Qual o angulo no meio 2?
Solucao: Isolando
2
encontra-se
tg
2
= tg 45
o
850
1
= 850

2
= 89, 93
o
Observa-se uma grande variacao angular na passagem do ar para um meio ferromagnetico.
PR 6.3: Considere um o retilneo, situado no ar, proximo a um material com permeabilidade
r
. Fazer
um esboco das linhas de uxo para: (a)
r
= 1; (b) 1 <
r
= 1 << ; e, (c)
r
.
Solucao: Quando
r
= 1, as linhas sao circulares, em volta do o. Quando 1 <
r
= 1 << , as
linhas tem o formato mostrado na gura ... Quando
r
, a espessura do uxo e a intensidade do
campo magnetico no ferro cai para zero.
PR 6.4: A tensao de Hall do semicondutor com R
H
= 0, 63 10
3
m
3
/As, espessura = 10
3
m, e
corrente I = 100 mA e:
V
H
= 63 mV/T
PR 6.5: Dado o campo vetorial

F = x
3
a
x
+y
3
a
y
+z
3
a
z
, calcular:
6. Fluxo, inducao e forca magnetica 39
1. O uxo de

F atraves de uma esfera de raio R centrada na origem.
2. A circulacao de

F atraves de uma circunferencia de raio R centrada na origem e situada no plano
xy.
Solucao: Fazendo o divergente de

F:


F = 3 (x
2
+y
2
+z
2
) = 3 r
2
De acordo com o teorema da divergencia podemos escrever:
_ _
S

F d

S =
_ _ _
V
(


F) dV
=
_
R
0
(3r
2
) (4r
2
dr) =
12R
5
5
Como


F = 0, a circulacao de

F e nula para qualquer contorno fechado C.
PR 6.6: Representar o campo vetorial

A = 2y a
x
z a
y
+ 3x a
z
em coordenadas esfericas, especicando
A
r
, A

, e

A

; calcular o uxo de

A atraves de uma esfera de raio R = 5 centrada na origem; e, calcular
a circulacao de

A atraves de uma circunferencia de raio R = 2 centrada na origem e localizada no plano
xy.
Solucao: As coordenadas esfericas de

A sao:
A
r
= 2rsen
2
cos sen
r cos sen sen + 3rsen cos cos
A

= 2rsen cos sen cos r cos


2
cos 3rsen
2
cos
A

= 2rsen sen
2
r cos cos
Como


A = 0, implica
_ _

A d

S = 0
O rotacional


A = a
x
3 a
y
2 a
z
, e o teorema de Stokes permite obter:
_

A d

=
_ _
S
(


A) d

S = 8
PR 6.7: Seja o campo vetorial:

F = ru
r
+Rcos u

, calcule:
1. O uxo de

F atraves do cilindro fechado de altura 2h, raio R e centrado na origem.
2. A circulacao de

F atraves da circunferencia C : {r = 1, = /3}.
6. Fluxo, inducao e forca magnetica 40
Solucao: Fazendo uso da equacao:


F = 3
Rsen
rsen
e pelo teorema da divergencia teremos:
_ _
S

F d

S =
_ _ _ _
3
Rsen
rsen
_
r
c
dr
c
d dz = 6R
2
H
(Tambem pode-se fazer diretamente, sem usar o teorema da divergencia).
Agora, calculamos diretamente a integral de linha:
_

F d

=
_
2
0
Rcos (rsen )d = 0
PR 6.8: Dado o campo vetorial:

A =
_
xz
2
+xy
2
/2, x
2
y +yz
2
/2, y
2
z +x
2
z/2
_
calcular:
1. O rotacional de

A no ponto r = 5, = /2 e = /2;
2. O divergente de

A no ponto r
c
= 5, = /2 e z = 1; e,
3. A circulacao de

A atraves do retangulo com vertices: (2, 2, 1); (2, 2, 1); (25, 2, 1); (2, 2, 1).
Solucao: Usando diretamente a expressao do rotacional em coordenadas cartesianas:


A = yz a
x
+zx a
y
+xy a
z
=


A
(0,5,0)
= 0
Como r
c
= 5, = /2 e z = 1, e o mesmo que (x, y, z) = (0, 5, 1), calculamos o divergente:


A =
3
2
(x
2
+y
2
+z
2
) =


A
(0,5,1)
= 39
Pelo teorema de Stokes, a circulacao de

A e a integral de superfcie do rotacional de

A:
_

A d

=
_ _
(


A) d

S =
_
2
2
_
2
2
xydxdy = 0
PR 6.9: Seja o campo vetorial:

F = kr, onde k e uma constante.
1. Obtenha o divergente e o rotacional de

F.
2. Ache um campo escalar f tal que

F =

f; e se nao for possvel, explicar a razao.
3. De uma expressao para um campo vetorial

A tal que

F =


A; e se nao for possvel, explicar a
razao.
6. Fluxo, inducao e forca magnetica 41
Solucao: Aplicacao direta das expressoes de divergencia e rotacional em coordenadas cartesianas:


F = 3k


F = 0
O incremento innitesimal do campo escalar f e tal que:
df =

f d

=

F dr
df = k(x, y, z) (dx, dy, dz) = k(xdx +ydy +zdz)
e, fazendo a integral de df encontra-se f:
f =
_
df =
_
k(xdx +ydy +zdz)
f =
_
kxdx +
_
kydy +
_
kzdz
f =
k
2
(x
2
+y
2
+z
2
) + cte
Nao e possvel encontrar uma expressao para um campo vetorial

A tal que

F =



A porque


F = 0, e

F e um campo com divergencia (pois o rotacional do divergente e sempre nulo).
PR 6.10: Demonstrar que o campo vetorial

F = yz a
x
+ zx a
y
+ xy a
z
e irrotacional (



F = 0) e
solenoidal (


F = 0); encontre um campo

A tal que


A =

F.
Solucao: De acordo com a expressao para o rotacional em coordenadas cartesianas teremos:
_
Az
y

Ay
z
_
= yz;
_
Ax
z

Az
x
_
= zx; e
_
Ay
x

Ax
y
_
= xy.
Pode-se escrever as parcelas das derivadas parciais como:
A
z
y
= 2yz; e
A
y
z
= yz
A
x
z
= 2zx; e
A
z
x
= zx
A
y
x
= 2xy; e
A
x
y
= xy
Integrando estas equacoes obtemos:

A =
_
xz
2
+
xy
2
2
_
a
x
+
_
x
2
y +
yz
2
2
_
a
y
+
_
y
2
z +
zx
2
2
_
a
z

6. Fluxo, inducao e forca magnetica 42
PR 6.11: Comprovar o teorema da divergencia para o campo vetorial

F = ru
r
+rsen u

, atraves da
superfcie S denida pela esfera de raio R centrada na origem. Explicar o que acontece.
Solucao: Primeiro, fazemos a integral de superfcie de

F ao longo da superfcie da esfera:
_ _

F d

S =
_ _
(Ru
r
) cot(R
2
sen ddu
r
) = 4R
3
(6.1)
Agora, num segundo passo, fazemo o divergente de

F encontra-se


F = 4; (6.2)
e, fazendo sua integracao no volume da esfera, obtem-se:
_ _ _
V
(


F) dV =
16
3
R
3
Como (6.1) nao e igual a (6.2), aparentemente

F estaria violando o teorema da divergencia. Mas o
que acontece e que o campo

F nao e uma funcao, porque

F assume mais de um valor para um mesmo
angulo. Por exemplo,

F( = 0) =

F( = 2), e portanto

F nao e um campo vetorial.
PR 6.12: Calcular a integral de linha
_

F d

desde (0, 0, 0) ate (1, 2, 4) se



F = x
2
a
x
+y a
y
+(xz y) a
z
:
1. Ao longo do segmento de reta que une os pontos dados; e,
2. Ao longo da curva x = t
2
; y = 2t; z = 4t
3
Solucao: Substituindo

F e d

= dx a
x
+dy a
y
+dz a
z
, a integral de linha de

F ao longo de um caminho
qualquer C ca sendo
_
C

F d

=
_
C
x
2
dx +ydy + (xz y)dz
O segmento de reta que une os pontos (0, 0, 0) a (1, 2, 4) tem a seguinte equacao parametrica:
x = t, y = 2t; e z = 4t,
com respectivas derivadas:
dx = dt, dy = 2dt; e dz = 4dt.
Substituindo estes valores na integral de linha, ela resulta:
_
C

F d

=
_
t
t
2
dt + (2t)2dt + (4t
2
2t)4dt
=
_
t=1
t=0
_
17t
2
4t
_
dt =
11
3
Ao longo do caminho x = t
2
; y = 2t; z = 4t
3
, tem-se dx = 2tdt; dy = 2dt; e, dz = 12t
2
dt.
Substituindo na integral de linha, tem-se:
_
C

F d

=
_
t=1
t=0
_
48t
7
+ 2t
5
24t
3
+ 4t
_
dt =
7
3
Observa-se que os dois valores sao diferentes, isto e, a integral de linha por um caminho e diferente
da integral de linha pelo outro, signicando que o campo vetorial

F nao e um campo conservativo.
6. Fluxo, inducao e forca magnetica 43
PR 6.13: Dado o campo vetorial

F = r cos u
r
+zsen a
z
, expresso em coordenadas cilndricas, calcular
a circulacao de

F atraves de uma circunferencia de raio R centrada no eixo z e em um plano paralelo
ao plano xy.

F e um campo vetorial conservativo?
Solucao:
_

F d

=
_
=2
=0
(r cos u
r
+zsen a
z
) (Rdu

) = 0
Por esta integral, diramos que

F e conservativo. Mas, de acordo com a equacao do rotacional (nao
nulo),


F = z cos u
r
zsen u

+rsen a
z
=

0
arma-se que

F nao e um campo vetorial conservativo.
PR 6.14: Calcular
_ _
S
(


F) d

S quando F = (x
2
+y 4; 3xy; 2xz +z
2
) e S e a superfcie:
1. O hemisferio x
2
+y
2
+z
2
= 16 sobre o plano xy; e,
2. A parabola z = 4 (x
2
+y
2
) sobre o plano xy.
Solucao: Pelo teorema de Stokes:
_ _
S
(


F) d

S =
_

F d

.
Na circunferencia x
2
+y
2
= 16, e z = 0, tem-se
_

F d

=
_
(x
2
+y 4)dx + 3xydy
e, fazendo a troca de variaveis x = 4 cos , y = 4sen , dx = 4sen d e dy = 4 cos d obtem-se
_

F d

=
_
=2
=0
[(4 cos )
2
+ 4sen 4)
(4sen d) + 3(4 cos )(4sen )4 cos d]
_

F d

=
_
=2
=0
[64 cos
2
sen
16sen
2
+ 16sen + 192 cos
2
sen ]d = 16
Agora, fazendo o mesmo procedimento para a circunferencia de integracao x
2
+y
2
= 4
_

F d

= 4
PR 6.15: Usando os teoremas da divergencia e de Stokes (se for conveniente), calcular as seguintes
integrais:
1.
_ _
S
(xdydz +ydzdx +zdxdy onde S e uma esfera de raio R centrada na origem.
2. A circulacao de

F, onde

F = 3y a
x
+3x a
y
+ a
z
, e o contorno C e a circunferencia de raio R = 1
localizada no plano z = 2y centrada no eixo z.
6. Fluxo, inducao e forca magnetica 44
Solucao: A primeira integral pode ser escrita na forma:
_ _
S
(xdydz +ydzdx +zdxdy =
_ _
S
r d

S
=
_
=2
=0
_
=2
=0
R(R
2
sen dd) = 4R
3
Para a segunda integral, usando o teorema de Stokes, com


F = 6 a
z
, encontra-se
_

F d

= 6
PR 6.16: Um material com vetor magnetizacao

M = 1, 7 10
6
A/m, tem 8, 1 10
28
atomos por metro
c ubico. Qual e o momento de dipolo de cada atomo, em Am
2
?
PR 6.17: Qual e a corrente que seria necessaria para produzir um campo magnetico de 1, 7 10
6
A/m
em uma barra de ferro cilndrica, de comprimento = 10cm e diametro D = 3cm, que esta envolvida
por um solenoide de 100 espiras?
PR 6.18: Calcular a inducao magnetica de uma peca de ferro que tem permeabilidade relativa igual a
2500, quando o campo magnetico e de 300A/m.
PR 6.19: As linhas de uxo de

B podem fazer dobras, ou mudar abruptamente sua direcao? Por que?
PR 6.20: Considerando

H = 1400.0 a
x
4500.0 a
y
A/m, e o tensor de relutividade igual a
|||| =
_
_


_
_
=
1
||||
onde = .700E+04 m/H, e = .610E+03 m/H. Determinar o vetor de magnetizacao

M.
PR 6.21: A curva de desmagnetizacao de uma amostra de ma-permanente, apos imantado, e apresen-
tada na tabela.
Ponto Induc~ao (T) H desmag. (A/cm)
1 0.65 4.0
2 0.59 12.0
3 0.52 20.0
4 0.43 28.0
5 0.31 36.0
6 0.14 44.0
A densidade de uxo usada no entreferro de um instrumento de medida e escolhida para 0,09 weber por
metro quadrado. O comprimento do entreferro unico e 0,12 cm, e a area do entreferro e 10 centmetros
quadrados. Considerando que o uxo de dispersao seja igual ao uxo util no entreferro, calcule as
dimensoes do ma permanente com menor custo.
6. Fluxo, inducao e forca magnetica 45
PR 6.22: O campo vetorial

F = 10x a
x
poderia ser o vetor inducao magnetica

B? Por que?
PR 6.23: Uma carga puntual de 1,2C tem uma velocidade de 5 a
x
3 a
z
m/s. Encontre a intensidade
da forca sobre ela no campo: (a)

E = 18 a
x
+5 a
y
10 a
z
kV/mm; (b)

B = 0, 4 a
x
+0, 4 a
y
+0, 3 a
z
T;
e, (c)

E e

B juntos.
PR 6.24: Um o retilneo e percorrido por corrente I, perpendicularmente a um uxo magnetico uni-
forme. Usando o princpio da superposicao, mostre como cam as linhas de uxo proximo do condutor.
PR 6.25: Qual e a forca para movimentar ou xar um o condutor com secao 16mm
2
e 2m de compri-
mento, passando 100A, quando o mesmo esta imerso num campo de 10000A/m? (Resp.: 2,51 N)
PR 6.26: Considere um contator, contendo material magnetico ideal (
r
= ), que tem entreferro

ar
= 1mm, secao magnetica S = 4cm
2
, e uma bobina de N = 500 espiras percorrida por I = 2A. Qual
e a energia magnetica armazenada no contator? (Resp.: 126mJ)
PR 6.27: Demonstre a equacao da forca entre dois condutores paralelos.
PR 6.28: Uma ta de corrente de largura igual a 1,8cm conduz uma corrente de 25A na direcao positiva
do eixo x. Calcular a forca sobre a ta por unidade de comprimento se a inducao magnetica uniforme
for

B = 0, 5 a
y
T.
PR 6.29: Um momento magnetico de 1 Am
2
equivale a um conjugado de 1 Nm quando a espira estiver
orientada perpendicular `a inducao magn`etica de 1 Tesla, ou seja
= m

B
onde m = i

S. Demonstrar esta equacao e determinar o conjugado de uma espira de S = 20 8 cm


2
numa indu cao de 0,9 T e corrente i = 1 A.
PR 6.30: Enquanto que a unidade basica da eletricidade e a carga eletrica, a unidade basica do mag-
netismo e o dipolo magnetico. Entao, usando a expressao
0
p, alguns pesquisadores calculam o monopolo
magnetico ou carga magnetica, analogo `a carga eletrica. Qual e a carga magnetica Q

de um dipolo
p = 3, 0 10
9
Am? Qual e a expressao do campo magnetico criado pelo dipolo num ponto qualquer
do espaco? (Resp.:

H = Q

u
r
/(4
0
r
2
) A/m)
PR 6.31: O plano y +2z = 3 divide o espaco em duas partes: a regiao 1, contendo a origem do sistema
de coordenadas, possui
r1
= 5; e, a regiao 2, com
r2
= 20. Dado

B
1
= a
x
+ 0, 5 a
y
T, obter H
1
, H
2
e
B
2
.
6. Fluxo, inducao e forca magnetica 46
PR 6.32: Dois condutores paralelos estao afastados 5,5 cm, a uma diferenca de tensao 80 V ecaz, na
freq uencia de 60 Hz, e angulo de fase 0 graus. O primeiro condutor e percorrido por uma corrente
alternada, de 5,8 A, com angulo de fase 55,3 graus. O segundo condutor e percorrido por 8,9 A, com
angulo de fase 173,7 graus, tomando o mesmo sentido das correntes no circuito. Qual a intensidade e o
sentido das forcas atuando em cada condutor?
PR 6.33: Duas bobinas sao dispostas uma ao lado da outra, com seus eixos alinhados, colocadas no ar.
As duas bobinas sao percorridas por uma corrente I, de forma que a forca entre elas e de repulsao. Um
agente externo realiza forca em sentido contrario `a forca magnetica.
Caso Bob
1
Bob
2
F
externa
Inverte I
2
Aproxima Bob
2
7. INDUT

ANCIA E CIRCUITOS MAGN

ETICOS
PR 7.1: Qual e a tensao induzida num enrolamento com 1 cm
2
de secao, quando a inducao magnetica
varia uniformemente de 2.5 T para 1.0 T em 1 ms?
PR 7.2: Como funciona um transformador ideal? Quais suas equacoes basicas?
PR 7.3: Deduzir a equacao da indutancia por metro de comprimento, para uma linha de transmissao
constituda por um cabo coaxial com raio interno a e raio externo b.
PR 7.4: Deduzir a indutancia m utua entre um o innito e um circuito retangular.
PR 7.5: Duas linhas de transmissao a dois os condutores paralelos, tem afastamento d entre os centros
dos quatro condutores. Qual a indutancia m utua entre as duas linhas? (Resp.: M = 2/(15d))
PR 7.6: Uma espira de secao

S = 0, 02m
2
e atravessada pela inducao magnetica variavel no tempo, e
uniforme no espaco

B = 0, 5 a
x
+ 2, 0t
2
a
y
T
Qual e a equacao da tensao induzida na espira?
PR 7.7: Uma antena circular, de area A e resistencia R e perpendicular a um uxo magnetico uniforme,
que varia desde zero ate B num intervalo de tempo t. Qual e a expressao para a energia termica
dissipada na antena?
PR 7.8: Qual deve ser a relacao de espiras de um transformador de 220/5 Volts? Qual deve ser a secao
magnetica mnima se a densidade de uxo de pico for 1,5T e a freq uencia for 60Hz?
PR 7.9: Uma bobina retangular, de comprimento a e largura b e girada numa freq uencia f numa inducao
uniforme

B. Qual e a tensao induzida na bobina?
PR 7.10: Um pedaco de o retilneo esta colocado no eixo ox, desde x
1
= 0 ate x
2
= 3m. Qual e o
campo eletrico induzido no o quando a densidade de uxo e dada por

B = 1, 4 cos 250t Tesla? Qual
a f.e.m. nos terminais do o?
7. Indutancia e circuitos magneticos 48
PR 7.11: Qual deve ser a inducao magnetica

B(t) num ponto onde o campo eletrico seja dado por

E(t) = y cos t a
x
+xsen t a
y
V/m?
PR 7.12: Uma fonte chaveada tem um transformador operando a 50 kHz. Qual e a vantagem desta
freq uencia, em relacao a uma fonte de 60 Hz?
PR 7.13: Uma maquina eletrica tem potencia P
1
e um volume X
1
. Qual sera o volume X
2
para a
potencia P
2
, mantendo as mesmas caractersticas e proporcoes?
PR 7.14: Por que o acionamento de uma maquina eletrica tem por princpio manter a relacao V/f
constante?
PR 7.15: Sendo

B = 2 cos(310
5
t y) a
z
Wb/m
2
, ache a fem V (t) induzida no sentido generico +a

ao longo do caminho fechado:


1. (0,0,0) a (1,0,0) a (1,1,0) a (0,1,0) a (0,0,0);
2. (0,0,0) a (1,0,0) a (1,2,0) a (0,2,0) a (0,0,0).
PR 7.16: Considere uma inducao

B = 0, 4x a
z
Wb/m
2
e dois trilhos paralelos posicionados em x = 0 e
x = a = 5cm. Um voltmetro liga os dois trilhos com um o reto desde o ponto (0, 0) ate (0, a), e uma
barra deslizante paralela ao eixo y fecha o circuito. A posicao da barra e dada por x = 5, 4t3t
2
metros,
e se estende ate 2,4m alem do voltmetro.
1. Ache V
ab
quando a barra deslizante se encontra em x = 1m;
2. Esboce V
ab
em funcao do tempo.
PR 7.17: Um n ucleo magnetico em forma de C, tem secao magnetica de 4cm
2
e comprimento medio
do caminho magnetico de 12cm, e foi alimentado com tensao senoidal ecaz de 58,80 V, em 52,60 Hz,
e a corrente ecaz circulante e 25,70 A. O enrolamento possui 929 espiras, e entreferro e = 0, 91mm.
Pergunta-se: (a) O uxo magnetico circulante de pico. (b) A inducao magnetica de pico no ar. (c) A
inducao magnetica de pico no ferro. (d) A forca magneto motriz de pico. (e) O campo magnetico de
pico no ar. (f) O campo magnetico de pico no ferro. (g) A permeabilidade magnetica relativa do ferro.
(h) A maxima energia magnetica armazenada. (i) A indutancia nos terminais do enrolamento.
PR 7.18: Uma bobina, com N = 220 espiras, foi ligada a rede com tensao V = 220 Volts ecaz, e
freq uencia f = 60 Hz. Sabendo-se que esta bobina foi enrolada num n ucleo de ferro tipo C, e separado
por um entreferro, com as seguintes caractersticas: Secao magnetica = 4 cm
2
; Comprimento do ferro
= 13 cm; Permeabilidade relativa do ferro = 500; e, Entreferro = 0,5 mm. Determinar:
7. Indutancia e circuitos magneticos 49
O uxo magnetico;
A relutancia equivalente do circuito;
A for ca magneto motriz; e,
Corrente ecaz na bobina.
PR 7.19: Um disco de cobre com 150 mm de diametro esta girando com = 5 rpm, numa densidade
de uxo ortogonal B = 0, 8 T. Ligou-se uma escova de carvao no centro e outra na periferia do disco, e
conectou-se a um voltmetro. Qual a leitura do instrumento?

E um valor cc ou ca?
PR 7.20: Um condutor retilneo com 0, 2 m de comprimento gira no plano xy com um terminal colocado
na origem e com velocidade angular = 200 rpm, numa inducao magnetica

B = 0, 5 a
z
T. Qual a
f.e.m. induzida nos terminais do o?
PR 7.21: O polo Norte de um ma, que possui uxo total de 0, 048Wb, e aproximado de uma bobina
de 500 espiras, num intervalo de 0,1s. Qual e a tensao induzida na bobina?
Solucao:
V = 500
0, 048
0, 1
= 240 V.
PR 7.22: Considere uma barra condutora, que se move com velocidade v numa inducao magnetica

B.
B = B
0
cos t
determinar a equacao da tensao induzida V . Solucao: Devido ao movimento, tem-se
V =
_
(v

B) d

= vbB
0
cos t
A f.e.m. induzida pela variacao de

B no tempo e dada por Stokes, e vale
V =
_
S

B
t
d

S = abB
0
sen t
Somando as duas parcelas resulta
V = vbB
0
cos t +abB
0
sen t
V = B
0
b
_
v
2
+ (a)
2
sen (t +) V.
onde = tan
1
(v/a)
7. Indutancia e circuitos magneticos 50
PR 7.23: Uma leve moldura de bra, retangular, de area A = 0,01 m
2
funciona como carretel onde se
enrolam N = 42 espiras de o de cobre esmaltado. Esse quadro e posto a girar com freq uencia f = 60Hz
(r.p.s.) em um campo de inducao uniforme de intensidade B = 1, 00Wb/m
2
(ou, o mesmo que 1,00
tesla). Determinar a lei de variacao da forca eletromotriz induzida, em funcao do tempo.
Solucao: A velocidade angular do quadro e aproximadamente:
= 2f = 377 rad/s
resultando
V = 158 cos 377 t
sendo V em volts e t em segundos.
PR 7.24: Esse experimento tem por objetivo evidenciar o princpio de funcionamento dos transfor-
madores.
1. Construa duas bobinas toroidais, ambas com o de cobre esmaltado #24; uma com 200 a 300
espiras e a outra com 100 a 150 espiras. A bobina grande leva, em serie, um soquete para lampada
incandescente comum. Ligue o cordao de forca na tomada eletrica domiciliar e observe o brilho
da lampada pequena. Faca essa observacao usando no soquete comum lampada de 40W, 60W,
100W e 200W. Nao demore demasiado nessas observacoes para evitar aquecimento exagerado na
bobina grande.
2. Coloque a bobina pequena dentro da bobina grande, ajustando bem. Aos terminais A e B da
bobina pequena ligue uma lampada para 6V (usadas em lanternas de 4 pilhas). Sempre e bom
dispor de soquete para tal lampada para facilitar as ligacoes e as trocas. Na falta dele, basta
soldar as extremidades dos os A e B (devidamente lixadas) aos terminais da lampada.
3. Varie a posicao relativa entre as duas bobinas e verique a tensao no terminais da bobina menor.
A relacao entre as duas bobinas e chamada de indutancia m utua.
4. Depois, no interior do conjunto passe um feixe de laminas de ferro-silcio. Repita todo o proced-
imento anterior para essa nova situacao ... n ucleo de ferro dentro das bobinas. Se as laminas
envolvem as bobinas e, com isso, ocorre maior concentracao das linhas de inducao, aumentando
a corrente induzida no secundario (bobina menor). Repita todo o procedimento anterior nova-
mente, observando o brilho da lampadinha. Troque a lampada (40W, 60W, 100W e 200W) em
cada observacao.
PR 7.25: Considerando que a dimensao a = 5cm, calcular a potencia do transformador, quando f =
60Hz, B = 1T, e J = 4A/mm
2
. Considere um fator de empilhamento das laminas do n ucleo igual a
0,95 e fator de ocupacao da janela pelo enrolamento de 0,35.
7. Indutancia e circuitos magneticos 51
Solucao: A tensao e a corrente nos enrolamentos e
V = 4, 44 f N B A
m
C
m
I = J A
w
C
w
/ N
e a potencia do transformador S(VA) e
S = V I = 4, 44 A
m
A
w
C
m
C
w
f B J
As secoes magnetica e de janela sao
A
m
= 2a 2a = 4a
2
= 100 10
4
m
2
A
w
= 1, 5a 3a = 4, 5a
2
= 112, 5 10
4
m
2
Assim, temos a potencia
S = 4, 44 100 10
4
112, 5 10
4
0, 95 0, 35 60 1 4 10
6
= 9965 VA
PR 7.26: Considere um transformador de potencia de 50 kVA que tem uma relacao de tensao de
10000V/200V. A partir do kVA e da tensao, a corrente de carga do enrolamento da maior tensao e
50000VA/ 10000V = 5A, e do enrolamento de menor tensao e 50000VA / 200V = 250 A. Tal transfor-
mador com carga maxima com seu enrolamento conectado ao terminal sem ponto do outro enrolamento.
O circuito secundario de 10000 V pode ser carregado com no maximo 250 + 5 = 255 A sem que um dos
enrolamentos tenha uma corrente de sobrecarga. Sendo que a corrente da fonte e 250 A, o transformador
pode fornecer 10200 x 255 = 2550kVA. Isto pode ser tambem determinado pelo circuito do secundario:
10000 x 255 = 2550 kVA. Na verdade, a conexao como autotransformador teve um aumento na potencia
de 50 para 2550 kVA.
A explicacao para esse aumento e que o transformador original de 50 kVA nao tem conexao metalica
entre os dois enrolamentos, e entao o 50 kVA devem ser transmitidos atraves do transformador pelo
acoplamento magnetico. Mas com os enrolamentos conectados para fornecer a operacao de auto - trans-
formacao, existe uma conexao metalica entre os dois enrolamentos que transmite 2550 - 50 = 2500 kVA
sem ser transformada magneticamente. Assim, essa conexao metalica e que fornece o aumento de kVA.
Embora vantajoso a esse respeito, tal conexao destroi a propriedade de isolacao dos transformadores
convencionais, o signica que a auto - transformacao nao pode ser aplicada em qualquer transformador.

PR 7.27: Demonstrar a expressao


v
L
= L
di
L
dt
7. Indutancia e circuitos magneticos 52
PR 7.28: Demonstrar a equacao da energia armazenada num indutor.
PR 7.29: Considere uma bobina de indutancia igual a 1 Henry, que esta sendo percorrida pela corrente
I = 20A, e que a corrente seja interrompida a zero num intervaldo de 0,1s. O que acontece com esta
bobina? O que poderia ser feito para amenizar este efeito?
PR 7.30: Indutancia entre dois os paralelos - Considerem-se dois os condutores paralelos, que sao
percorridos por correntes eletricas com sentidos opostos e mesma intensidade i
1
= i
2
= i. Nestas
condicoes, a intensidade do campo magnetico gerado por qualquer um dos dois condutores num ponto
P do plano (no plano denido pelos dois condutores) e dada pela expressao
H =
i
2d
em que d = d
1
ou d = d
2
dene a distancia entre o condutor 1 ou 2 e o ponto. Pode-se calcular o uxo
por unidade de comprimento, considerando
r
= 1, e tendo em conta os sentidos opostos das correntes,
a integral da densidade do uxo magnetico criado pelos dois os conduz ao resultado
=

0
i
2
_
d+r/2
r/2
(
1
r
+
1
d r
) dr
=

0
i

ln(
d
r
)
e na qual se inscreve a indutancia por unidade de comprimento:
L =

0

ln(
d
r
) H/m
Este procedimento pode ser adotado para calcular a indutancia de outras estruturas de corrente eletrica.

PR 7.31: Indutancia de um cabo coaxial - Calcular a indutancia linear de cabo coaxial reto e innito.
Resposta: O valor da indutancia por metro e
L =

0
2
ln(
r
ext
r
int
) H/m
PR 7.32: Indutancia de uma bobina com n ucleo toroidal - Nos exemplos considerados, calculou-se o
uxo magnetico em superfcies convencionais, como sejam, por exemplo, o plano denido pelos dois
condutores paralelos e o plano no qual se inscreve o diametro dos condutores concentricos caractersticos
do cabo coaxial. No entanto, no caso das bobinas com N espiras e n ucleo cilndrico ou toroidal, o uxo
magnetico deve ser multiplicado por N espiras, para obter-se o uxo concatenado.
7. Indutancia e circuitos magneticos 53
O campo magnetico e dado pela equacao 5.4. A inducao magnetica no n ucleo e igual ao produto
do campo magnetico multiplicado pela permeabilidade magnetica do material do toroide. O uxo
magnetico e o produto da secao S
m
do toroide pela inducao. Assim, o uxo concatenado vale
=
0

r
S
m
1
2 r
N
2
I
e a indutancia resulta
L =
0

r
S
m
L
m
N
2
Observacoes:
a) A indutancia e diretamente proporcional ao quadrado do n umero de espiras;
b) A indutancia e diretamente proporcional `a secao transversal, e inversamente proporcional ao
comprimento do caminho magnetico, que neste caso vale L
m
= 2r.
c) A indutancia e diretamente proporcional `a permeabilidade magnetica.
d) Obviamente, se quizermos aumentar ou diminuir a indutancia, devemos variar estes parametros.
e) Normalmente, quando desejamos diminuir a corrente de uma bobina devemos aumentar a
indutancia.
f) Normalmente, quando aumenta a potencia de um equipamento, diminui a indutancia.
PR 7.33: Considerando um indutor, com N espiras, envolvendo um circuito magnetico simples, com
um material de permeabilidade , de comprimento medio
m
, area media da secao transversal S
m
, e
indutancia L, demonstrar que

B =

H e a forma local da equacao N = L I.


Solucao: Partindo da equacao da indutancia
L = N

I
= N
2
S
m

m
e, considerando que H = N I/
m
; = B S
m
, encontra-se
L = N
B S
m
H
m
/N
= N
2
S
m

m
que simplicando os termos S
m
e
m
resulta:
=
B
H
ou

B =

H
PR 7.34: Ao longo do percurso da ciencia, deniram-se muitas unidades e leis. As denicoes basicas ou
fundamentais sao os materiais, e suas relacoes de causa-efeito. Quais os principais meios ou materiais
existentes na natureza e estudadas no eletromagnetismo, e suas relacoes constituintes?
Resposta: Os tres meios sao: condutores, dieletricos e magneticos, que sao resumidos na tabela 7.1.

7. Indutancia e circuitos magneticos 54


Tab. 7.1: Meios condutores, dieletricos, e magneticos.
Fonte Fluxo Relac~ao
Condutor V ou

E I ou

J R ou
Dieletrico V ou

E Q ou

D C ou
Mangetico I ou

H ou

B L ou
PR 7.35: Qual e o campo magnetico no interior de um material magnetico ideal? Por que?
PR 7.36: Um uxo magnetico de 24000 linhas (maxwell) atravessa uma superfcie de 6 cm
2
. Qual o
valor da densidade de uxo ou inducao magnetica? (R:0,4 T)
1 maxwell = 1 linha de campo.
1 Gauss = 1 linha de campo / cm
2
.
1 Tesla = 1 T = 1 Wb/m
2
= 10
4
Gauss.
PR 7.37: Quantas espiras sao necessarias para produzir uma inducao de 1 T num entreferro de 5 mm,
quando pelo condutor se deslocam 1 A? (R:3979 espiras)
PR 7.38: Considerando um circuito magnetico simples, pretende-se obter no entreferro de 2mm, uma
inducao magnetica de 0,95 Teslas. Quanto deve valer a corrente, se o n umero de espiras for N = 1000?
(R: I

= 1, 6A)
PR 7.39: Um n ucleo ferromagnetico toroidal, com permeabilidade relativa igual a 1000, com raio medio
de 10cm e secao de 3cm
2
, deve ser atravessado por uma densidade de uxo magnetico igual a 1,2 T.
Qual a for ca magnetomotriz necessaria?
PR 7.40: Qual a inducao magnetica num toroide de ferro, com raio medio de 10 cm e permeabilidade
magnetica relativa igual a 100, que e envolvido por 200 espiras, percorrido por 40 A. (R: 1,6 T)
PR 7.41: Um n ucleo laminado possui comprimento medio de 12,6 cm, e permeabilidade relativa
r
=
4890. Qual o n umero de espiras necessario para produzir uma inducao de 1,3T, quando pelo condutor
se delocam 1,4A?
PR 7.42: Um n ucleo ferromagnetico retangular simples, sem entreferro, possui tres lados com secao de
150 cm
2
, e comprimento total de 130 cm, e o quarto lado possui secao de 100 cm
2
e comprimento de 45
cm. O n umero de espiras e 200 e a permeabilidade relativa
r
= 2500. Qual e o uxo produzido por 1
A de corrente na bobina? (

= 0, 0048 Weber)
PR 7.43: Um n ucleo ferromagnetico quadrado, tem um comprimento medio de 55 cm e uma area de
150 cm
2
. A bobina tem 200 espiras. Escolher o material e considerando sua curva B H, calcule:
7. Indutancia e circuitos magneticos 55
1. Qual a corrente requerida para produzir 0,012 Wb de uxo no n ucleo?
2. Qual e a permeabilidade relativa do ferro neste nvel de corrente?
3. Qual e a relutancia do ferro?
PR 7.44: Um circuito magnetico simples tem dois materiais de mesma secao colocados em serie. Qual
deve ser a relacao entre os comprimentos do caminho magnetico para que a energia total nos dois
materiais sejam iguais.
PR 7.45: Dado o circuito magnetico,que tem quatro entreferro com espessura e = 1mm, e secao
magnetica S = 4cm
2
. Escreva as equacoes de circulacao de campo e continuidade de uxo. Cal-
cule os campos H nos entreferros quando NI = 1000A, e o uxo total gerado pela bobina. (Resp.:
428,719A/m e 3,3E-4Wb)
PR 7.46: Calcular a indutancia m utua M
12
entre dois aneis com seus eixos coindicentes, e afastados de
uma distancia c, sendo o primeiro de raio a e o segundo de raio b. Considere a >> b e n
1
= n
2
= 1.
1
o
. Calculo do campo magnetico
O campo magnetico no eixo de um anel e
H =
I R
2
2(R
2
+x
2
)
3/2
Nesse caso, H = H
1
, I = I
1
, R = a e ,x = c, de modo que
H
1
=
I
1
a
2
2(a
2
+c
2
)
3/2
2
o
. Calculo do uxo
Sendo a >> b, pode-se considerar o campo

H da primeira bobina de raio a constante na segunda
de raio b. Assim, o uxo da primeira bobina, que passa atraves da segunda e

12
=
_ _
S
2

0
H
1
dS =
0
H
1
S
2

12
=
0
I
1
a
2
2(a
2
+c
2
)
3/2
b
2

12
=

0
I
1
a
2
b
2
2(a
2
+c
2
)
3/2
3
o
. Calculo da indutancia m utua
A indutancia m utua e igual ao uxo concatenado (produto do uxo pelo n umero de espiras) dividido
pela corrente que originou este uxo.
M
12
= n
2

12
I
1
7. Indutancia e circuitos magneticos 56
Como n
2
= 1, a indutancia m utua e
M
12
=

0
a
2
b
2
2(a
2
+c
2
)
3/2
Este e o procedimento padrao para o calculo da indutancia: calculo do campo magnetico; calculo do
uxo e calculo da indutancia m utua. Observa-se que, considerou-se a >> b, para calcular o valor da
indutancia m utua. Entretanto, se desejarmos um valor mais exato, precisaremos fazer uso de metodos
numericos, como o arquivo MUTUA.exe, disponvel na pasta DEMAG.
PR 7.47: Consideremos duas bobinas proximas, percorridas por correntes I
1
e I
2
, variaveis no tempo.
Solucao: Dividindo o n umero de linhas pela corrente, temos as indutancias:
L
11
= 5 linhas / 5 A = 1,00 linha / ampere;
L
12
= 2 linhas / 8 A = 0,25 linha / ampere;
L
22
= 6 linhas / 8 A = 0,75 linha / ampere; e,
L
21
= 3 linhas / 5 A = 0,60 linha / ampere.
PR 7.48: Coeciente de acoplamento. Vamos supor que as duas correntes I
1
e I
2
estejam ligadas ao
mesmo tempo. Determinar os uxos nas bobinas e o coeciente de acoplamento k.
Solucao: Considerando positivo o uxo o produzido pela corrente propria, temos

1
= 5 2 = 3 linhas

2
= 6 3 = 3 linhas
Observa-se que 2/5 das linhas se perdem em (a) e 4/6 se perdem em (b). Caso nenhuma linha se
dispersasse, o coeciente de acoplamento seria k = 1. Mas, com a dispersao, temos a media geometrica
k =
_
2
5

4
6
= 0, 516
PR 7.49: Escrever as equacoes das tensoes V
1
e V
2
e das correntes I
1
e I
2
das bobinas, que variam
senoidalmente com freq uencia .
Solucao: Considerando positivo o uxo produzido pela corrente propria, temos para correntes quais-
quer
V
1
= L
11
I
1
L
12
I
2
V
2
= L
21
I
1
+L
22
I
2
Com as correntes e indutancias dadas
V
1
=
d
1
dt
= (1 5 0, 25 8) = 3V
V
2
=
d
2
dt
= (0, 75 8 0, 60 5) = 3V
7. Indutancia e circuitos magneticos 57
Observa-se que, a relacao 1:1 nao signica transformador ideal, pois:
S
1
= V
1
I
1
= 3 5 = 15
S
2
= V
2
I
2
= 3 8 = 24
PR 7.50: Duas bobinas chatas com eixos coinncidentes tem raios R
1
e R
2
, estao afastadas uma da outra
da distancia d. Usando o programa MUTUA.EXE, disponvel na pasta DEMAG, completar a tabela
com os valores da indutancia m utua e explicar o seu comportamento.
R
1
(cm) R
2
(cm) d L
12
0,1 0,35 0,2 3,6677E-10 H
1,0 3,5 2,0 3,6677E-9 H
10 35 20 3,6677E-8 H
100 350 200 3,6677E-7 H
Solucao: A indutancia m utua diminui proporcionalmente com as dimensoes, porque o uxo tambem
diminui proporcionalmente com as dimensoes, para uma mesma corrente. Quando diminui as dimensoes
o campo magnetico aumenta proporcionalmente. Embora diminua o caminho magnetico e aumente o
campo 10 vezes, com a reducao de 10 vezes das dimensoes, a area diminui 100 vezes com o quadrado
do raio, e assim o uxo diminui 10 vezes.
PR 7.51: Um indutor com n ucleo de ar, na forma de um solenoide de comprimento d = 3cm, area media
A = r
2
= 12cm
2
e com N = 1000 voltas, tem uma indutancia
L =
0
N
2
A
d
= 50 mH.
O permetro medio de cada espira e 2r = 10, 3cm, o que da um comprimento total l
tot
= 123 metros. Se
o o e de cobre (resistividade = 1, 8 10
6
cm, de diametro D = 0, 25mm (area da secao transversal
S = D
2
/4), entao a resistencia serie desse indutor e
r
s
= l
tot
/S = 45.
Para uma frequencia de 100 Hz, a reatancia e X
L
= 2fL = 31, 4, que e menor que a sua resistencia
interna. Por outro lado, para uma frequencia de 10 MHz, X
L
= 188M >> r
s
(mesmo considerando
o efeito pelicular, que daria r
s
= 130). Apesar disto, em certos casos, principalmente em circuitos
ressonantes, r
s
nao podera ser ignorada, mesmo que a frequencia seja alta. A frequencias mais altas e
necessario considerar a capacitancia parasita entre as espiras da bobina, c
p
, em paralelo com o indutor.

8. FASORES
PR 8.1: (Ulaby Ex 7.1) O campo eletrico de uma onda eletromagnetica e
E(z, t) = 10 cos( 10
7
t + z/15 +/6) V/m.
Determinar: (a) a direcao de propagacao da onda; (b) a frequencia da onda; (c) o comprimento de onda
; e, (d) a velocidade de propagacao ou de fase u
p
.
Solucao:
(a) z porque os dois coecientes de t e z tem mesmo sinal (neste caso positivo).
(b) f =

2
=
10
7
2
= 5 MHz
(c)
2

=

15
= 30 m
(d) u
p
= f = 1, 5 10
8
m/s
PR 8.2: (Ulaby Ex 7.2) Uma onda eletromagnetica propaga-se na direcao +z num meio com constante
de atenuacao = 0, 5 Np/m. Se a amplitude do campo eletrico e 100 V/m em z = 0, qual e a distancia
para que a amplitude seja atenuada para: (a) 10 V/m; (b) 1 V/m; e, (c) 1 V/m?
Solucao:
(a) 100e
0,5z
= 10 z = 4, 6 m
(b) 100e
0,5z
= 1 z = 9, 2 m
(c) 100e
0,5z
= 10
6
z = 37 m
PR 8.3: (Ulaby Ex 7.3) Representar as funcoes complexas z
1
= (4 j3)
2
e z
2
= (4 j3)
1/2
na forma
polar.
Solucao:
z
1
= (4 j3)
2
=
_
(4
2
+ 3
2
)
1/2

tan
1
3/4
_
2
= [5

36, 87

]
2
= 25

73, 7

z
2
= (4 j3)
1/2
=
_
(4
2
+ 3
2
)
1/2

tan
1
3/4
_
1/2
= [5

36, 87

]
1/2
=

18, 4


PR 8.4: (Ulaby Ex 7.4) Demonstrar que

2j = (1 +j).
Solucao:
e
j/2
= 0 +jsen (/2) = j
_
2j = [2e
j/2
]
1/2
=

2e
j/4
= (1 +j).
8. Fasores 59
PR 8.5: (Ulaby Ex 7.5) Um circuito RL serie (R = 400 , L = 3mH e = 10
5
rad/s) e conectado
a uma fonte de tensao v(t) = 150 cos t Volts. Qual e: (a) o fasor de corrente I
s
; e, (b) a corrente
instantanea?
Solucao:
I
s
=
V
s
R +jL
=
150
400 +j300
= 0, 3

36, 9
o
A
i(t) = [I
s
e
jt
] = [0, 3 e
j36,9
o
e
jt
] = 0, 3 cos(10
5
t 36, 9
o
) A
PR 8.6: (Ulaby Ex 7.6) O fasor da tensao e V
s
= j5 V. Encontre v(t).
Solucao:
v(t) = [V
s
e
jt
] = [5 e
j/2
e
jt
] = 5 sen t; V
PR 8.7: Um predio e alimentado com tres os vivos de 127 V (ecazes) e fases (vivos) R, S e T. A
diferenca de fase entre dois fases quaisquer e de 120 graus. Represente as tres tensoes no plano complexo
e mostre que a diferenca de potencial entre dois vivos quaisquer e DV cos(t +2/3), onde DV = 311, 1
Volts (pico) ou 220 Volts ecazes.
Solucao: As tres fases estao defasadas de 120

eletricos, e formam uma estrela, que ligam o baricentro


ate cada ponto de um triangulo equilatero. Se este triangulo for cinscunscrito num crculo de raio igual
a 127 Volts, podemos demonstrar facilmente que os seus lados correspondem a 220 Volts.
PR 8.8: Suponha que a tensao v(t) e a corrente i(t) senoidais sejam v(t) = 110 cos 377t volts e
i(t) = 8, 39 cos(377t /6) amps. Comprovar que a equacao da potencia ativa P = V I cos e a media
de p = vi num perodo.
Solucao: A potencia ativa P e a media das potencias instantaneas p(t) = v(t) i(t) num perodo
P =
1
T
_
T
0
p(t)dt
=
377
2
_
2/377
0
924 cos 377t cos(377t /6)dt
= 5, 53 10
4
_
2/377
0
cos 377t(cos 377t cos /6 + sen 377tsen /6)dt
= 5, 53 10
4
_
2/377
0
(0, 866 cos
2
377t + 0, 217sen 754t)dt
= 400 watts
PR 8.9: Mediu-se a tensao v(t) e a corrente i(t) de um dispositivo desconhecido, encontrando-se v =
220sen 377t Volts e i = 11 cos 377t Amperes. Determine o circuito equivalente do elemento desconhecido.
Solucao: Escrevendo-se os fasores de tensao e corrente, obtem-se a impedancia:
Z =
220

90

11

= j20
Como Z e puramente imaginario negativo, corresponde a uma capacitancia pura, onde:
C =
1
X
C
=
1
377 . 20
= 133 Farads.
8. Fasores 60
PR 8.10: Mediu-se a tensao v(t) e a corrente i(t) de um dispositivo desconhecido, encontrando-se
v = 220sen 377t Volts e i = 11 cos 377t Amperes. Determine o circuito equivalente do elemento
desconhecido.
Solucao: Escrevendo-se os fasores de tensao e corrente, obtem-se a impedancia:
Z =
220

90

11

180

= 20

270

= 20

90

= +j20
Como Z e puramente imaginario positivo, corresponde a uma indutancia pura, onde:
L =
X
L

=
20
377
= 53 milihenrys.
PR 8.11: Um motor eletrico alimentado com tensao alternada de 110 V consome uma corrente de 5 A
atrasada de 20 graus eletricos da tensao. Desenhar as formas de onda de v(t) e i(t) e calcular a potencia
media num perodo. Qual e a equacao da potencia?
Solucao: Sabemos que a potencia e o produto de V e I ecazes pelo fator de potencia cos :
P = V I cos = 110 5 cos 20

= 516, 83 Watts
PR 8.12: Demonstrar que a soma de duas impedancias complexas em corrente alternada e identica ao
caso de resistencias em corrente contnua.
Solucao: Duas impedancias em serie tem a mesma corrente, e a tensao total e a soma das tensoes
de cada impedancia.
V = V
1
+V
2
+. . . = Z
1
I +Z
2
I +. . . = (Z
1
+Z
2
+. . .) I = Z I
Z = Z
1
+Z
2
+. . .
PR 8.13: Demonstrar que a soma de duas admitancias complexas em corrente alternada e identica ao
caso de duas condutancias em corrente contnua.
Solucao: Duas admitancias em paralelo tem a mesma tensao, e a corrente total e a soma das correntes
de cada admitancia.
I = I
1
+I
2
+. . . = Y
1
V +Y
2
V +. . . = (Y
1
+Y
2
+. . .) V = Y V
Y = Y
1
+Y
2
+. . .
PR 8.14: Dado um circuito serie Z = R + jL , encontre o seu circuito equivalente paralelo Y =
Gj/(L) Siemens?
PR 8.15: Dado um circuito paralelo Y = G + jC Siemens, encontre o seu circuito equivalente serie
Z = R +jL j/(C) ?
8. Fasores 61
Fig. 8.1: Circuito RLC, para calculo da impedancia de entrada usando a Carta de Smith.
PR 8.16: Determinar a impedancia de entrada ou equivalente do circuito eletrico de componentes con-
centrados apresentado na Fig. 8.1, na frequencia de 100 MHz. Sugestoes: normalizar as impedancias e
admitancias, usando a referencia base de 50 , somar impedancias em serie e admitancias em paralelo.
Solucao: Inicialmente, calculamos as impedancias e admitancias normalizadas:
C1= 4,00E-11 Farads
YC1= 2,51E-02 Siemens = 1,256637061 p.u.
ZC1= -3,98E+01 Ohms = -0,795774715 p.u.
L1= 5,30E-08 Henrys
ZL1= 3,33E+01 Ohms = 0,666017643 p.u.
YL1= -3,00E-02 Siemens = -1,501461727 p.u.
C2= 1,38E-10 Farads
YC2= 8,67E-02 Siemens = 4,34E+00 p.u.
ZC2= -1,15E+01 Ohms = -2,31E-01 p.u.
C3= 3,60E-11 Farads
YC3= 2,26E-02 Siemens = 1,130973355 p.u.
ZC3= -4,42E+01 Ohms = -0,884194128 p.u.
No passo seguinte, partimos da carga para a fonte . . . :
ZA= 1 .+j. -0,795774715
YA= 0,612273363 .+j. 0,487231661
YB= 0,612273363 .+j. -1,014230066
ZB= 0,436234662 .+j. 0,722622175
ZC= 0,436234662 .+j. 4,92E-01
YC= 1,009036072 .+j. -1,137938575
YD= 1,009036072 .+j. -6,97E-03
ZD= 0,990997627 .+j. 0,006840703
ZIN= 49,54988134 .+j. 0,342035157
Correspondendo ao valor de 50 .
PR 8.17: Escreva a impedancia complexa para cada caso do arquivo Impedancia.xls.
9. CAMPOS DIN

AMICOS - LEI DE FARADAY


PR 9.1: Considerando a densidade de uxo magnetico

B = B
0
e
kt
a
z
, situado na regiao cilndrica < b.
Qual e a f.e.m. induzida e o campo eletrico em uma espira de raio = a, onde a < b, no plano z = 0?
Solucao: A f.e.m.

dada pela Lei de Faraday
f.e.m. =
_
d

B
dt
d

S = k B
0
e
kt
a
2
e, devido a simetria do problema, a f.e.m. e o campo multiplicado pelo permetro,
f.e.m. = 2a E

logo o campo eletrico induzido e:

E =
1
2
k B
0
e
kt
a


PR 9.2: Hayt 10.1 Na Fig. 9.1, seja B(t) = 0, 2 cos 120t T, assumindo um resistor perfeito de 250 ,
e desconsiderando o campo magnetico produzido pela corrente induzida I(t), determine: a) V
ab
(t); e,
b) I(t).
Solucao: A f.e.m. e dada pela Lei de Faraday
f.e.m. = V
ba
(t) = d/dt = (120)(1, 41 10
2
)sen 120t = 5, 33sen 120t V.
e, a corrente induzida e:
I(t) = V
ba
(t)/R =
5, 33sen (120t)
250
= 21, 3sen (120t) mA
PR 9.3: Hayt 10.2 Sendo a inducao magnetica variavel no tempo

B = (0, 5 a
x
+0, 6 a
y
0, 3 a
z
) cos 5000t
T, e uma espira retangular com cantos em(2,3,0), (2,-3,0), (-2,3,0), e (-2,-3,0), determine a corrente
induzida se a resistencia total da espira e 400 k.
Solucao: A f.e.m. e dada pela Lei de Faraday
f.e.m. = emf =
_

E d

L = d/dt =
_ _
superf.

B a
z
dS
f.e.m. =
d
dt
(0, 3)(4)(6) cos 5000t = 7, 2(5000)sen 5000t V.
e, a corrente induzida e:
I(t) =
f.e.m.
R
=
7, 2(5000)sen 5000t
400 10
3
= 90sen 5000t mA
9. Campos Dinamicos - Lei de Faraday 63
Fig. 9.1: Espira circular no campo magnetico.
PR 9.4: Hayt 10.3 Seja

H = 300 a
z
cos(3 10
8
t y) A/m no vacuo, determinar a f.e.m. desenvolvida
na espira retangular situada nos cantos (0,0,0), (1,0,0), (1,1,0), e (0,1,0).
Solucao: O uxo magnetico e
=
_
1
0
_
1
0
300
0
cos(3 10
8
t y) dx dy = 300
0
sen (3 10
8
t y)|
1
0
= 300
0
sen (3 10
8
t 1) sen (3 10
8
t) Wb
A tensao induzida e
f.e.m. =
d
dt
= 300 (3 10
8
)(4 10
7
) = cos(3 10
8
t 1) cos(3 10
8
t)
f.e.m. = 1, 13 10
5
_
cos(3 10
8
t 1) cos(3 10
8
t)
_
V
PR 9.5: Hayt 10.5 A posicao da barra deslizante na Fig. 9.2 e dada por x = 5t + 2t
3
, e a separacao
das duas barras xas (trilhos) e 20 cm. Considerando

B = 0, 8x
2
a
z
T, determine a leitura no voltmetro
no instante de tempo t = 0, 4 segundos.
Solucao: O uxo magnetico e
=
_
0,2
0
_
x
0
0, 8(x

)
2
dx

dy =
0, 16
3
x
3
=
0, 16
3
(5t + 2t
3
)
3
Wb
A tensao induzida e
f.e.m. =
d
dt
= 0, 16 (5t + 2t
3
)
2
(5 + 6t
2
) = 4, 32 V
PR 9.6: (Wentworth 5.1) Qual o tempo gasto para que uma densidade de carga decaia de um valor
qualquer para 1% do seu valor inicial, no material poliestireno?
9. Campos Dinamicos - Lei de Faraday 64
Fig. 9.2: Barra deslizante no campo magnetico.
Solucao: O poliestireno tem
r
= 2, 56 e = 10
17
Siemens/metro.

v
=
0
e
(/)t
0, 01
0
=
0
exp
_
10
17
2, 56 8, 854 10
12
_
=
0
e
4,4110
9
t = 10, 4 10
6
segundos = 120 dias
PR 9.7: (Wentworth 5.5) A densidade de uxo magnetico aumenta `a taxa de 10 Wb/m
2
/s na direcao
z. Uma espira condutora quadrada com 10x10 cm
2
possui uma resistencia distribuda de 10 . Deter-
mine a direcao (por meio de um desenho) e a intensidade da corrente induzida na espira condutora.
Solucao:
d

B
dt
= 10
Wb
m
2
s
a
z
V
emf
=
_
d

B
dt
d

S =
_
10
Wb
m
2
s
a
z
dx dy a
z
V
emf
= 0, 1
Wb
s
V s
Wb
= 0, 1 V
I =
0, 1 V
10
= 10 mA
Sentido horario, quando visto do eixo +z.
9. Campos Dinamicos - Lei de Faraday 65
PR 9.8: (Wentworth 5.7) No ar, uma espira condutora estacionaria e posicionada proxima a uma
linha innita de corrente como ilustrado na Fig.5.17. A corrente linear aumenta com o tempo pela
relacao i(t) = 110(A/s) t. Se a distancia a e igual a 4 cm, calcule a tensao V
R
na espira.
Solucao: Adotaremos i(t) = A t, onde A = 100 A/s. Agora podemos resolver
V
emf
=

t
_

B d

S
Sabendo-se que V
emf
= V
R
, devemos fazer a integracao no sentido horario, adotando a regra da mao
direita d

S = dy dz ( a
x
). Da linha de corrente de comprimento innito, tem-se:

B =

0
i
2
a

=

0
A t
2 y
( a
x
)
E entao

B
t
=

0
A t
2 y
( a
x
)
V
R
=
_

0
A t
2 y
( a
x
) dy dz ( a
x
) =

0
A
2
_
2a
a
dy
y
_
a
0
dz =

0
A a
2
ln(2)
Substituindo-se os valores numericos, V
R
= 610 nV.
PR 9.9: (Wentworth 5.8) Considerando a Fig.5.2, suponha que a area de uma unica espira de um par
seja 100 cm
2
e que a densidade de uxo magnetico seja constante sobre a area das espiras, mas que se
modique com o tempo como

B = B
0
e
t
a
z
, onde B
0
= 4 mWb/m
2
e = 0, 30 Np/s. Determine V
R
em t = 1, 10, e 100 segundos.
Solucao:
V
emf
= N
_

B
t
d

B
t
= B
0
e
t
a
z
V
emf
= 2 B
0
e
t
S = 24 10
6
e
0,30 t
para t = 1 s, V
R
= 17, 8 V para t = 10 s, V
R
= 1, 20 V para t = 100 s, V
R
= 2, 25 10
18
Volts.
PR 9.10: (Wentworth 5.9) Um transformador e algumas vezes utilizado como um conversor de impe-
dancia, onde ela e dada por V/I. Determine a expressao para a impedancia Z
1
vista pelo lado primario,
para um transformador que possui uma impedancia Z
2
na carga do lado secundario.
Solucao: Sabendo-se que i
2
=
N
1
N
2
i
1
, v
2
=
N
2
N
1
v
1
, e Z
1
=
v
1
i
1
, tem-se:
Z
2
=
v
2
v
1
=
N
2
N
1
v
1
N
1
N
2
i
1
=
_
N
2
N
1
_
2
Z
1
Z
1
=
_
N
1
N
2
_
2
Z
2

9. Campos Dinamicos - Lei de Faraday 66
PR 9.11: (Wentworth 5.10) Um o de cobre com 1,0 mm de diametro e modelado como uma espira
quadrada de lado 4,0 cm. Ela e colocada em um plano normal a um campo magnetico que aumenta
com o tempo pela expressao

B = 1, 0 t a
z
Teslas, onde t e dado em segundos. (a) Calcule a magnitude
da correne induzida e indique sua direcao, com um desenho. (b) Calcule a densidade de uxo magnetico
no centro da espira resultante da corrente induzida, e compare com a densidade de uxo magnetico
original responsavel por gerar a corrente induzida em t = 1, 0 segundos.
Solucao: A corrente induzida e V
emf
dividida pela resistencia distribuda da espira.
R =
1

A
=
1m
5, 8 10
7
4 0, 04m
(0, 0005m)
2
= 3, 5 m
d

B
dt
= 1
Wb
m
2
s
a
z
;
V
emf
=
_
d

B
dt
d

S = 1
Wb
m
2
s
_
0,04
0
dx
_
0,04
0
dy = 1, 6 mV
I
ind
=
1, 6
3.5
= 0, 46 A
Observamos que esta resposta nao depende do tempo.
(b) O campo no centro da espira, devido a um lado, e dado pela equacao:

H =
I
4
_
z

2
_
z
2
+
2
_
2
( a
z
) = 2, 59 a
z
A
m

B = 4
0

H = 13 T a
z
.
Este valor e muito menor do que o valor original (fonte).
PR 9.12: (Wentworth 5.11) O comprimento medio do n ucleo de nquel de um transformador e 16
cm e a area da secao transversal e igual a 1 cm
2
. Existem 30 espiras no lado primario e 45 espiras no
lado secundario. Se a corrente no primario e 1, 0 sen(20 10
6
t) mA, (a) calcule a amplitude do uxo
magnetico no n ucleo na ausencia da bobina de sada. (b) Considerando a presenca da bobina de sada,
calcule i
2
.
Solucao:
=
V
m
R
=
N
1
I
1

r
0
A
= 14 10
9
Wb
i
2
=
N
1
N
2
i
1
=
2
3
sen(20 10
6
t) mA
PR 9.13: Demonstre a equacao da tensao induzida V
rms
= 4, 44 f N
pico
.
PR 9.14: Demonstre a equacao da tensao de um indutor V
L
= L di/dt.
9. Campos Dinamicos - Lei de Faraday 67
PR 9.15: Usando uma tensao sinosoidal qualquer, com velocidade angular , demonstre a equacao da
impedancia de um indutor Z
L
= jL.
PR 9.16: Um condutor perfeito une as extremidades a e b de um resistor de 100, formando um circuito
fechado circular de raio R = 20cm. A densidade de uxo magnetico atraves do circuito e dada por

B = 0, 4sen (120t) a
z
T, perpendicular ao plano do circuito. Despreze a indutancia da espira e calcule
em funcao do tempo: (a) a diferenca de potencial sobre o resistor; e, (b) a corrente no circuito.
PR 9.17: Uma inducao magnetica de 1,5 T, e freq uencia de 60 Hz, esta incidindo numa chapa de cobre
com espessura de 15 mm. Qual e a inducao no outro lado? E qual e a defasagem em relacao `a incidente?
PR 9.18: Qual deve ser a espessura de uma chapa de alumnio para reduzir 99% dos campos em uma
blindagem? Qual seria a espessura de uma chapa de ferro com
r
= 500?
PR 9.19: Suponha que exista um campo oscilante na superfcie superior de uma arruela metalica. Con-
siderando: R
i
= 1.0cm, R
e
= 1.7cm, B
0
= 1.0T, = 0.57E + 08(m)
1
, f = 64.80Hz, e = 1.70mm, e

r
= 1.60; calcule: (a) a espessura de efeito pelicular dos campos ; (b) a potencia media dissipada por
efeito Joule na peca.
PR 9.20: Um disco de cobre, se encontra sob a acao da inducao

B(r, t) = B
0
(r/R)sent. Calcule:
(a) a densidade de corrente induzida J(r, t); (b) a corrente induzida I(t) no disco (B
0
= 0, 5T, =
5, 8.10
7
(m)
1
, R = 1cm, e = 1mm, f = 50Hz); (c) a potencia media dissipada no disco por efeito joule
com os dados numericos acima.
PR 9.21: Um condutor perfeito circular, de raio r = 20cm, colocado num uxo magnetico perpendicular
uniforme (no espaco) e variavel no tempo, une as duas extremidades de um resistor de 100. Sendo
B = 0, 4sen 120Wb/m
2
e, considerando desprezvel o uxo produzido pela propria corrente da espira
(desprezando a indutancia propria da espira), determine: (a) V
ab
(t); e, b) I(t).
PR 9.22: Os lados de uma espira quadrada, situada no plano z = 0, estao localizados em x = 0, 6
y = 0, 6m. Ha um campo magnetico

B = (0, 2 a
x
0, 4 a
y
+ 0, 4 a
z
) cos 2000 t Wb/m
2
, nesta regiao. Se
a resistencia da espira e 1k, qual e a corrente no sentido horario (quando observada da parte positiva
do eixo z) que se faz presente no circuito? Considere desprezvel o uxo o uxo produzido pela propria
corrente da espira, isto e, despreze a indutancia propria da espira.
PR 9.23: Uma espira condutora situada no plano z = 0 e limitada por x = 0, x = 0, 2m, y = 0, 3m, em
t = 0. A espira se desloca no sentido +a
x
com uma velocidade uniforme de 6m/s, e possui um pequeno
resistor de 20. Sabendo que existe um campo magnetico nao uniforme, porem constante no tempo
nesta regiao, representado por

B = 2x
2
y a
z
Wb/m
2
, esboce a curva que indica a potencia dissipada em
R como funcao do tempo, para o intervalo 0 < t < 100ms.
9. Campos Dinamicos - Lei de Faraday 68
PR 9.24: A partir de qual frequencia o efeito pelicular deve ser levado em consideracao para um o de
grate (condutividade 0,12 S/m) de 1 mm de diametro?
PR 9.25: Para diminuir as perdas ohmicas em instalacoes de alta potencia e redes de transmissao de
energia eletrica, se utilizam cabos de cobre grossos. Se a frequencia e de 60 Hz, a partir de que valor,
aproximadamente, nao adianta aumentar o diametro do cabo?
PR 9.26: Uma aruela de cobre (=5,8E7 (m)
1
) com espessura de 1mm tem raio interno de 1cm e
raio externo de 3cm, esta situada no plano xy. Um uxo magnetico variavel tem B
z
= B
0
cos t, sendo
B
0
= 0, 3T e f = 60Hz. Calcular a espessura de efeito pelicular e a potencia media dissipada na peca
por efeito Joule. (R:8,53mm e 103,88W)
PR 9.27: Um disco de cobre (=5,8E7 (m)
1
) com espessura de 1mm tem raio R = 1cm, esta situado
no plano xy. Um uxo magnetico variavel tem B
z
(r, t) = B
0
(r/R)sen t, sendo B
0
= 0, 5T e f = 50Hz.
Calcular a espessura de efeito pelicular; a corrente induzida I(t); e a potencia media dissipada na peca
por efeito Joule. (R:0,832W)
PR 9.28: Um anel, considerado liforme, possui secao S = 1mm
2
, raio R = 2cm, condutividade
=1,0E7 (m)
1
, esta situado no plano xy. Na superfcie interna deste anel existe um uxo magnetico
variavel dado por B
z
(r, t) = B
0
(1r/R)sen t. Calcular a corrente i(t) que ira circular no anel, quando
B
0
=1T e f =400Hz. (R:-83,78cos t A)
PR 9.29: Uma espira, considerada liforme, possui secao S = 1 mm
2
, raio R = 2 cm, condutividade
=5,8E7 (m)
1
, esta situado no plano xy. Na superfcie interna deste anel existe um uxo magnetico
variavel dado por
B
z
(r, t) = B
0
(r
2
/R
2
)(1 e
t/T
).
Calcular a energia dissipada na espira desde T = 0 ate T =0,01s, quando B
0
=0,2T. (R:0,364E-3J)
PR 9.30: Estime a capacitancia, C, a indutancia, l
s
, e resistencias serie, r
s
, e paralelo, r
p
, de um
capacitor de laminas de alumnio = 2, 8 10
6
cm) de w = 2cm de largura, t = 5m de espessura,
= 2m de comprimento separadas por um lme plastico ( = 30pF/m, = 1, 210
18
cm) de espessura
d = 10m. Note que a indutancia parasita depende de se os contatos forem soldados `as laminas de Al
pelos extremos ou pelos lados (apos enrolado); calcule l
s
nos dois casos.
PR 9.31: Um material magnetico laminado tem as seguintes perdas na freq uencia de 50 Hz: a) Histerese
(5 W/kg); e, b) Foucault (3 W/kg). Determinar as perdas na freq uencia de 200 Hz.
Solucao: Considerando o modelo de perdas
P(W/kg) = W
H
f +K
C
B
2
m
f
2
9. Campos Dinamicos - Lei de Faraday 69
obtemos
W
H
= 5/50 = 0, 1 J/kg
K
C
B
2
m
= 3/50
2
= 0, 066 Js/kg
e, substituindo para f = 200Hz
P = 0, 1 200 + 0, 066 200
2
= 20 + 2640 = 2660 W/kg
Observa-se um crescimento muito grande da perda por Foucault, que certamente inviabilizaria o projeto.

PR 9.32: Considerando que a energia dissipada por histerese seja a equacao:


W = W
h
+W
c
= K
S
B

+K
C
fB
2
onde K
S
= 3, 70W/kg (em 50,0 Hz) e o coeciente de Steinmetz, = 1, 600 e o expoente da inducao, e
K
C
= 0, 216E 02Js/kg/T
2
e o coeciente de perdas por correntes induzidas de Foucault. Determinar
a potencia dissipada em 0,00378 m
3
de material magnetico; quando esta operando em 68.10 Hz, com
inducao de pico igual a 0.8 T. A densidade do ferro e 7,652 kg/litro.
PR 9.33: Considerando a curva de histerese dada na tabela, determinar a densidade de energia magnetica
absorvida pelo material, em J/m
3
, para passar do ponto 1 ao ponto 8.
Ponto Induc~ao (T) H (A/cm)
1 .00 .00
2 .28 4.86
3 .73 9.60
4 .55 3.88
5 .37 -.58
6 .55 3.15
7 .83 7.07
8 1.10 9.82
PR 9.34: Uma bobina de um rele possui uma resistencia de 10 e uma indutancia de 50mH. Qual sera
o tempo necessario para atingir a corrente de atuacao de i = 0, 4A, quando este rele for ligado a uma
fonte CC de V = 5V?
Solucao: A soma das tensoes da indutancia L e da resistencia R e igual `a tensao da fonte
L
di
dt
+Ri = V
9. Campos Dinamicos - Lei de Faraday 70
Resolvendo esta equacao diferencial obtem-se
i =
V
R
(1 e

R
L
t
)
Substituindo-se os valores
0, 4 = 0, 5
_
1 e

t
LIGA
0,005
_
encontra-se o tempo t
LIGA
= 1, 11ms. Vemos, entao, que este sera o intervalo de tempo desde que
uma chave for ligada, ate o fechamento e/ou abertura dos contatos do rele. Entretanto, na pratica,
este tempo sera muito . . . muito bem diferente, pois existe a inercia mecanica, alem de que os valores
de resistencia, indutancia, e a corrente de atuacao (proporcional `a forca de reacao da mola), nao sao
constantes. Mas, a solucao da equacao diferencial permite-nos fazer uma analise qualitativa.
10. CORRENTE DE DESLOCAMENTO
PR 10.1: Explicar o funcionamento de um circuito ressonante LC.
Resposta: Um oscilador LC e composto por um indutor e um capacitor. Seu funcionamento se
baseia no armazenamento de energia em forma de diferenca de potencial e cargas eletricas no capacitor
e em forma de uxo magnetico e corrente no indutor.
O capacitor, em um tempo igual a zero, oferece uma impedancia proxima a zero ohms, o que permite
uir uma grande intensidade de corrente eletrica atraves do qual vai diminuindo ate que suas placas
tenham cargas eletricas positivas e negativas como permite o tamanho do mesmo e a permissividade
eletrica do isolante que tem entre as placas do capacitor.
Num instante o capacitor funciona como um isolante, ja que nao pode permitir a passagem de
corrente, e se cria um campo eletrico entre as duas placas, que cria a forca necessaria para manter
armazenadas as cargas eletricas positivas e negativas, em suas respectivas placas.
Por outra parte, num tempo igual a zero o indutor possui uma impedancia quase innita, que nao
permite o uxo de corrente atraves dele e, a medida que passa o tempo, a corrente comeca a uir,
criando-se entao um campo magnetico proporcional a magnitude da mesma. Passado um tempo, o
indutor atua praticamente como um condutor eletrico, pelo que a sua impedancia tende a zero.
Por estar o condensador e o indutor em paralelo, a energia armazenada pelo campo eletrico do
capacitor (em formas de cargas eletrostaticas), e absorvida pelo indutor, que armazena em seu campo
magnetico, porem a continuacao e absorvida e armazenada pelo capacitor, para novamente ser absorvido
pelo indutor, e assim sucessivamente. Isto cria um vai e vem de corrente entre o capacitor e o indutor.
Este vai e vem constitui uma oscilacao eletromagnetica, no qual o campo eletrico e o magnetico sao
perpendiculares entre si, o que signica que nunca existe os dois ao mesmo tempo, ja que quando o
campo eletrico esta no capacitor existe campo magnetico no indutor, e vice-versa.
PR 10.2: Explique como se formam as ondas eletromagneticas.
Resposta: Ondas eletromagneticas sao ondas que se formam a partir da combinacao dos campos
magnetico e eletrico, que se propagam no espaco transportando energia. O conceito de onda eletro-
magnetica foi postulado pelo famoso fsico escoces James C. Maxwell.

E dele o trabalho mais notavel
no campo do eletromagnetismo. Utilizando-se das leis experimentais de Coulomb, Faraday, Amp`ere e
tambem das suas proprias concepcoes, Maxwell construiu um conjunto de equacoes que resume os con-
hecimentos sobre o eletromagnetismo. Hoje conhecemos essas equacoes como as equacoes de Maxwell
e sabemos que foram elas que possibilitaram a existencia das ondas eletromagneticas. Essas equacoes
10. Corrente de Deslocamento 72
sao importantes para o estudo da eletricidade, assim como as leis de Newton sao importantes para a
mecanica.
Maxwell provou, atraves das suas equacoes, que o dist urbio eletromagnetico, o qual e causado pela
superposicao do campo eletrico e campo magnetico, apresenta todas as caractersticas ondulatorias e
que, sendo assim, a radiacao eletromagnetica tambem deveria sofrer os fenomenos da reexao, refracao,
difracao e a interferencia, assim como acontece em uma onda. Foi por esse motivo que o dist urbio
causado pelo campo eletrico e magnetico acabou por ser denominado de ondas eletromagneticas.
Os campos eletrico e magnetico que dao origem `as ondas eletromagneticas se propagam perpen-
dicularmente um ao outro.

E importante saber que, ao contrario das ondas mecanicas, a onda eletro-
magnetica nao necessita de um meio material para se propagar, pois o campo eletrico e o campo
magnetico podem ser estabelecidos na ausencia de materia, ou seja, no vacuo. Sendo assim, a radiacao
eletromagnetica pode se propagar no espaco vazio.
Um resultado muito importante obtido por James Maxwell foi o da velocidade com que as ondas
eletromagneticas se propagam. Utilizando suas equacoes e por meio de calculos, ele mostrou que no
vacuo, como tambem no ar, a velocidade de propagacao da radiacao eletromagnetica e igual a: c =
3, 0 10
8
m/s.
Essa descoberta foi muito importante porque esse valor coincide com a velocidade da luz, fato esse que
levou Maxwell a suspeitar que a luz era uma onda eletromagnetica. Ja no seculo XIX, os fsicos sabiam
que a luz se tratava de um fenomeno ondulatorio, mas nao sabiam qual a natureza dela. Hoje ja se sabe
que a suspeita de Maxwell e verdadeira: a luz e uma onda eletromagnetica. A descoberta da natureza
da luz foi um fato muito importante, o qual possibilitou a unicacao da

Otica e do Eletromagnetismo.
Como os fenomenos luminosos tem origem no eletromagnetismo, por consequencia a otica pode ser
considerada um ramo do eletromagnetismo e suas leis podem ser deduzidas a partir das equacoes de
Maxwell.
Maxwell morreu muito cedo e por isso nao viu suas ideias serem conrmadas. Foi somente no nal
do seculo XIX que o fsico alemao H. Hertz conseguiu, em laboratorio, obter ondas eletromagneticas
com todas as propriedades e caractersticas propostas por Maxwell. As experiencias que Hertz realizou
conrmaram as hipoteses elaboradas por Maxwell, conrmando, dessa forma, que a luz e uma onda
eletromagnetica.
PR 10.3: Deduzir a equacao diferencial da carga de um circuito RLC serie, alimentado com a tenao
v(t) = V
m
cos t, em regime permanente.
Resposta:
L
d
2
q
dt
2
+R
dq
dt
+
1
C
q = V
m
cos t
PR 10.4: Demonstrar a equacao da velocidade angular de ressonancia para o circuito RLC serie, e
desenhar o diagrama fasorial da corrente e das tensoes.
10. Corrente de Deslocamento 73
Resposta:
=
1

LC
PR 10.5: (Halliday, Ex.1, pg.1070) Um capacitor de capacitancia C = 1, 0 Farads, e carregado ate
atingir a tensao de 50 Volts. Desliga-se o gerador e liga-se uma bobina com indutancia L = 10 mH aos
terminais do capacitor, de modo a estabelecer-se oscilacoes no circuito LC. Qual sera a corrente maxima
na bobina, supondo-se o circuito com resistencia nula?
Resposta: De acordo com o princpio da conservacao da energia, a energia maxima armazenada no
capacitor deve ser igual `a energia maxima armazenada no indutor.
U
E
=
1
2
q
2
m
C
=
1
2
L i
2
m
= U
m
i
m
= V
0

C
L
= 0, 50 A
PR 10.6: (Halliday, Fig.38-8, pg.1084) Fazer um histograma com oito estagios de um ciclo de oscilacao
de uma cavidade ressonante eletromagnetica cilndrica, representando as energias eletrica e magnetica
armazenadas. Os pontos e cruzes indicam as linhas de inducao magnetica

B, e as linhas horizontais
representam

E.
PR 10.7: Inicialmente, lembramos da equacao H =

J, onde arma que o campo

H e produzido pela
densidade de corrente eletrica

J. Agora, considere uma cavidade eletromagnetica cilndrica, como na
Fig. 38.10, Halliday, pg.1094, e explique como e possvel ter campo magnetico no interior da cavidade
ressonante, sem a presenca de corrente eletrica no seu interior?
Resposta: As correntes de conducao estao nas paredes da cavidade, enquanto as correntes de deslo-
camento estao no interior da cavidade, na mesma direcao do campo eletrico.
PR 10.8: (Halliday P37-14) Somente 60 anos depois de Maxwell ter enunciando o conceito de corrente
de deslocamento, foi que, em 1929, M.R. Van Cauwenberghe conseguiu medir diretamente, pela primeira
vez, a corrente de deslocamento i
d
entre as placas de um capacitor de placas paralelas, submetido a
uma diferenca de potencial alternado v(t) = V
m
sen t. Ele usou placas circulares cujo raio efetivo era
de 40 cm e cuja capacitancia era 100 pF. A diferenca de potencial aplicada tinha um valor maximo de
174 kV na frequencia 50 Hz. Determinar: (a) Qual foi a corrente de deslocamento maxima obtida entre
as placas? (b) Por que foi escolhida uma diferenca de potencial tao elevada?
Solucao:
i
d
= C
dV
dt
= 2f C V
m
cos(2f t)
i
dmax
= 2f C V
m
= 5, 47 10
3
A
A corrente de deslocamento maxima e diretamente proporcional `a maxima diferenca de potencial apli-
cada. Um valor grande de V
m
produz um valor de corrente de deslocamento mais facilmente mensu-
ravavel do que com um V
m
menor.
10. Corrente de Deslocamento 74
PR 10.9: (Wentworth 5.22) Suponha que um campo vetorial qualquer seja

A = 3x
2
yz
3
a
x
. Verique
se o divergente do rotacional deste campo vetorial e igual a zero.
Solucao: Vamos fazer (

A), iniciando pelo rotacional, que e o produto vetorial ...


A =

a
x
a
y
a
z
/x /y /z
3x
2
yz
3
0 0

= 9x
2
yz
2
a
y
3x
2
z
3
a
z
. . . e agora, fazendo o seu divergente:
(

A) =

y
(9x
2
yz
2
)

z
(3x
2
z
3
) = 9x
2
z
2
9x
2
z
2
= 0
Comprovando que o divergente do rotacional de qualquer vetor e sempre nulo.
PR 10.10: (Wentworth 5.23) Suponha que um campo vetorial qualquer seja

A =
2
cos a
z
. Verique
se o divergente do rotacional deste campo vetorial e igual a zero.
Solucao: Vamos fazer (

A), iniciando pelo rotacional, que e o produto vetorial ...


A =
1

(
2
cos ) a

(
2
cos ) a

= sen a

2 cos a

. . . e agora, fazendo o seu divergente:


(

A) =
1

[(sen )] +
1

(2 cos )
=
1

2sen + 2sen = 0
Comprovando que o divergente do rotacional e nulo.
PR 10.11: (Wentworth 5.24) Duas placas planas paralelas com area de 60 cm
2
estao separadas com
um dieletrico com espessura 2,0 mm, e caracterizado pela constante dieletrica
r
= 9, 0.. Considerando
a tensao v(t) = 1, 0sen (2 10
3
t) Volts aplicada nas placas, determinar a corrente de deslocamento.
Solucao:
C =
0

r
S
d
= 239 pF
i
d
= C
dv
dt
= 239 10
12
2 10
3
cos(2 10
3
t) = 1, 5 cos(2 10
3
t) A
PR 10.12: (Wentworth 5.26) Um cabo coaxial com 1,0 m de comprimento tem um condutor interno
com diametro 2,0 mm e externo com diametro 6,0 mm, que sao separados por um isolante com constante
dieletrica
r
= 10, 2.. Considerando a tensao v(t) = 10 cos(6 10
6
t) mV aplicada nos terminais do
cabo, determinar a corrente de deslocamento entre os condutores interior e exterior.
Solucao:
C = 2
0

ln(b/a)
= 516, 27 pF
i
d
= C
dv
dt
= 97sen (6 10
6
t) A
10. Corrente de Deslocamento 75
PR 10.13: (Ulaby Ex 6.5) Um mau condutor caracteriza-se pela condutividade = 100 S/m e per-
missividade = 4
0
. Qual e a frequencia em que a amplitude das densidades de corrente de conducao

J
c
e deslocamento

J
d
sao iguais? Comentar o resultado.
Solucao:
|

J
c
| = |

E|
|

J
d
| =

D
t

= |j

E| = |

E|
=

= 2, 82 10
12
rad/s
f =

2
= 448 GHz
Assim, em baixas frequencias, normalmente se desconsidera a corrente de deslocamento.
PR 10.14: (Hayt 10.10a) Demonstrar que a relacao da amplitude da densidade de corrente de conducao
pela amplitude da densidade de corrente de deslocamento e /, quando o campo eletrico for E =
E
m
cos t. Assumir =
0
.
Solucao: Inicialmente, D = E = E
m
cos t. Entao a densidade de corrente de deslocamento e:
J
d
=
D
t
= E
m
sen t
Em segundo lugar, calculamos a densidade de corrente de conducao:
J
c
= E = E
m
cos t
Usando estes resultados encontra-se:
|Jc|
|Jd|
=


PR 10.15: (Hayt 10.11) Um capacitor formado por um cabo coaxial tem a = 1, 2 cm, b = 4 cm, e
material homogeneo com = 10
11
F/m, = 10
5
H/m, e = 10
5
S/m. Considerando o campo
eletrico

E(, t) = (10
6
/) cos(10
5
t) a

V/m, determinar: (a) A densidade de corrente de conducao; (b)


A corrente de conducao; (c) A densidade de corrente de deslocamento; (d) a corrente de deslocamento;
e, (e) o fator de qualidade do capacitor, que e a relacao |I
d
|/|I
c
|.
Solucao: (a)

J =

E = (10/) cos(10
5
t) a

A/m
2
(b) I
c
=
_ _

J d

S = 8 cos(10
5
t) A
(c)

J
d
=

D
t
=

E
t
=
1

sen (10
5
t) A/m
2
(d) I
d
= 2J
d
= 0, 8sen (10
5
t) A
(e)
|I
d
|
|I
c
|
=
0, 8
8
= 0, 1
10. Corrente de Deslocamento 76
PR 10.16: (Hayt 10.12) Seja uma linha de transmissao coaxial com b/a = e
2,5
,
r
=
r
= 1, e a
intensidade do campo eletrico

E = (200/) cos(10
9
t 3, 336z) a

V/m. Determinar: (a) A tensao V


ab
entre os condutores; e, (b) a densidade de corrente de deslocamento.
Solucao:
(a) V
ab
=
_
a
b
200

cos(10
9
t 3, 33z) d = 500 cos(10
9
t 3, 33z) V
(b)

J
d
=

D
t
=
200 10
9

sen (10
9
t 3, 33z) a

=
1, 77

sen (10
9
t 3, 33z) a

A/m
2

PR 10.17: Uma fonte de tensao V


0
sen t esta conectada entre duas esferas condutoras concentricas,
r = a e r = b, b > a, e a regiao entre elas esta preenchida com um material para o qual =
0

r
, =
0
e = 0. Encontre a corrente total de deslocamento atraves do dieletrico e compare-a com a corrente
da fonte, determinada a partir da capacitancia e metodos de analise de circuitos.
PR 10.18: A densidade de corrente de deslocamento e dada por 2 cos(t5z) a
x
A/m2 em um material
para o qual = 0, = 4
0
e = 5
0
.
1. Use a denicao da densidade de corrente de deslocamento para encontrar

D e

E.
2. Agora utilize a forma pontual da Lei de Faraday e uma integracao no tempo, para encontrar

B e

H.
3. Finalmente, utilize a forma pontual da lei circuital de Ampere para achar a densidade de corrente
de deslocamento. Qual deve ser o valor de ?
PR 10.19: Um material para o qual
r
= 1, 5 e
r
= 1 tem uma condutividade . Sendo E =
60 cos 105t a
x
V/m, ache: (a)

J
c
; (b)

J
d
; e, (c) a condutividade para a qual sao iguais amplitudes
das densidades de corrente de deslocamento e de conducao.
PR 10.20: Demonstre e comente as equacoes de Maxwell no vacuo em termos de

E e

B.
Resposta: No vacuo, as equacoes de Maxwell sao:


E =
1
c
d

B
dt
e


B =
1
c
d

E
dt
Estas equacoes denem uma onda eletromagnetica para os campos

E e

B situados a 90

e se deslocando
com velocidade c no espaco.
PR 10.21: Deduzir a equacao da onda TEM, com campo eletrico no eixo x.
Solucao: Vamos considerar uma Onda Eletromagnetica Transversal (TEM), onde

E e

H cam num
plano perpendicular a direcao de propagacao. Tomamos a onda que se propaga segundo o eixo z, o
10. Corrente de Deslocamento 77
campo

E tem somente componente na direcao de x e o campo

H tem componente somente na direcao
de y. E, como o meio e nao condutor

J = 0 a equacao de Maxwell


H =

D
t
se reduz a

H
y
z
=
E
x
t
(1)
A equacao de Maxwell obtida da lei de Faraday e


E =

B
t
e, procedendo como na equacao anterior temos:
E
x
z
=
B
y
t
como B = H, temos
E
x
z
=
H
y
t
(2)
(1) relaciona a derivada de

H em relacao a posicao com a derivada no tempo de

E.
(2) relaciona a derivada de

E em relacao a posicao com a derivada no tempo de

H.
Derivando (1) em relacao ao tempo e (2) em relacao `a posicao.

t
[
H
y
z
] =

2
E
x
t
2
e

2
E
x
z
2
=

t
[
H
y
z
]
Comparando as duas equacoes anteriores tem-se

2
E
x
t
2
=
1

2
E
x
z
2
Esta equacao diferencial descreve a variacao da grandeza E
x
(intensidade do campo eletrico) na posicao
e no tempo.

E chamada equacao da onda em E
x
ou Equacao de DAlembert.
PR 10.22: Calcule a razao entre as amplitudes das densidades de corrente de conducao e de deslocamento
para o campo eletrico E = E
0
cos t V/m no:
1. Cobre, = 5, 8 10
7
(m)
1
, =
0
, = 1.000rad/s;
2.

Agua destilada, = 2 10
4
(m)
1
,
r
= 80, = 1.000rad/s;
3. Polestireno, = 2 10
16
(m)
1
,
r
= 2, 53, = 1.000rad/s.
10. Corrente de Deslocamento 78
PR 10.23: MIT Open Course
1
Problem 1: The Displacement Current. What does the displacement
current term added to Amperes Law mean in practice? Discuss this in terms of an open surface and
the contour bounding that surface. Apply this meaning to the classic problem of a charging capacitor.
PR 10.24: MIT Open Course Problem 2: Comparing the Displacement Current to the Conduction
Current in a Wire. A wire with a circular cross-sectional area of 40 square millimeters carries a current
of 30 A. The resistivity of the wire is 2 X 10-8 Ohm-meter. What is the uniform electric eld in the
wire? If the current changes at a rate of 6000 Amps/second, what rate is the electric eld changing?
What is the displacement current density in the material? What is the magnetic eld 5 centimeters from
the center of the wire? Note that you must include both the displacement current and the conduction
current in this calculation. Is the contribution from the displacement current in the case important?
1
http://ocw.mit.edu/courses/physics/8-02sc-physics-ii-electricity-and-magnetism-fall-2010/the-displacement-current-
and-maxwells-equations/
11. ONDAS ELETROMAGN

ETICAS
PR 11.1: (Ulaby Ex 7.16) Converter os valores seguintes de ganho de potencia G para o n umero
natural em decibeis:
(a) G = 2, 3 G(dB) = 10 log 2, 3 = 3, 6 dB
(b) G = 4 10
3
G(dB) = 10 log 4 + 10 log 10
3
= 6 + 30 = 36 dB
(c) G = 3 10
2
G(dB) = 10 log 3 + 10 log 10
2
= 4, 8 20 = 15, 2 dB
PR 11.2: (Ulaby Ex 7.17) Converter os valores seguintes de ganho natural em decibeis para o ganho
de potencia G e ganho de tensao g.
(a) G(dB) = 23 dB 10 log G = 23 G = 199, 53 g =

G = 14, 13
(a) G(dB) = 14 dB 10 log G = 14 G = 0, 04 g =

G = 0, 2
(a) G(dB) = 3, 6 dB 10 log G = 3, 6 G = 0, 436 g =

G = 0, 66
PR 11.3: (Ulaby Ex 7.7) Uma onda plana uniforme com f = 10 MHz, esta situada num meio com
=
0
e
r
= 9. Obter: (a) a velocidade de fase; (b) o n umero de onda ou a constante de fase; (c) o
comprimento de onda; e, (d) a impedancia intrnseca do meio.
Solucao:
(a) u
p
=
1

=
c

r
= 10
8
m/s
(b) k = =

u
p
= 0, 2 rad/m
(c) =
2

= 10 m
(d) =
_

= 125, 67
PR 11.4: (Ulaby Ex 7.8) O fasor do campo eletrico de uma onda plana uniforme num meio sem
perdas, com impedancia intrnseca de 188,5 e

E
s
= 10 a
z
e
j4y
mV/m. Determinar (a) o fasor de
campo magnetico associado, (b) a expressao para o campo

E instantaneo, considerando =
0
.
Solucao:

E se propaga na direcao a
z
e esta polarizado no eixo a
y
. Assim,

H
s
= a
z


E
s
= a
x
E.
Logo,

H
s
= a
x
10 10
3
188.5
e
j4y
= 53 a
x
e
j4y
A/m

E(z, t) = a
z
10 cos(6 10
8
t 4y) mV/m
11. Ondas eletromagneticas 80
PR 11.5: (Ulaby Ex 7.9) Um meio com impedancia intrnseca = 100 tem campo magnetico

H
s
= (10 a
y
+ 20 a
z
)e
j4x
mA/m. Qual e o fasor do campo eletrico associado?
Solucao:

E
s
= a
z


H
s
= 100[ a
x
(10 a
y
+ 20 a
z
)]e
j4x
10
3

E
s
= ( a
z
+ 2 a
y
)e
j4x
V/m.
PR 11.6: (Ulaby Ex 7.10) Um meio com impedancia intrnseca = 100 tem campo magnetico

H
s
= a
y
(10e
j3x
20e
+j3x
) mA/m. Qual e o fasor do campo eletrico associado?
Solucao:

H
s1
= 10 a
y
e
j3x
mA/m.

H
s2
= 20 a
y
e
+j3x
mA/m.

E
s1
= a
z


H
s1
= 100( a
x
10 a
y
)e
j3x
= a
z
e
j3x
V/m.

E
s2
= a
z


H
s2
= +100( a
x
20 a
y
)e
j3x
= 2 a
z
e
j3x
V/m.

E
s
=

E
s1
+

E
s2
= a
z
(e
j3x
+ 2e
j3x
) V/m.
PR 11.7: (Ulaby Ex 7.11) O campo eletrico de uma onda plana e

E(z, t) = a
x
3 cos(t kz) + a
y
4 cos(t kz) V/m
Determine: (a) o tipo de polarizacao; (b) o modulo de

E; e, (c) o angulo de inclinacao.
Solucao: (a) Como as componentes de x e y estao em fase, a polarizacao tem que ser linear. Isto e
comprovado, obtendo-se = 0. Para a expressao dada,
x
=
y
= 0. Entao, =
y

x
= 0.

0
= tan
1
_
a
y
a
x
_
= tan
1
_
4
3
_
= 53, 1

sen 2 = (sen 2
0
)sen = 0 = 0
(b) |

E| = [E
2
x
+E
2
y
]
1/2
= 5 cos(t kz) V/m.
(c) Da parte (a),
0
= 53, 1

.
PR 11.8: (Ulaby Ex 7.13) Os parametros constitutivos do cobre sao =
0
= 4 10
7
H/m, =

0
= (1/36) 10
9
F/m, e = 5, 8 10
7
S/m. Considerando que estes parametros sao independentes
da frequencia, qual e a faixa do espectro eletromagnetico onde o cobre pode ser considerado um bom
condutor?
Solucao: Num bom condutor

> 100
f <

200
= 1, 04 10
16
Hz
11. Ondas eletromagneticas 81
PR 11.9: (Ulaby Ex 7.14) A partir de qual frequencia um solo seco, com
r
= 3,
r
= 1, e = 10
4
S/m, pode ser considerado como um meio dieletrico de baixa perda?
Solucao: Num dieletrico de baixa perda

< 0, 01 f > 60 MHz


PR 11.10: (Ulaby Ex 7.15) Qual e a amplitude percentual do campo

E para uma onda que se propaga
uma distancia de 3
s
, sabendo-se que o meio tem espessura de efeito skin ou pelicular
s
?
Solucao: Sabendo-se que
s
= 1/, pode-se escrever:
|E(z)| = |E
0
|e
z
= |E
0
|e
z/s
Para o ponto z = 3
s
:
|E(z = 3
s
)|
|E
0
|
= e
3
= 0, 05 = 5%
PR 11.11: (Hayt 11.6) O campo eletrico

E(z, t) = (25 a
x
30 a
y
) cos(t 50z) V/m esta presente no
vacuo. Determinar: (a) A velocidade angular; (b) A densidade de corrente de deslocamento; (c) O uxo
magnetico passando atraves de um retangulo denido por 0 < x < 1, y = 0, 0 < z < 1, em t = 0.
Solucao: (a) = u
p
= (3 10
8
) (50) = 15, 0 10
9
rad/s
(b)

J
d
(z, t) =

D
t
=
0
(25 a
x
30 a
y
) cos(t 50z)
= (3, 32 a
x
+ 3, 98 a
y
)sen (1, 5 10
10
t 50z) A/m
2
Usando a impedancia intrnseca, podemos obter o campo magnetico:

H(z, t) =
_
25

0
a
y
+
30

0
a
x
_
cos(t 50z) A/m

B(z, t) = (1/c)(25 a
y
+ 30 a
x
) cos(t 50z) Wb/m
2
=
_
1
0
_
1
0

B a
y
dxdz = 0, 44 nWb
PR 11.12: A principal caracterstica de uma onda eletromagnetica e a capacidade de transportar energia
entre dois pontos. A quantidade dessa energia por unidade de tempo e por unidade de area e chamada
de vetor de Poynting

S =

E

H. O campo eletrico de uma onda eletromagnetica plana e dado pela
expressao:

E = E
0
cos (

z t) a
x
+E
0
sen (

z t) a
y
V/m
em que E
0
e uma constante. Encontre o campo magnetico correspondente e o vetor de Poynting.
PR 11.13: Uma onda plana uniforme esta se propagando no polietileno (ver tabelas com suas pro-
priedades), com uma freq uencia de 9,4GHz. Se a amplitude do campo magnetico for 0,007 A/m e o
material for considerado sem perdas, ache: (a) a velocidade de propagacao; (b) o comprimento de onda;
(c) a constante de fase; (d) a impedancia intrnseca; e (e) a amplitude do campo eletrico.
11. Ondas eletromagneticas 82
PR 11.14: Dado

E = E
0
sen (t z) a
y
V/m no espaco livre, utilize as equacoes de Maxwell para
encontrar as equacoes dos campos vetoriais

D,

B e

H e tracar o graco dos campos eletrico e magnetico
em funcao de z, nos instantes t = 0 e t = T/12.
PR 11.15: Dado o campo magnetico

H = H
0
e
(tz)j
A/m no espaco livre, determine

E.
PR 11.16: No espaco livre:

D = D
0
sen (t +z) a
x
C/m
2
.
Use as equacoes de Maxwell para mostrar que
B =
D
0

0
sen (t z) a
y
T.
Esboce o graco dos campos para t = 0 ao longo de z, supondo que D
0
e sao maiores que zero.
PR 11.17: A partir das equacoes de Maxwell, obtenha a equacao fasorial-vetorial de Helmholtz dada
por

2
E
F
=
2

0
E
F
PR 11.18: O campo eletrico de uma onda plana uniforme no ar tem uma amplitude de 8 V/m no sentido
do eixo x. Se a onda se propaga no sentido do eixo z, com um comprimento de onda = 0, 5m, encontre:
(a) a freq uencia; (b) o perodo; (c) o valor de k se o campo for expresso por E
0
cos(t kz); e, (d) a
amplitude do campo magnetico.
PR 11.19: Calcule a razao entre as densidades de corrente de conducao e de deslocamento para o campo
eletrico E = E
0
sen (t) V/m, quando = 5, 8 10
7
S/m, =
0
, =
0
e uma freq uencia de 1,0MHz.
PR 11.20: A densidade de corrente de deslocamento em um certo material em que = 0, = 4
0
,
=
0
e dada por 2 cos(t 5z) a
x
A/m
2
. Encontre o campo deslocamento eletrico e o campo
magnetico.
PR 11.21: Sendo o campo eletrico

E = 200e
(4xkt)
a
y
V/m no vacuo, use as equacoes de Maxwell para
encontrar o campo magnetico e a expressao da constante k na equacao do campo eletrico.
PR 11.22: Sendo o campo eletrico

E = 200e
(4xkt)
a
y
V/m no vacuo, use as equacoes de Maxwell para
encontrar o campo magnetico e a expressao da constante k na equacao do campo eletrico.
PR 11.23: Sabendo-se que a radiacao eletromagnetica do sol que atinge a superfcie terrestre e da ordem
de 1350 W/m2, determine o valor da amplitude do campo eletrico E
o
a amplitude da inducao magnetica
B
0
.
11. Ondas eletromagneticas 83
Resposta: Com o vetor de Poynting e a impedancia intrnseca do meio, podemos calcular a amplitude
do campo eletrico e E
o
igual a 1, 01 10
3
V/m. Podemos calcular tambem a amplitude para o campo
magnetico, que e igual a 2,6 A/m. Usando a relacao B =
0
H encontramos a amplitude da inducao
magnetica B
0
= 3, 37 10
6
T.
PR 11.24: Uma linha de transmissao coaxial com superfcies condutoras em r
c
= 1mm, r
c
= 5mm e
z = 0, tem o ar como dieletrico e um campo

H = (0, 031/r
c
) cos 6 10
7
t cos 0, 2zu

A/m.
1. Ache a fem gerada ao longo do caminho (10
3
, 0
o
, 0) a (5 10
3
, 0
o
, 0) a (5 10
3
, 0
o
, 2) a
(10
3
, 0
o
, 0).
2. Ao longo de que segmentos do caminho temos
_

E d

= 0?
PR 11.25: Uma onda eletromagnetica tem a densidade de potencia de 2 W/m
2
, e se propaga num meio
com impedancia caracterstica de 200 . Qual e a amplitude dos campos eletrico

E e magnetico

H?
PR 11.26: Para a agua destilada temos permissividade relativa
r
= 50 e condutividade = 20( m)
1
.
Calcular:
1. constante de propagacao;
2. constante de atenuacao;
3. constante de fase;
4. comprimento de onda; e,
5. impedancia intrnseca do meio para a situacao onde = 10
11
rad/s.
PR 11.27: Uma linha de transmissao coaxial tem raio interno a = 1mm, raio externo b = 4mm, e
um dieletrico homogeneo com
r
= 2, 25,
0
= 1 e = 0. O campo eletrico e dado por

E =
(100/r
c
) cos(108 t z)u
rc
V/m.
1. Use as equacoes de Maxwell que envolvem o rotacional para determinar .
2. Ache

H.
3. Determine a densidade supercial de carga no condutor interno em funcao de , z e t.
4. Calcule a amplitude da corrente total de deslocamento no comprimento 0 z 1m.
PR 11.28: O campo magnetico proximo ao motor de um secador de cabelos varia senoidalmente com
uma frequencia de 60 Hz.
1. Mostre que a expressao simples

B = cos 260t a
x
T nao satisfaz `as equacoes de Maxwell no ar.
11. Ondas eletromagneticas 84
2. Ache o valor de k sabendo que

B = cos(260t ky) a
x
satisfaz `as equacoes de Maxwell.
PR 11.29: Um campo eletrico no vacuo e dado, em coordenadas esfericas por

E = (0, 1/r)sen sen (15
10
8
t 5r)u

V/m. Ache

H considerando que todos os campos variam senoidalmente no tempo com a
mesma freq uencia.
PR 11.30: O campo eletrico na origem e dado por 2 a
x
10 a
y
+ 3 a
z
V/m em t = 0.
1. Se a origem pertence a uma superfcie condutora perfeita, enquanto que para o material adjacente
`a origem
r
= 10,
r
= 2 e = 0, ache o modulo da densidade supercial de carga na origem em
t = 0.
2. Se
r
= 8,
r
= 3 e = 0, para x 0, enquanto que
r
= 3,
r
= 8 e = 0, para x > 0, ache |

E|
em t = 0 no ponto (0
+
, 0, 0).
PR 11.31: Michel Faraday mostrou que um campo magnetico variante, como por exemplo

B = B
0
e
kt
a
z
,
da origem ao campo eletrico
E =
1
2
kB
0
e
kt
u

.
1. Mostre que estes campos nao satisfazem `a outra equacao de Maxwell onde aparece no rotacional.
2. Sendo B
0
= 1 T e k =1/10s, nos estamos estabelecendo um campo magnetico razoavelmente
grande em 1s. Use a equacao que envolve


H para mostrar que a taxa com a qual B
z
deveria
(mas nao o faz) variar com r
c
seria cerca de 510
6
T por metro de espaco livre (vacuo) em t = 0.
PR 11.32: Sejam
r1
= 1,
r1
= 1 e
1
= 0 na regiao 1 (z < 0), enquanto que
r2
= 5,
r2
= 20 e

2
= 0 na regiao 2 (z > 0). Sabendo que o campo eletrico na regiao 1 e

E
1
= [60 cos(15 10
8
t 5z) +
20 cos(15 10
8
t +5z)] a
x
V/m, e que na regiao 2,

E
2
= Acos(15 10
8
t +5z) a
x
V/m, determine: (a) A;
(b)

H
1
; (c)

H
2
; (d) Mostre que

H
1
e

H
2
satisfazem `as condicoes de contorno necessarias em z = 0.
PR 11.33: Sejam
1
= 10
11
F/m,
1
= 4x10
6
H/m e
1
= 10
3
(m)
1
na regiao 1 (x < 0), enquanto
que
2
= 2
1
,
2
=
1
/2 e
1
= 4
1
, na regiao 2 (x > 0). Sabendo que

E
1
= (10 a
x
+ 20 a
y
+
30 a
z
) cos 10
9
t V/m no ponto P(0, 0, 0), ache: (a) E
n1
, E
t1
, D
n1
e D
t1
em P
1
; (b) J
n1
e J
t1
em P
1
;
(c) E
t2
, D
t2
e

J
2
em P
2
(0+, 00); (d) Use a equacao da continuidade como partida para mostrar que
J
n1
J
n2
= D
n2
/t D
n1
/t, e entao determine D
n2
, J
n2
e E
n2
.
PR 11.34: Temos superfcies condutoras perfeitas localizadas em r
c
= 5mm, r
c
= 20mm, z = 0 e
z = 50cm (coordenadas cilndricas). A regiao envolvida e um dieletrico para o qual
r
= 2, 25,
r
= 1 e
= 0. Nesta regiao

H = (2/r
c
) cos 2z cos 410
8
tu
p
hi A/m. Determine:
1. A densidade supercial de corrente em r
c
= 5mm, = 0, z = 5cm;
11. Ondas eletromagneticas 85
2.

E;
3. A densidade supercial de carga em r
c
= 20mm, = /2, e z = 25cm;
4. A densidade de corrente de deslocamento em r
c
= 10mm, = 0, 2, e z = 25cm.
PR 11.35: Um condutor lamentar se estende desde z = 5 ate z = 5m sobre o eixo x no vacuo, e
conduz uma corrente I = 4t A no sentido + a
z
. Ache e esboce

A(t) em (0, 0, 10) para 0, 1 t 0, 1 s.
PR 11.36: O campo eletrico no interior de uma linha de transmissao em forma de duas laminas condu-
toras muito longas e de pequena largura (2 mm) e afastamento (0,2 mm), pode ser considerado como
sendo dado por

E = 10
5
cos(10
9
t 4z)u
y
V/m.
Ache

A(x, y, z, t) se

A(x, 0, z, t) = 0.
PR 11.37: Estudar a polarizacao do campo:

E = 4/ 0 a
x
+ 3/ 90 a
y
Solucao:
E(t) = 4 cos t + 3sen t
E
x
= 4 cos t
E
y
= 3sen t
eliminando o tempo, vem:
E
2
x
16
+
E
2
y
3
= 1
elipse sobre a qual a extremidade do vetor vai car, ao girar. A onda e dita elipticamente polarizada.
Caso as amplitudes das duas componentes fossem iguais, seria circularmente polarizada. O sentido de
rotacao depende da defasagem ser mais ou menos noventa graus.
PR 11.38: Discutir a polarizacao da onda de campo eletrico dada por:

E = 5/ 90e
jz
a
x
+ 5/ 0e
jz
a
y
Solucao:

E(t, z) = 5sen (t z) a
x
+ 5 cos(t z) a
z
E
x
(t, z) = 5sen (t z)
E
y
(t, z) = 5 cos(t z)
eliminando (t z), temos o crculo:
E
2
x
+E
2
y
= 25
Para determinado z, a variavel (t z) cresce com o tempo portanto a rotacao.
11. Ondas eletromagneticas 86
PR 11.39: Estudar a polarizacao do campo:

E = 4/ 0 a
x
+ 3/ 0 a
y
Solucao: Passando para o domnio do tempo:
E(t) = E
x
a
x
+E
y
a
y
E
x
= X = 4 cos t
E
y
= Y = 3 cos t
eliminando o tempo vem:
Y =
3
4
X
Reta sobre a qual o campo vai car variando: a onda e dita linearmente polarizada.
PR 11.40: Se = 0, =
0
e =
0
, determine se os campos

E = 100sen 6 10
7
tsen z a
y
V/m

H = 0, 1328 cos 6 10
7
t cos z a
y
A/m
satisfazem as equacoes de Maxwell.
PR 11.41: (Wentworth 6.1) Partindo das Equacoes de Maxwell para um meio simples, livre de cargas,
obtenha a equacao de Helmoltz para

H.
Solucao:
(

H) =
_

E +

E
t
_
=

E +

t


E =


H
t

2

H
t
2
Usando a identidade vetorial (

H) =

H
2

H sabendo-se que

H = 0 obtem-se:

2

H =


H
t
+

2

H
t
2

PR 11.42: (Wentworth 6.5) Dado = 1, 0 10
5
S/m,
r
= 2, 0,
r
= 50, 0 e f = 10 MHz, calcule ,
, e .
Solucao:
=
_
j
r

0
( +j
r

0
) = 9, 4 10
3
+j2, 11 1/m
= 9, 4 10
3
Np/m e = 2, 11 rad/m.
=

j
r

0
+j
r

0
= 1880

257


11. Ondas eletromagneticas 87
PR 11.43: (Wentworth 6.8) Suponha no espaco livre,

H(x, t) = 100 cos(2 10
7
t x + /4) a
z
mA/m. Determine

E(x, t).
Solucao:

E
s
= a
p


H
s
= 120 a
x
0, 100e
jx
e
j/4
= 12e
jx
e
j/4
a
y

E(x, t) = 12 cos
_
2 10
7
t
2
30
x +

4
_
a
y
V/m
PR 11.44: (Wentworth 6.16) Em um meio com propriedades = 0, 00964 S/m,
r
= 1, 0,
r
= 100, 0
e f = 100 MHz, uma onda eletromagnetica se propaga na direcao +x com seu vetor campo na direcao
z. Determine a forma instantanea da intensidade do campo eletrico associada.
Solucao:

H
s
= H
0
e
x
e
jx
a
z

E
s
= a
p


H
s
= a
x
H
0
e
x
e
jx
a
z
= H
0
e
x
e
jx
a
y
=

j
+j
= 2664e
j30

=
_
j( +j) = 14, 8 +j25, 7 1/m

E(x, t) = 2, 66e
15x
cos
_
200 10
6
t 26x + 30

_
a
y
V/m
PR 11.45: (Wentworth 6.20) Calcular a profundidade pelicular a 1 GHz para os seguintes materiais:
(a) cobre; (b) prata; (c) ouro; e (d) nquel.
Solucao:
(S/m)
r
(mm)
Cobre 5,8E7 1 2,1
Prata 6,2E7 1 2,0
Ouro 4,1E7 1 2,5
Nquel 1,5E7 600 0,17

PR 11.46: (Wentworth 6.23) Em um material condutor nao magnetico

E(z, t) = 10e
200z
cos(2 10
9
t 200z) a
x
V/m
Determine

H(z, t).
Solucao: Como = o meio e um bom condutor. Com
r
= 1 pode-se isolar a condutividade na
espessura de efeito pelicular = :
=

2
f
0
= 10, 13 S/m
=

e
j45

11. Ondas eletromagneticas 88

H
s
=
1

a
p
10e
z
e
jz
a
x
=
10

e
z
e
jz
a
y

H(z, t) = 360e
200z
cos
_
2 10
9
t 200z + 45

_
a
y
mA/metro
PR 11.47: (Wentworth 6.28) Assuma agua destilada ( = 1, 0 10
45
S/m,
r
= 81,
r
= 1, 0)
preenchendo uma regiao z > 0. Na superfcie, temos

E(0, t) = 80 cos(2 10
9
t) a
x
V/m
Determine: (a)

E(z, t), (b)

H(z, t), (c) P
ave
e (d) a potencia passando por uma superfcie de 10 m
2
localizada em z = 1, 0 m.
Solucao: Tangente de perdas: // << 1 (dieletrico de baixa perda).
=

2
_

= 0, 0021 Np/m
=

= 18, 88 rad/m
=
_
/ = 41, 9

E(z, t) = 8e
0,0021z
cos
_
2 10
8
t 18, 8z
_
a
x
V/metro

H
s
=
1

a
p


E
s
= 191e
0,0021z
e
j18,8z
a
y
mA/metro

H(z, t) = 191e
0,0021z
cos
_
2 10
8
t 18, 8z
_
a
y
mA/metro

P
avg
=
E
2
x0

e
z
a
z
0, 761 W/m
2
P(10m
2
) = 7, 6 W
PR 11.48: (Wentworth 6.33) Dado

H(z, t) = 2 cos(t z) a
x
+ 6 cos(t z 120

) a
y
A/m. De-
termine a polarizacao e o sentido de rotacao.
Solucao:

E
s
= a
p


H
s
=
0
a
z

_
2e
jz
a
x
+ 6e
jz
e
j120

a
y
_
= 2
0
e
jz
a
y
+ 6
0
e
jz
e
j120

a
x

E(z, t) =
0
6 cos (t z 120

) a
x
+
0
2 cos (t z 180

) a
y
V/m
Esta e uma polarizacao elptica, em sentido horario.
PR 11.49: Uma onda propagante e descrita por y = 10sen (z t). Esbocar a onda em t = 0 e em
t = 1, quando ela avancou /8 considerando sua velocidade com 3 10
8
m/s e a freq uencia angular
= 10
6
rad/s. Repetir para = 2 10
6
rad/s.
PR 11.50: No espaco livre

E(z, t) = 10
3
cos(t z) a
y
V/m.
Obter H(z, t).
11. Ondas eletromagneticas 89
PR 11.51: No espaco livre E = 50 cos(t z)V/m. Calcular a potencia media que atravessa uma area
circular de raio 2,5 m pertencente a um plano Z constante.
12. LINHAS DE TRANSMISS

AO
PR 12.1: (Halliday, pg.1105) Aplica-se uma diferenca de potencial dada por V
0
= V
m
sen t entre os
terminais de uma longa linha de transmissao, suposta sem resistencia. A frequencia e igual a 3 10
9
Hertz. Escrever a expressao para V (t) num ponto P, distante 1,5 comprimentos de onda.
Resposta: A equacao geral de uma onda progressiva na direcao x pode ser escrita como:
V = V
m
sen (t x)
onde = 2/ e a constante de fase ou n umero de onda. Assim,
V = V
m
sen [t
2

(1, 5)] = V
m
sen t
PR 12.2: (Ulaby Ex 8.1) Calcular os parametros de uma LT a dois os, isolada com ar com distancia
de 2 cm, tendo raio de 1 mm em cada condutor. Os os podem ser considerados condutores ideais com
condutividade innita.
Solucao:
R
s
= R

= 0
c
= 0
L

=

0

ln
_
_
_
d
2a
_
+

_
d
2a
_
2
1
_
_
= 1, 2 H/m
G

= 0 porque = 0.
C

=

0
ln
_
_
d
2a
_
+
_
_
d
2a
_
2
1
_ = 9, 29 pF/m
PR 12.3: (Ulaby Ex 8.2) Calcular os parametros de uma LT em 1 MHz para um condutor coaxial
rgido, isolado com ar, tendo diametro interno de 0,6 cm e externo de 1,2 cm. Os condutores usados
sao de cobre.
Solucao:
R
s
=
_
f
c
/
c
= [ 10
6
410
7
/(5, 8 10
7
)]
1/2
= 2, 6 10
4

=
R
s
2
_
1
a
+
1
b
_
= 2, 08 10
2
/m
L

=

0
2
ln
_
b
a
_
= 0, 14 H/m
G

= 0 porque = 0.
C

=
2
ln(b/a)
= 80, 3 pF/m
12. Linhas de Transmissao 91
PR 12.4: (Ulaby Ex 8.3) Comprovar que V (z) = V
+
0
e
z
+ V

0
e
+z
e uma solucao da equacao da
onda.
Solucao:
d
2
V
s
(z)
dz
2

2
V
s
(z) 0
d
2
dz
2
_
V
+
0
e
z
+V

0
e
+z
_

2
_
V
+
0
e
z
+V

0
e
+z
_
0

2
V
+
0
e
z
+
2
V

0
e
+z

2
V
+
0
e
z

2
V

0
e
+z
= 0
PR 12.5: (Ulaby Ex 8.4) Uma LT com dois os paralelos, isolada com ar, tem os seguintes parametros:
R

= 0, 404 m/m, L

= 2 H/m, G

= 0, e C

= 5, 56 PF/m. Determinar, para a frequencia de operacao


de 5 kHz, os seguintes valores: (a) constante de atenuacao ; (b) constante de fase ; (c) a velocidade
de fase u
p
; e, (d) a impedancia caracterstica Z
0
.

s
=
_
(R

+jL

) (G

+jC

) = 3, 37 10
7
+j1, 05 10
4
1/m
= 3, 37 10
7
Np/m
= 1, 05 10
4
rad/m
u
p
=

= 3 10
8
m/s
Z
0
=
R

+jL

+j
= (600 j2)
PR 12.6: (Ulaby Ex 8.5) Uma LT sem perdas, tem = 20, 7 cm na frequencia de 1 GHz. Qual e o
valor
r
do material isolante?
Solucao:
=

0

r
=
_

_
2
=
_
c
f
_
2
= 2, 1
PR 12.7: (Ulaby Ex 8.7) Uma LT sem perdas de 50 e terminada com uma impedancia Z
L
=
(30 j200) . Calcular o coeciente de reexao de tensao na carga.
Solucao:
=
Z
L
Z
0
Z
L
+Z
0
=
30 j200 50
30 j200 + 50
= 0, 93

27, 5


PR 12.8: (Ulaby Ex 8.9) Se = 0, 5

60

e = 24 cm, encontre a localizacao do maximo e mnimo,


da onda estacionaria, mais proximos da carga.
Solucao:
l
Max
=

4
+

2
= 10 cm
l
Min
= l
Max


4
= 4 cm
12. Linhas de Transmissao 92
PR 12.9: Supoe-se que uma LT seja innitamente longa e que se aplica uma diferenca de potencial dada
por
V
s
= V
m
sen t
entre os terminais. A freq uencia f = /2 e igual a 3 10
9
Hz. Escrever a expressao de v(t) num ponto
P, distante 1,5 comprimentos de onda.
Solucao: A equacao geral de uma onda progressiva na direcao do eixo dos x pode ser escrita como:
v = V
m
sen (t kx)
onde k = 2/ e denominado n umero de onda. No ponto x = 1, 5, tem-se
v
P
= V
m
sen (t
2

1, 5) = V
m
sen (t 3) = V
m
sen (t)
Logo, v
P
sera sempre igual em modulo `a tensao da fonte, mas de sinal oposto.
PR 12.10: Calcular a tensao na carga para a LT, considerando que a onda eletromagnetica de tensao
se propaga com v = 3 10
8
m/s, para os seguintes casos de comprimento e freq uencia:
Solucao:
a) = 5m e f = 60Hz:
Da teoria de circuitos:
v
i
(t) = 5 cos(2ft)
Entretanto, o atraso de propagacao introduz uma defasagem :
= = (2/) = 2 10
6
rad = 0, 00036
o
onde
=
v
f
=
3 10
8
60
= 5 10
6
m
Assim,
v
i
(t) = 5 cos(2ft 0, 00036
o
)
b) = 1000km e f = 60Hz:
= =
2

=
2
5
rad = 72
o
v
i
(t) = 5 cos(2ft 72
o
)
c) = 5m e f = 10MHz:
=
v
f
= 3 10
8
10 10
6
= 30 m
= = (2 5/30) =

3
rad = 60
o
v
i
(t) = 5 cos(2ft 60
o
)
Conclusao: A teoria de circuitos, que e uma aproximacao da teoria mais geral de LTs, apresenta bons
resultados somente quando << .
12. Linhas de Transmissao 93
PR 12.11: (Wentworth 2.2) Calcule os parametros distribudos a 1MHz para um cabo com dois con-
dutores paralelos gemeos, formado por condutores de cobre AWG 26 com os centros separados, no ar,
por 1,0 cm.
Solucao: Usamos as formulas da Tabela 2.1 do livro, para dois condutores paralelos, para um cabo
AWG26 de cobre, que tem diametro 25,94 mil, e a = 7, 97 mils ou a = 202 m. Assim:
R

=
1

c
= 0, 729 /m
L

cosh

1
_
d
2a
_
= 1, 16 H/m
G

=

d
cosh

1(d/2a)
= 0
C

=

cosh

1(d/2a)
= 7, 12 10
12
pF/m
PR 12.12: (Wentworth 2.8) As especicacoes de um cabo coaxial RG-214 sao as seguintes:
(1) condutor interno de cobre com diametro de 2,21 mm;
(2) condutor externo com diametro interno de 7,24 mm;
(3) condutor externo com diametro externo de 9,14 mm; e,
(4) dieletrico Teon (
r
= 2, 10).
Calcule a impedancia caracterstica e a velocidade de propagacao para esse cabo.
Solucao:
Z
0
=
60

r
ln
_
b
a
_
= 49, 1
u
p
=
c

r
= 2, 07 10
8
m/s
PR 12.13: (Wentworth 2.10) Se uma potencia de 1,0 W e injetada em um cabo coaxial e 1,0 W de
potencia e medido a 100 m do ponto de injecao de potencia na linha, qual e a atenuacao da linha em
dB/m?
Solucao:
A = 10 log
_
1 10
6
W
1 W
_
= +60 dB
PR 12.14: (Wentworth 2.16) Uma fonte com impedancia de 50 alimenta uma LT de 50 que possui
1/8 de um comprimento de onda, terminada com uma carga Z
L
= 50 j25 . Calcule o coeciente de
reexao (complexo); a relacao de onda estacionaria (ROE); e a impedancia de entrada vista pela fonte.
Solucao:

L
=
Z
L
Z
0
Z
L
+Z
0
=
50 j25 50
50 j25 + 50
= 0, 24e
j76

= 0, 24

76

V SWR = ROTE = ROE = s =


1 +|
L
|
1 |
L
|
= 1, 64
12. Linhas de Transmissao 94
=
2

8
=

4
tan
_

4
_
= 1
Z
in
= Z
0
Z
L
+jZ
0
tan()
Z
0
+jZ
L
tan()
= 50
50 j25 +j50 1
50 +j(50 j25) 1
= 30, 8 j3, 8
PR 12.15: (Wentworth 2.19) A impedancia de entrada para uma LT sem perdas, de comprimento 30
cm e 100 de impedancia, operando em 2 GHz, e Z
in
= 92, 3 j67, 5 . A velocidade de propagacao
e 0,70c. Determine a impedancia da carga.
Solucao: Isolando-se Z
L
na equacao da impedancia de entrada de uma LT:
Z
in
= Z
0
Z
L
+jZ
0
tan()
Z
0
+jZ
L
tan()
Z
L
= Z
0
Z
in
jZ
0
tan()
Z
0
jZ
in
tan()
Z
L
= 50 +j0, 912 = 50 +j2 (2 10
9
) L L = 73 pH
Esta e uma indutancia muito baixa, de modo que Z
L

= 50 .
PR 12.16: (Wentworth 2.24) Suponha que uma LT da Fig.2.13 seja caracterizada pelos seguintes
parametros distribudos em 100 MHz: R = 5, 0 /m; L = 0, 010 H/m; G = 0, 010 S/m; e, C = 0, 020
nF/m. Se Z
L
= 50j25 , v
s
= 10 cos t V; Z
s
= 50 (impedancia de sada da fonte); e o comprimento
da linha e 1,0 metro, determine a tensao para cada extremidade da linha de transmissao.
Solucao: Inicialmente, calcula-se os parametros da LT:
=
_
(R

+jL

) (G

+jC

) = 0, 2236 +j0, 2810 m


1
Z
0
=

+jL

+jC

= 22, 3607

L
= 0, 4479 j0, 1908
Z
in
= Z
0
Z
L
+jZ
0
tan()
Z
0
+jZ
L
tan()
= 27, 2079 j15, 413
Fazendo a divisao da tensao da fonte V
SS
entre as impedancias Z
S
e Z
in
, tem-se a tensao V
in
na entrada
da LT:
V
in
= V
SS
Z
in
Z
in
+Z
SS
= 3, 97e
j18,2

V . . . v
in
(t) = 4, 0 cos(t 18, 2

) V
Entretanto, a tensao na entrada da LT pode ser escrita em funcao dos seus parametros:
V
in
= V
+
0
_
e

+
L
e

_
= V
+
0
(1, 504 +j0, 101) = V
+
0
_
1, 507e
j3,84

_
Isolando-se a amplitude da onda progressiva para a direita V
+
0
, encontra-se:
V
+
0
=
3, 97

18, 2

1, 507

3, 84

= 2, 63

22

= 2, 63e
j22

Volts
E, assim obtem-se facilmente a tensao V
L
nos terminais da carga:
V
L
= V
+
0
(1 +
L
) = 3, 85e
j29,6

Volts . . . v
L
(t) = 3, 85 cos(t 29, 6

) V
13. CARTA DE SMITH
A carta de Smith, inventada por Phillip H. Smith(1905-1987), e uma ajuda graca ou nomograma pro-
jetado para engenheiros eletricistas e eletronicos especializados em radio frequencia(RF), para ajudar na
resolucao de problemas com linhas de transmissao e circuitos associados. O uso da Carta de Smith tem
crescido ao longo dos anos e ainda e amplamente utilizado hoje em dia, nao so como ajuda para resolucao
de um problema, mas como um demonstrador graco do comportamento em uma ou mais freq uencias
de um dispositivo em RF, ou ainda uma alternativa ao uso deinformacoes tabulares ou numericas.
A carta de Smith pode ser usada para representar muitos parametros, incluindo impedanciase ad-
mitancias,coecientes de reexao, parametros de espalhamento, e gura de crculos de rudo, contornos
de ganho constante, regioes para a estabilidade incondicional, incluindo analise de vibracoes mecanicas.
O analisador de rede (HP8720A)(Fig. 13.1), e um dos instrumentos mais importantes em RF, porque
apresenta o graco de Smith.
Usando a carta de Smith para analisar circuitos de elementos concentrados
Os circuitos concentrados tem o comprimento de onda (da frequencia de operacao) muito maior do
que as dimensoes dos proprios componentes. O graco de Smith pode ser utilizado para analisar tais
circuitos, caso em que os movimentos de todo o graco sao gerados pelas impedancias (normalizadas)
e admitancias dos componentes com uma frequencia de operacao. Neste caso, o comprimento de onda
de escala na circunferencia graco de Smith nao e utilizado. O circuito da Fig. 8.1 sera analisado
utilizando a carta de Smith, com uma frequencia de funcionamento de 100 MHz. Neste frequencia o
comprimento de onda no espaco livre e de 3 m. As dimensoes dos componentes proprios estara na
ordem de milmetros de modo que o pressuposto de componentes concentrados sera valido. Apesar
de nao existir nenhuma linha de transmissao, como tal, uma impedancia do sistema deve ainda ser
denidas para permitir calculos de normalizacao e desnormalizacao e e uma boa opcao neste caso.
A analise comeca com o graco de Smith Z olhando para R1, apenas com outros componentes
presentes. Comoe o mesmo que a impedancia do sistema, isto e representado por um ponto no centro do
graco de Smith. A primeira transformacao e OP1 ao longo da linha de resistencia normalizada constante
neste caso, a adicao de uma reactancia normalizada de -j0,80, correspondendo a um condensador em
serie de 40 pF. Pontos com P de suxo sao no plano Z e pontos com suxo Q estao no planoY. Portanto
transformacoes P1 a Q1 e P3 a Q3 sao do graco de Smith Z para a Smith Y e Q graco transformacao
2 2 para Pe a partir do graco de Smith Y para o graco de Smith Z. A tabela seguinte mostra os passos
dados para trabalhar com os componentes restantes e transformacoes, retornando eventualmente, de
volta para o centro do graco de Smith e uma correspondencia perfeita de 50 .
13. Carta de Smith 96
Fig. 13.1: Network Analyser ou Analisador de Redes, tracando a Carta de Smith.
Fig. 13.2: Calculo da impedancia de entrada do circuito RLC, usando a Carta de Smith.
13. Carta de Smith 97
Tab. 13.1: Pontos da grade, e comparacao das solucoes analtica e numerica.
Transform. Y ou Z x ou y normal. Capac./ indut. Formula para resol. Resultado
O P
1
Z -j80 Capacitncia (Srie) j80 = j/C
1
Z
0
40 pF
Q
1
Q
2
Y -j1,49 Indutancia (Paralelo) j1, 49 = j/L
1
Y
0
53 nH
P
2
P
3
Z -j0,23 Capacitancia (Serie) j0, 23 = j/C
2
Z
0
138 pF
Q3 O Y +j1,14 Capacitancia (Shunt) +j1, 14 = jC
3
/Y
0
36 pF
PR 13.1: Determinar a impedancia de entrada da LT, sabendo que e terminada por uma impedancia
de 15 + j25. Um trecho de 27, 4cm de cabo coaxial de 50 ohms de impedancia caracterstica tem
a freq uencia de 146 MHz, uma velocidade de transmissao v = 0, 67c = 20 cm/nano segundos, e um
comprimento de onda = 20 1000/146 cm = 1, 37 metros.
Solucao: Procedimento: a impedancia de carga normalizada, calculada como:
Z
R
= 0, 3 +j0, 5 =
15
50
+j
25
50
p.u.
Como o comprimento da LT e 27, 4/137 = 0, 12, deve-se caminhar no sentido horario na carga de
Smith, signicando da carga para o gerador, ate a nova impedancia. Observarmos na carta que ela
mede 1, 6 +j1, 7 p.u. Entao, a impedancia de entrada, vista pela fontes, vale
Z
S
= (1, 6 +j1, 7) 50 = 80 +j85
Observa-se o efeito dramatico de alguns centmetros de cabo. A carta de Smith tambem fornece o
coeciente de reexao . Seu modulo corresponde ao raio da carta, e seu angulo ao angulo medido a
partir do ponto de maxima impedancia (direita).
PR 13.2: Uma L.T. tem constantes iguais a R = 1010
3
/m, G = 1, 510
6
S/m, L = 1, 210
6
H/m,
e C = 1, 5 10
9
F/m. Qual e a impedancia caracterstica, a atenuacao em dB/km, a defasagem em
rad/km, e a velocidade da onda, para a freq uencia f = 10 000Hz?
PR 13.3: Um cabo coaxial com raio interno igual a 0,5mm e raio externo de 2,8mm, tem
r
= 1 e

r
= 3, 1, e atua com uma freq uencia de 400MHz e uma impedancia de carga igual a 100. Qual e o
comprimento de onda e o coeciente de reexao desta linha?
PR 13.4: Uma linha de 50 sem perdas tem comprimento de 1, 5, uma impedancia de carga Z
L
= 60
e uma tensao na carga de 20

40
o
V. Calcular: (a) a potencia media fornecida `a carga; (b) o modulo da
tensao mnima na linha; e, (c) o modulo da corrente maxima na linha.
PR 13.5: Uma L.T. casada de 50 e alimentada por um transmissor que e equivalente a uma fonte de
amplitude 300V com resistencia interna de 50. O comprimento da linha e 5m e a freq uencia e de
30MHz. Obter as ondas de tensao e corrente, usando o cosseno como referencia.
13. Carta de Smith 98
PR 13.6: Uma L.T. corretamente terminada tem um coeciente de atenuacao de 2dB/km e um coe-
ciente de deslocamento de fase de 0,2rad/km. Qual a tensao no ponto de 10km, ao longo da linha, se
houver uma entrada de 1Vrms?
PR 13.7: Uma linha de transmissao casada, de comprimento 10 km, possui: R = 50/km, G =
1, 0S/km, L = 600H/km, e C = 40nF/km. Se a potencia de entrada da L.T. for 1W na freq uencia
de 1kHz, calcular: (a) a impedancia caracterstica da linha; (b) o coeciente de propagacao; (c) o coe-
ciente de atenuacao; (d) o coeciente de defasagem; (e) os modulos da tensao e da corrente nos lados
da fonte e de recepcao da linha; e, (f) a potencia recebida no nal da linha.
PR 13.8: Duas L.T. sem perdas, perfeitamente terminadas, tem resistencias caractersticas iguais a
200 e 400. Qual deve ser a resistencia caracterstica de uma secao de um quarto de comprimento de
onda para conecta-las a um casamento perfeito?
PR 13.9: Encontre a impedancia de entrada de uma linha cuja impedancia caracterstica vale 500,
tendo comprimento de 60 cm, e operando em um comprimento de onda de 2m. Para isso use a
impedancia de terminacao Z
L
= 25 +j50.
PR 13.10: (Wentworth 2.25) Posicione em uma Carta de Smith as seguintes impedancias de carga,
terminando uma LT de 50 : (a) Z
L
= 200 ; (b) Z
L
= j25 ; (c) Z
L
= 50+j50 ; e, (d) Z
L
= 25j200
.
Solucao:
PR 13.11: (Wentworth 2.26) Repita o problema Usando a Carta de Smith: Uma fonte com impedancia
de 50 alimenta uma LT de 50 que possui 1/8 de um comprimento de onda, terminada com uma
carga Z
L
= 50 j25 . Calcule o coeciente de reexao (complexo); a relacao de onda estacionaria
(ROE); e a impedancia de entrada vista pela fonte.
Solucao:
PR 13.12: (Wentworth 2.27) Uma LT sem perdas com Z
0
= 75 e 0, 690 de comprimento e termi-
nada em uma carga Z
L
= 15 +j67 . Utilize a carta de Smith para determinar: (a)
L
; (b) ROTE ou
ROE; (c) Z
in
; e, (d) a distancia entre a extremidade de entrada da LT e o primeiro maximo de tensao
a partir da extremidade de entrada da linha.
Solucao:
PR 13.13: (Wentworth 2.30) Em uma LT sem perdas de Z
0
= 50 , o ROTE e medido como sendo
3,4. Um maximo de tensao se localiza 0, 079 a partir da extremidade da carga. Determine a carga.
Solucao:
13. Carta de Smith 99
PR 13.14: (Wentworth 2.35) Uma rede de casamento de impedancia, utilizando um elemento reativo
em serie com um comprimento d de uma LT, e utilizado para casar uma carga de 35 j50 em uma
LT de 100 a 1 GHz. Determine o comprimento completo da linha d e o valor do elemento reativo se:
(a) um capacitor serie for utilizado; e, (b) um indutor serie for utilizado.
Solucao:
PR 13.15: (Wentworth 2.39) 2.37 Deseja-se casar uma carga de 170 em uma LT de 50 . Determine:
(a) A impedancia caracterstica exigida para um transformador de quarto de onda; e, (b) Quais os
comprimentos de linha e de stub exigidos para uma rede de casamento com stub em paralelo?
Solucao:
PR 13.16: (Wentworth 2.38) Uma impedancia de carga Z
L
= 200 + j160 e casada a uma linha
de 100 utilizando um sintonizador de Stub paralelo em curto. Determine a solucao que minimiza o
comprimento do Stub em curto.
Solucao:
PR 13.17: (Wentworth 2.39) Repita o problema 2.38 do Wentworth para um sintonizador de Stub
paralelo em aberto.
Solucao:
14. EQUAC

OES DE MAXWELL
PR 14.1: Sendo E = 200e
4xkt
a
y
V/m no vacuo, use as equacoes de Maxwell para encontrar k e

H,
sabendo que todos os campos variam com e
kt
.
PR 14.2: Um problema unidimensional de valores de contorno - Dois pontos extremos de um intervalo,
x = 0 e x = L, possuem valores conhecidos V (0) e V (L), respectivamente. A equacao diferencial que
rege a distribuicao da funcao V ao longo de x e
d
2
V
dx
2
= AV +B
Dividir o intervalo 0 x L em n+1 intervalos igualmente espacados de x = x
j+1
x
j
, j = 0, n+1,
e resolver o problema usando diferencas nitas.
Solucao: Substituindo a derivada d
2
V/dx
2
por diferencas nitas centrais, a equacao diferencial para
o ponto j pode ser representada por
V
j+1
2V
j
+V
j1
(x)
2
= AV
j
+B
Entao nos temos n equacoes lineares simultaneas, para os V
1
, V
2
, . . . V
n
valores da funcao V em cada
ponto j = 1, n. Apos multiplicar cada equacao por (x)
2
, este conjunto de equacoes pode ser escrito
como
V
2
2V
1
+V (0) = (AV
1
+B) (x)
2
V
3
2V
2
+V (1) = (AV
2
+B) (x)
2
. . . . . .
V
n
2V
n1
+V
n2
= (AV
n1
+B) (x)
2
V (L) 2V
n
+V
n1
= (AV
n
+B) (x)
2
(14.1)
Se V (0) = V (L) = 0, = 2 A(x)
2
e = B(x)
2
, o conjunto de equacoes resulta no sistema de
equacoes:
14. Equacoes de Maxwell 101
_

_
1
1 1
1 1
. . . . . . . . .
. . . . . . . . .
1 1
1
_

_
_

_
V
1
V
2
V
3
. . .
. . .
V
n1
V
n
_

_
=
_

. . .
. . .

_
(14.2)
A matriz dos coecientes e tridiagonal, e pode ser resolvida por metodos numericos rapidos para
ordem muito grande (na faixa de n = 1000). Enquanto que uma solucao analtica e viavel apenas para
problemas muito simples, as tecnicas numericas sao fundamentais para considerar nao linearidades nas
equacoes, tais como as advindas das propriedades dos materiais.
PR 14.3: Considere o desenho em 2D, onde tem-se duas superfcies equipotenciais S
1
e S
2
, e um uxo
que vai de S
1
para S
2
. Desenhar a forma das linhas equipotenciais e das linhas de uxo.
PR 14.4: Um bloco de material semi-condutor foi inserido entre duas placas metalicas, como mostra-se
na gura. Fazer um esboco das linhas equipotenciais e das linhas de uxo da corrente no semicondutor.
PR 14.5: Dois materiais condutores foram inseridos entre duas placas metalicas, como mostra-se na
gura. Fazer um esboco das linhas equipotenciais e das linhas de uxo da corrente no semicondutor.
PR 14.6: Metodo de Euler. Escolher um intervalo de tempo t = t
j+1
t
j
, fazer a aproximacao
dy
dt
= df(y
j
, t
j
) =
y
j+1
y
j
t
j+1
t
j
e resolver as equacoes diferenciais anteriores passo a passo no tempo, para um determinado intervalo
de tempo, e comparar os valores numericos e analticos. Solucao: Apesar desta tecnica numerica ser
muito simples, voce necessita de valores de t muito pequenos para obter uma precisao razoavel. Por
isto, esta tecnica nao e muito usada na pratica, quando se necessita de exatidao.
BIBLIOGRAFIA
[1] WENTWORTH, Stuart M., Eletromagnetismo Aplicado, Bookman, Porto Alegre, 2009.
[2] ULABY, Fawwaz. T., Eletromagnetismo para Engenheiros, Bookmann, Porto Alegre, 2007.
[3] HAYT, W. H., Eletromagnetismo, LTC Editora, 1983.
[4] KRAUS, J.D., CARVER,K.R.. Eletromagnetismo. Guanabara Dois. 1953.
[5] BASTOS, J. P. A., Eletromagnetismo para Engenharia: Estatica e Quase-estatica, Editora da
UFSC, 1989.

Anda mungkin juga menyukai